You are on page 1of 128

Page 1

CSAT Paper 2

Practice Set 1
I N
T .
1. There are 80 questions in this paper.
2. All questions carry equal marks.
Penalty for wrong answer
O S
deducted in your total marks.

I P
3. There are four alternative answers in every question. When you select a wrong answer, then 1/3rd mark of that question is

4. If any candidate gives more than one answer and one of them is true but it is treated as a wrong answer and the candidate

A R
is penalised for that and1/3rd marks will be deducted.

Directions (Q. Nos. 1-8) Read the following six passages and answer the questions that follow

R K
each passage. Your answers to these questions should be based on the passages only.

B Passage 1

S A
Towards the beginning of the 20th century, Charles Darwin formulated ‘Darwinism’ which then
followed by Lamarckism, Mendelism. Before that Galileo, Copernicus had dedicated much to the field
of science. After them, in modern era Einstein, Archimedes, Newton and many brand new scientists
KNOW THE TREND

presented us facts, technology, means for convenience and pooh-poohed at our dogmas, preconceived
notions. As science advanced, superstitions faded. Men became modernised, cultured and
sophisticated.

1. The author’s view point can be best summed up in which of the following statements?
1. Superstitions are prevalent because people have no scientific outlook that make them
ignorant about reality and they began to believe whatever comes their way.
2. With the advancement in science and technology men will be modern as it will help in the
reduction of dogmas which ultimately lead to the banishment of old beliefs and the irrelevant
notions.
Stage 1

3. Every scientist has worked with a view to fade away the preconceived notions, superstitions
and dogmas with almost a similar approach and inclination.
(a) Only 1 (b) Only 2 (c) Only 3 (d) Both 1 and 2

PDF DOWNLOAD FROM : WWW.SARKARIPOST.IN


Page 2

2 Practice Set 1

B Passage 2
The basic aim of Science is to look for reality and to find those factors, formally uncommon. A scientist
cannot accept the principles, usually because; they were accepted by others formerly. Great discoveries
are always coming from reasoning.
A lot of determination and observation are required. Civilisation, as it is today, also came from
numerous discoveries that were made in the field of Science. Man is a logical being. It is possible for
everyone to save himself from the odds. Only through this way, he can win and control everyone on the
globe. Only wisdom can help a man to conquer the World. Science without conscience is death of
the soul.

2. Which among the following options is the most logical corollary to the above passage?
(a) Science accepts facts and reasoning on the basis of its veracity and not on the grounds that it
has been a perceived notion earlier that has the elements of acceptance of others.

I N
(b) A lot of dedication and observation is required for the discovery of new ideas and concepts of
science; civilisation also follows the same pathways as that of the scientific discovery.
(c) Science without conscience is useless and dangerous, therefore, new discoveries can be said the

a civilisation.

T .
outcome of reasoning and logic that forms the basis of ideas and innovations to it as well as for

(d) Science and civilisation, both have a common and mutual path for their discoveries which is

Passage 3 O S
based purely on the reasoning and logic of our conscience.

I P
The mind has some difficulty in accepting the idea that there is something that is not explainable. Mind

R
has a very mad urge for everything to be explained. Anything that remains a puzzle, a paradox, goes on
troubling your mind. The whole of history of philosophy, religion, science, mathematics, has the same

A
root, the same mind – the same itch.

K
You may scratch yourself one way, somebody else may do it differently, but the itch has to be
understood. Mind can work freely when there is no mystery about existence. Ideas doesn’t come where

R
life is mysterious and gap thus formed cann’t be filled with reality. But it is not possible. Whatever you

S A
do, life is a mystery and is going to remain a mystery.

3. Which contrast does the writer want to establish about the working and functioning of the
human mind?
(a) Human mind does have a mad urge for explanation of the mysteries, which, when answered,
lead to a satisfaction but actually there is no final explanation whatsoever for these mysteries
and it will remain as such.
(b) Human mind believes that nothing is a mystery, but, in actual, life is a mystery despite the
conviction that it is understood and holds good to every aspect of the “itch” of the mind.
(c) The “itch” of the human mind is nothing but the belief that existence is not a mystery and it
could be understood well, but, the stark reality is that it is a mystery and will remain a mystery
forever.
(d) Either (b) or (c).

PDF DOWNLOAD FROM : WWW.SARKARIPOST.IN


Page 3

Stage 1 Know the Trend 3

B Passage 4
As the world sees success, it is the achievement of a social status, completing a goal, reaching an
objective or the achievement of an action in a specified set of time. The way God defines success is so
much different than the way the world does. Success to those in the world means achieving something
that is useful in this life on earth. Success as God sees, it is something that is achieved, that is of infinite,
eternal value in this life and in the life to come. That is a huge difference indeed. Defining success is
difficult and is in constant flux for most people. As you achieve more your goal posts move further away.
As you move towards your next goal, you aim for more and try harder and grow and develop. For this
reason, it’s tough to define success and ultimately impossible to achieve it and reach an end point.

4. Success has been described in many ways in the passage above. Which of the following
options best sums up?
(a) Success is the achievement of an individual’s own goal, social status and the achievement of an
objective that defines the completion of one’s wish.

I N
(b) Success is what we think to accomplish in the time to come and to complete that with flying
colors in the individual’s interest or for the interest of the community.

.
(c) Success is what is accomplished by an individual with his own effort and it serves the
community purpose without any self involved or without any hidden interest.

T
(d) Success is what one achieves with diligence and it helps one develop one’s reputation when that

Passage 5 O S
success is used by the other fellow members in the interest of the community.

I P
Adolescence is the best stage in life. You are finally growing up and reaching an age where you have lots

R
of choices, freedom and chances. The time your parents nagging on you, holding your hands while
crossing the street is now over. You take care yourself and have lot more chances to get out, make new

A
friends, joining sports teams, student councils and all other after school activities. Getting involved in

K
such activities increase your popularity, helps to make new friends. You can go to parties and have fun at.
There are lot other things like responsibilities, handling of peer pressure which increase with

R
adolescence. This might turn people off from thinking this as best stage in life. The choices makes great

S A
differences in life. A wrong choice can get you into wrong crowd and end up in trouble with law. You
don’t want to get do anything stupid for which you have to regret later on.

5. Adolescence age is a stage of opportunities as well as challenge. According to the passage


which of the following is not correct about adolescence?
(a) While making choice one may sometimes end up doing stupid things.
(b) At this stage one have to handle responsibilities and peer pressure.
(c) For not being end up wrong one makes lots of choices.
(d) Going to parties and having fun helps individual to increase popularity.

B Passage 6
Democratic institutions are devices for reconciling social order with individual freedom and initiative
and for making the immediate power of a country’s rulers subject to the ultimate power of the ruled.
The fact that, in Western Europe and America, these devices have worked, all things considered, not too
badly is proof enough that the eighteenth century optimists were not entirely wrong. Given a fair
chance, I repeat; for the fair chance is an indispensable prerequisite.

PDF DOWNLOAD FROM : WWW.SARKARIPOST.IN


Page 4

4 Practice Set 1

No people that pass abruptly from a state of subservience under the rule of a despot to the completely
unfamiliar state of political independence can be said to have a fair chance of being able to govern itself
democratically. Liberalism flourishes in an atmosphere of prosperity and declines as declining prosperity
makes it necessary for the Government to intervene ever more frequently and drastically in the affairs of
its subjects. Over-population and over-organisation are two conditions which deprive a society of a fair
chance of making democratic institutions work effectively. We see, then that there are certain historical,
economic, demographic and technological conditions which make it very hard for Jefferson’s rational
animals, endowed by nature with inalienable rights and an innate sense of justice, to exercise their
reason, claim their rights and act justly within a democratically organised society. We in the West have
been supremely fortunate in having been given a fair chance of making the great experiment in
self-government. Unfortunately, it now looks as though, owing to recent changes in our circumstances,
this infinitely precious fair chance were being, little by little, taken away from us.

6. According to passage which of the following option checks liberalism?


(a )
(b )
(c )
Reconciling social order with individual freedom
Political independence and democracy
Declining prosperity and increasing government intervention
I N
(d ) Over-population and over-organisation

T .
7. Consider the following statements regarding the primary purpose of the author in the passage
1.
2.
3.
Define the conditions for social order.

S
Explain a requirement and introduce a warning about a particular requirement.
Credit certain thinkers with foresight.
O
4.
P
Argue for the limitation of a certain form of Government.

I
Which of the above statement(s) do not reflect the author’s primary purpose?
(a) 1 and 4 (b) 1, 3 and 4 (c) 3 and 4 (d) Only 2

the passage
A R
8. Consider the following statements regarding the ‘infinitely precious fair chance’ mentioned in

R K
1. Jefferson’s rational animals.
2. An atmosphere of prosperity.
3. Organised democratic institutions.

S A
4. Reconciliation of social order with individual freedom.
Which of the above statements are inconsistent with the idea of fair chance?
(a) 1 and 4 (b) 1, 2, 3 and 4 (c) 3 and 4 (d) None of these

9. A dealer marks his goods so as to make 20% profit. But due to a fire in his warehouse, 5% of
his goods are damaged, also 25% became soiled and hence, they had to be sold at half their
cost. If he sells the remaining at the marked price, his net gain or loss is
(a) 2% gain (b) 4% loss (c) 3% gain (d) 3.5% loss

10. A sailor can row a boat 8 km downstream and return back to the starting point in 1 h 40 min. If
the speed of the stream is 2 km/h, then the speed of the boat in still water is
(a) 5 km/h (b) 10 km/h (c) 15 km/h (d) 20 km/h

11. Three wheels making 60, 36, 24 revolutions in a minute start with a certain point in their
circumference downwards. After how long will they first come all together again in the same
position?
(a ) 5 s (b ) 7 s (c ) 9 s (d) 10 s

PDF DOWNLOAD FROM : WWW.SARKARIPOST.IN


Page 5

Stage 1 Know the Trend 5

12. A class has a strength of 48 students, out of which 18 are boys. The students are to be seated in
rows such that the number of students in all the rows is equal. If none of the rows has both
boys and girls, then what is the maximum number of students sitting in one row?
(a ) 4 (b ) 6 (c ) 9 (d ) 8

13. Calculate the number of triangles in the given figure.

(a) 20 (b) 19 (c) 18 (d) 16

Directions (Q. Nos. 14-15) Examine the information given below and answer the questions that follow.

I N
In the network diagram given below, the figure represents the flow of natural gas through
pipelines between major cities A, B, C, D and E (in suitable units). Assume that supply equals
demand in the network (although not on individual nodes).
200
B
T .
A
700

x
125

C
375

675
O S
E 1400

500
200

D
I P 350

14. What is the number of units demanded in B?


A R 350

(a) 175

R K (b) 200 (c) 225

15. If the number of units demanded in C is 225, then what is the value of x?
(d) 250

(a) 975

S A (b) 75 (c) 775 (d) 950

16. A car during its journey travels 30 min at a speed of 40 km/h, another 45 min at a speed of
60 km/h and 2 h at a speed of 70 km/h. Find its average speed.
(a) 58 km/h (b) 63 km/h (c) 67 km/h (d) 71 km/h

17. I forgot the last digit of a 7-digit telephone number. If I randomly dial the final 3 digits after
correctly dialing the first four, then what is the chance of dialing the correct number?
1 1
(a ) (b )
1001 990
1 1
(c ) (d )
999 1000

18. Madhuri travels 14 km westwards and then turns left and travels 6 km and further turns left
and travels 26 km. How far is Madhuri now from the starting point?
(a) 180 km (b) 80 km
(c) 100 km (d) None of these

PDF DOWNLOAD FROM : WWW.SARKARIPOST.IN


Page 6

6 Practice Set 1

19. The yield versus fertiliser input is shown in the graph.


50 (B Maximum point)

C (Saddle point)

Yield
20
A
8 D

O
1 5 10 15
Fertiliser input

Consider the following statements based on this graph.


I. Yield is zero at B and C.
II. There is no yield with no fertiliser input.
III. The yield is minimum at D.
V. The yield is neither minimum nor maximum at C.
Which of the above statements are correct?
(a) I, II and IV (b) III and IV (c) II and III
I N
(d) I, III and IV

T .
20. The average speed of a train in the onward journey is 25% more than that of the return
journey. The train halts for one hour on reaching the destination. The total time taken for the

onward journey, is
(a) 45 km/h (b) 47.06 km/h
O S
complete to and fro journey is 16 h covering a distance of 800 km. The speed of the train in the

(c) 50.00 km/h (d) 56.25 km/h

I P
Directions (Q. Nos. 21-28) Read the following seven passages and answer the questions that follow
each passage. Your answers to these questions should be based on the passages only.

A R B Passage 1

R K
Reservation came with the caveat that its usefulness and validity would be examined every few years.
However, since the inception of class-based quotas, the term has morphed to ‘caste-based’ quotas. At

S A
one time, class may have been synonymous with caste. This is not the case any longer. But it doesn’t suit
any politician, or any government, to acknowledge this, since vote bank politics suggest that it is wise to
keep communities, rather than classes, happy. There are other ways of appealing to the economically
backward, after all. The thought behind reservation was that those who come from families which have
been educated for generations have an unfair advantage over those who are first-generation learners.
There is some credence to this idea. But in exploiting the political mileage that this allowed, our parties
have ensured that the principle is all but lost. When you have economically well-off communities, which
have been educated for generations, fighting in the Supreme Court for the removal of the ‘creamy layer’
rider, you know that there is something very wrong about the approach we have taken to reservation.

21. According to the passage which of the following option is correct about reservation
1. The provision for the validity of reservation is examined every few year.
2. It is also termed as class based quotas.
3. It is perpetuating due to vote bank politics.
4. It is based on educational and economic backwardness
(a) Only 1 (b) 1 and 3 (c) 2 and 4 (d) All of the above

PDF DOWNLOAD FROM : WWW.SARKARIPOST.IN


Page 7

Stage 1 Know the Trend 7

B Passage 2
It’s unsure whether they have heard about the Right to Food Act, but this band of women has managed
to ensure that no one from their village goes to bed hungry. Women self-help groups are one of rural
India’s success stories but this group in Sakdi village of Madhya Pradesh has become an agent of change,
literally. They started off as any self-help group, depositing a small part of their income in a common
pool, so that when someone has a pressing need for money, this can come handy.
But saving money was not enough. They realised that a number of people were going hungry. So, they
decided to do something about it. From pooling in money, they graduated to contributing a fistful of
food grains to a food bank. This, in turn, helps feed the hungry. When they collect more than 50 kg of
grains, they organise an event where they donate sacks of food grains to widows, landless families,
elderly people and those who have no source of income," said one of the residents of Sakdi village.

22. Which of the underlying options captures the essence of the Self Help Group in the most
satisfying way as mentioned in the passage?

I N
(a) To think of betterment of the society and its different aspects and not just the individual
person.

needed.

T .
(b) To think of betterment of the persons in need with an inclination to get the favor back when

(c) To support the persons who need a social identity especially from the backward section of the
society.

O S
(d) To assist the persons in need with a mutual contribution for mutual benefit and keeping the
view in mind that they must be viewed as a part of the society.

B Passage 3 I P
A R
In the age of internet online shopping is emerged as a popular option among masses. Time and cost
effectiveness makes shopping easier and attractive. Lower overhead cost allows online stores to offer

K
attractive prices by giving huge discount. Buyers also no longer have to drive all the way to shops to buy
things. Online shopping beside providing various options also allows to buy things from any part of the

R
world irrespective of its availability within the country. One can also compare price before purchasing.

A
This ease of shopping makes one a compulsive shopper.

S
Nowadays numerous advertisement has bombarded internet and spending more time on internet may
encourage more buying. It not only leads to debt but also cause security concerns as there is risk of your
financial information getting exploited by hacker.

23. Which one of the following assumptions goes against the author’s view with respect to online
shopping?
(a) Online shopping offers attractive prices which ultimately makes a person compulsive shopper.
(b) Online shopping being popular in nature, gets the attention of many people and hence they are
subjected to the barrage of advertisements which results into buying things.
(c) Online shopping offers cost effective deals and it tempts to make frequent purchases thereby
putting a person under debts.
(d) Despite continued purchases made by the people, they still apprehend of the security concerns.

PDF DOWNLOAD FROM : WWW.SARKARIPOST.IN


Page 8

8 Practice Set 1

B Passage 4
‘Police’ means a system of regulation for the preservation of order and enforcement of law, the internal
government of state. Law is the body of rules recognised by people as binding. Order means prevalence
of constituted authority. No doubt police is essential part of any society. Law and order is the first and
foremost pre-requisite of a civilised society .This objective is attained through policing. Under the
Constitution, ‘law and order’ is the direct responsibility of State Governments .The very first entry in
the state list of functions in the seventh schedule is the ‘public order’, followed immediately by police,
including village and railway police. Administration of justice is the third entry. Police is the
instrumentality for both the public order and administration of justice. ‘Law and order’ is the single
most important function of state on which its reputation depends; many of them are in worst condition.
Every state wants more funds but don’t want to share power rather they want it absolute and
unquestioned.

rational and logical way?


I N
24. Which of the following arguments put forward strengthen the author’s view about police in a

(a) Police itself should see to the law and order of the state with an intervention of the judiciary
whenever required.

T .
(b) For a civilised society the police must be efficient enough within the granted powers to deal with
the oddities of the state so that law and order remains intact.

administration of justice can trust them.

O S
(c) Police should make it strong enough without the law being in their favor so that the

(d) Police must know morality and how the laws are to be enforced in a civilised state for up
keeping the law and order.

B I P
Passage 5

A R
Moral policing, fundamentally, refers to the act of enforcing morality in individuals who engage
themselves in, so called, ‘immoral acts’ and also to make sure others too don’t end up doing such things.

R K
The term morality, in today’s time and age, has different meanings for different people and hence moral
policing has different repercussions for every person. As long as a person, consciously, intends on being
morally policed is fine but when vigilante groups consisting of hypocritical people themselves turn to

S A
free the society from the shackles of immoral behaviour, then the problem begins. There have been
innumerable cases of politicians, especially those belonging to the Right wing, who have publicly made
comments which suggest hostility towards people, who they think are immoral. On one hand they want
the public to aspire for a life which is being lived in the West, but, at the same time they feel that most
part of it is not suited to the Indian lifestyle. Apart from the lawmakers even the police have been
actively involved in upholding Indian values, the sad part being the way they go about doing so. Cases of
incarceration of people they presumed to be involved in immoral acts, fining couples for kissing or
holding hands in public or even attacking foreigners for not paying bribe as they thought they exceeded
the time limit for partying.

25. Which of the given arguments about moral policing does the author think is worst in the
context?
(a) When some of the so called moral people start correcting the so called immoral people.
(b) When a politician makes a public comment about moral policing suggesting hostility even when
he advocates a lifestyle so elegant.
(c) When policemen start getting involved in the correction process despite the fact law is not
allowing them to do so.
(d) When foreigners are attacked because they denied paying bribes to the corrupt policemen on the
ground that they have exceeded their time limit for partying.

PDF DOWNLOAD FROM : WWW.SARKARIPOST.IN


Page 9

Stage 1 Know the Trend 9

B Passage 6
The National River Linking Project (NRLP) formally known as the National Perspective Plan, envisages
the transfer of water from water ‘surplus’ basins where there is flooding to water ‘deficit’ basins
where there is drought/scarcity, through inter-basin water transfer projects. Digging further into the
term ‘surplus’ as per the Government, states that it is the extra water available in a river after it
meets the humans’ requirement of irrigation, domestic consumption and industries thereby
underestimating the need of the water for the river itself.
The term ‘deficit’ has also been viewed in terms of humans only and not from the river’s perspective,
which includes many other factors. The project could also create many water conflicts both at the state
and international level. The country is already reeling due to many inter-state water conflicts like the
Ravi-Beas Water Dispute between Punjab-Haryana-Rajasthan and the Cauvery Water Dispute between
Kerala-Karnataka-Tamil Nadu-Puduchery to name a couple.

26. Which of the following options require the rational attention from the government on a
primary basis with respect to the river linking project in the country?
I N
T .
(a) Government must prepare an outline for the transfer of water from the water surplus area to an
area that is water deficit in order to secure equitable distribution of water resources.
(b) Government should see to it that every state gets the proper amount of water for its agriculture,
industry and its population and then it should decide about the project.

O S
(c) Government must seek help from the state level administration in order to resolve the existing
conflict of water sharing which is the primary concern in this whole issue.
(d) Government should start its project and let the things come in the way and then try to resolve

P
them without bothering too much about the help from the states.

I
B

A R
Passage 7
Einstein’s concept of the universe as a four-dimensional space time continuum becomes plain and clear,

K
when what he means by ‘continuum’ becomes clear. A continuum is something that is continuous. A
ruler, e.g. a one-dimensional space continuum. Most rulers are divided into inches and fractions, scaled

R
down to one-sixteenth of an inch. Will it be possible to conceive a ruler, which is calibrated to a

S A
millionth or billionth of an inch. In theory there is no reason why the steps from point to point should
not be even smaller. What distinguishes a continuum is the fact that the space between any two points
can be subdivided into an infinite number of smaller divisions. A railroad track is a one-dimensional
space continuum, on which position can be described at any time by citing a single coordinate, i.e. a
station or a milestone.
The surface of the sea is a two-dimensional continuum and the two-dimensional continuum are latitude
and longitude. An airplane pilot guides his plane through a three-dimensional continuum. He has to
consider latitude, longitude and height above the ground. The space of our world is a three-dimensional
continuum. Just indicating its position in space is not enough while describing any physical event, which
involves motion. How position changes in time also needs to be mentioned. This can be done either by
means of a time table or a visual chart. Similarly, for the best picturisation of the flight of an airplane
from one location to another, a four-dimensional space time continuum is essential. The latitude,
longitude and altitude will only make sense, if the time coordinate is also mentioned. Therefore, time is
the fourth dimension. If a flight has to be looked at, it should be perceived as a continuous
four-dimensional space time continuum curve.

PDF DOWNLOAD FROM : WWW.SARKARIPOST.IN


Page 10

10 Practice Set 1

27. The significant feature of a continuum, according to the passage, revolves around
1. the divisibility of the interval between any two points.
2. an ordinary ruler’s caliber for marking.
Select the correct answer using the codes given below
(a) Only 1 (b) Only 2 (c) Both 1 and 2 (d) None of these

28. The purpose of this passage is to highlight the point that


(a ) plots and sea captains have something in common
(b ) stock market charts may be helpful to physicists
(c ) the fourth dimension is time
(d ) non-mathematicians are often afraid of the common places

Directions (Q. Nos. 29-33) Study the pie charts given below and answer the following questions.

I N
The break-up of the volume share in Mumbai of different car models sold by Maruti car company
for years 2002 and 2003 is shown in figure I. The break-up of the car market according to market
share by sales possessed by different car manufacturing in Mumbai for 2002 is shown in figure II.

T .
Total volume of car sales in the year 2003 in Mumbai is 4 lakh. In the year 2003, the break-up of
Mumbai car market (volumewise) is the same as the break-up in the year 2002.

Zen
Omni
10%

O SHM
Others
10%
Maruti
20%

Esteem
60%

I P
Maruti 800 15%

Daewoo
15%
40%

10%

A R
Figure I
Hyundai 25%
Figure II

Esteems were sold?


R K
29. If in the year 2002, the total number of cars sold in Mumbai was 2.5 lakh, then how many

S A
(a) 1 lakh (b) 0.1 lakh (c) 1.1 lakh (d) 1.01 lakh

30. If the total number of cars sold in Mumbai in the year 2002 is 4 lakh, then how many cars did
Hyundai and Daewoo sell together?
(a) 1 lakh (b) 1.2 lakh (c) 1.4 lakh (d) 1.6 lakh

31. If the total sales of cars in the year 2002 in India was 24 lakh and Mumbai constituted 5% of
India’s total car sales, then what was the sales of Maruti 800 in Mumbai?
(a) 25000 (b) 27000 (c) 28800 (d) 32500

32. If the total car sales in Mumbai grows by 15% in the year 2004 and if the composition of market
shares remains the same as given in figure II, then what is the number of cars sold by HM in the
year 2004?
(a) 59000 (b) 69000 (c) 49000 (d) 79000

33. In Mumbai, if sales of Maruti went up by 20% in 2004 over its value in the year 2003 while the
sales of all other manufacturer remained same, then what percentage of the total sales would
Maruti have in 2004?
(a) 22.22% (b) 33.33% (c) 44.44% (d) 55.55%

PDF DOWNLOAD FROM : WWW.SARKARIPOST.IN


Page 11

Stage 1 Know the Trend 11

34. Two persons are climbing up on two moving escalators which have 120 steps. The ratio of first
person’s speed to that of 1st escalator is 2:3 (steps). The ratio of second person’s speed to that
of second escalator is 3 : 5 (steps). Find the total number of steps they both have taken
together.
(a) 85 (b) 93 (c) 80 (d) 75

35. Examine the information given below.


˜ All the civil servants are loyal. All the loyal persons are patriotic.
Which of the following is not a valid conclusion regarding the above arguments?
(a) All the civil servants are patriotic (b) All the patriotic are civil servants
(c) Some patriotic are civil servants (d) Some loyal persons are civil servants

36. Out of a total 85 children playing badminton or table tennis or both, total number of girls in
the group is 70% of the total number of boys in the group. The number of boys playing only
badminton is 50% of the number of boys and the total number of boys playing badminton is
60% of the total number of boys. The number of children playing only table tennis is 40% of
I N
What is the number of girls playing only badminton?
(a) 17 (b) 14 (c) 16
T .
the total number of children and a total of 12 children play badminton and table tennis both.

(d) 18

37. Examine the information given below.


˜

O S
Some officers are responsible persons. No responsible person is rich.

(a) Some responsible persons are officers


(c) No responsible person is an officer
P
Which of the following is a valid conclusion regarding the above arguments?

I (b) Some officers are rich


(d) No officer is rich

A R
38. Read the passage carefully and answer the question given below it.
Many persons have been attracted towards smuggling due to huge profits involved in this

K
anti-national activity. Some of them became millionaires overnight. India has a vast coastline
both on the East and West Coast. It has been a heaven for smugglers who have been carrying

R
on their activities with great impunity. There is no doubt that from time to time certain

(a ) S A
seizures were made by the enforcement authorities, during raids and ambush but even
allowing these losses, the smugglers made huge profits.
The passage best supports the statement that
smuggling hampers the economic development of a nation
(b ) smuggling needs to be curbed
(c ) authorities are taking strict measures to curb smuggling
(d ) smuggling is fast increasing in our country owing to the quick profit it entails

39. Examine the information given below.


˜
All the faces of cubes are painted with red colour. The cubes are cut into 64 equal small cubes.
How many small cubes have only one face coloured?
(a ) 8 (b) 16 (c) 24 (d) 32

40. The traffic lights at three different road crossings change after every 48 s, 72 s and 108s,
respectively. If they all change simultaneously at 08 : 20 : 00 h, then they will again change
simultaneously at
(a) 08 : 27 : 12 h (b) 08 : 27 : 24 h (c) 08 : 27 : 36 h (d) 08 : 27 : 48 h

PDF DOWNLOAD FROM : WWW.SARKARIPOST.IN


Page 12

12 Practice Set 1

Directions (Q. Nos. 41-47) Read the following six passages and answer the questions that follow
each passage. Your answers to these questions should be based on the passages only.

B Passage 1
Traditional knowledge mining may seem too tedious nowadays, but it was the best way to mine
knowledge before the start of activities such as micro blogging. It involves a series of steps that will
eventually lead to the correct information being found. When mining knowledge bases for knowledge,
one would have to first of all understand the knowledge base they are looking for. Micro blogging has
become a growing trend, both in business and socially. It however has its benefits and limitations that
one should be aware of when doing it, especially when it’s for knowledge management. Micro blogging
offers an easy, convenient and instant way of communication. It may have its limitations but it is
growing by the minute, which should say something about its benefits.

the micro blogging?


I N
41. Which of the following statements is the most suitable corollary with respect to significance of

.
(a) Micro blogging is a modern approach of mining knowledge from the persons who are active on
the platform and it is growing tremendously with some limitations.

T
(b) Micro blogging has its own advantages and limitations like other social platform but it is the
instant way of communication.

communication simpler.

O S
(c) The exceptional growth of micro blogging clearly reflects the benefits it has and it has made

(d) Micro blogging is popular among the professionals and it is likely to gain charm in the time to
come.

I P
A R B Passage 2
While producing alternative energy, less greenhouse gas is emitted, thereby saving the environment from

K
climate changes such as global warming. Pollution is becoming a rising issue as greenhouse gas levels rise
in the atmosphere from long-term dependence on fossil fuels. Long-term effects of pollution include a

R
global increase in temperature which harms wildlife and also increase in sea levels.

A
This increase in temperature harms many different forms of life such as those of the polar bears. Their

S
homes are starting to melt away as the higher temperatures are disturbing their normal lifestyle. On the
contrary, renewable energy sources emit close to none when it comes to greenhouse gases. Cutting down
on harmful gas emissions will benefit the environment in the long-run and such adaptations to
renewable energy are the beginning steps to reducing pollution. The need to find new energy sources is
rising, making renewable energy sources a rising option. The future holds many challenges; one of them
is being secure in the supply of energy. With this, new sources for energy must be implemented in order
to replace the fossil fuels that are running out.

42. Which of the given options advocates author’s view of finding the alternative sources of
energy for future?
(a) There is an essence to reduce the use of fossil fuels to make the environment safe for living and
sustenance.
(b) The dependence on the fossil fuel should be reduced so that the abodes of the animals can be
protected.
(c) People should cut down the use of fossil fuels in order to bring the level of greenhouse gas down.
(d) The reserves of the fossil fuels are depleting fast so for the proper supply of energy alternatives
are to be found.

PDF DOWNLOAD FROM : WWW.SARKARIPOST.IN


Page 13

Stage 1 Know the Trend 13

B Passage 3
An important feature of the Indian Judicial System is that it’s a ‘common law system’. In a common law
system, law is developed by the judges through their decisions, orders, or judgements. These are also
referred to as precedents. Unlike the British legal system which is entirely based on the common law
system, where it had originated from, the Indian system incorporates the common law system along
with the statutory law and the regulatory law.
Another important feature of the Indian Judicial system is that our system has been designed on
the pattern of the adversarial system. This is to be expected since courts based on the common
law system tend to follow the adversarial system of conducting proceedings instead of the inquisitorial
system. In an adversarial system, there are two sides in every case and each side presents its
arguments to a neutral judge who would then give an order or a judgement based upon the merits of
the case.

system in the passage?


I N
43. What is the most rational assumption that the author has implied regarding the common law

judges.

T .
(a) This system allows the development of law based on the decisions, orders and judgments of the

(b) The Statutory Law and the Regulatory Law can be incorporated in the common law system
that makes it really sane.

S
(c) The system allows adversarial system and hence offers an equal chance to the parties concerned
in the case.
O
B Passage 4 I P
(d) It being the oldest system in the country, has an insight despite some limitations to oversee.

A R
To keep the prices of essential commodities under control, and within reasonable limits, the Indian
Government had constituted the Cabinet Committee on Prices and the Special Committee of

R K
Secretaries on Monitoring Prices. These bodies monitor the prices and supplies of essential commodities
regularly. Apart from these, the Department of Consumer Affairs monitors the prices of essential
commodities on a daily and weekly basis.

S A
Those commodities that are in short supply are imported. Though the government’s steps to check
inflation are laudable, these measures will have a positive impact on the prices only when they are
coupled with a massive drive against hoarders, black marketers and anti-social elements.

44. As per the passage which viewpoint of the author can be inferred from the passage?
(a) Inflation is a man made as well as a natural phenomenon that needs to be addressed by
government and public both especially in the segment of essential commodities.
(b) To combat the problems of inflation not only government action is needed but the public also
needs to raise a voice against those who hoard and black market the essential commodities.
(c) The public must support the government in the weekly or monthly monitoring process so that
the hoarders or black marketers can be booked under law.
(d) A mutual attempt of public and the government would help curb the situation of black
marketing and hoarding which is the only way out possible.

PDF DOWNLOAD FROM : WWW.SARKARIPOST.IN


Page 14

14 Practice Set 1

B Passage 5
Cancer is a group of diseases characterised by an uncontrolled growth of abnormal cells and it is the
result of an uncontrolled cell-cycle. If the spread of these abnormal cells is not controlled by radiation
therapy or other means, then cancer can cause death. Most cancers take the form of tumours, although
not all tumours are cancers. A tumour is simply a mass of new tissue that serves no physiological
purpose. It can be benign, like a wart or malignant, like cancer. Benign tumours are made up of cells
similar to the surrounding normal cells and are enclosed in a membrane that prevents them from
penetrating neighbouring tissues. A malignant tumour or cancer, is capable of invading surrounding
structures, including blood vessels, the lymph system and nerves.
These invading cells interfere with the normal functioning of different body cells and it is because of this
that death of the host tissue usually follows. In addition to spreading to surrounding areas, cancer can also
spread to distant sites by the blood and lymphatic circulation and so can produce invasive tumours in
almost any part of the body. In 2010, approximately 7500000 people in the India were diagnosed with

I N
cancer and approximately 5000000 died of the disease. Early screening for cancer is believed to be able to
drastically reduce the number of deaths due to the disease. Knowing what to look for when detecting
cancer, as well as knowing if you are in a high risk population, are two aspects of early prevention which

45. Which of the following can be inferred from the passage?


T .
have been proven to increase survival rates and lower the length and severity of treatments.

O S
1. Cancer past a certain point always results in death.
2. Women have a higher risk than men of contracting lung cancer.
3. Cancer is characterised by an uncontrolled cell-cycle and a result in the spread of tumours
throughout the body.

(a) 1 and 2 (b) 2 and 3


I P
Select the correct answer using the codes given below
(c) Only 3 (d) All of these

A R
46. Which of the following can be inferred from the passage?
1. Finding of tumours will result into cancer.

(a) 2 and 3
R K
2. Cancer is capable of invading the lymph system and nerves.
3. There are two aspects of early prevention of cancer.
(b) All of these (c) 1 and 2 (d) 1 and 3

S A B Passage 6
Social security is as old as society itself, but its forms have been changing according to the needs and the
level of social consciousness of the people. Before the industrial revolution, the requirements of social
security were met by institutions like the joint family, church, guilds and caste. The family was the first
line of defense and it constituted the original cell of security. Measures adopted by different societies for
protecting the needy individuals have been manifold. Beginning with individual acts of charity and
philanthropy, these devices progressed to include mutual benefit schemes, both formal and informal.

47. Which one of the following is the most suitable corollary in the context of social security
scheme?
(a) It is the universal approach of helping the persons who are socially unconscious and they do not
know about the measures of doing it.
(b) It is one of the oldest means of keeping the public interest and allowing them to be secure in the
down turns of the time.
(c) It is an old concept that keeps on varying in its forms with the advent of new time and the
social concepts.
(d) It is deep rooted in the civilisation itself and the change is seen when society needs a reform.

PDF DOWNLOAD FROM : WWW.SARKARIPOST.IN


Page 15

Stage 1 Know the Trend 15

48. Examine the following information and answer the question that follows.
Employed people
8
6 Backward people
11 3
17 5 7
Educated people

How many backward uneducated people are employed?


(a ) 3 (b ) 4 (c ) 5 (d ) 6

49. Examine the information given below.


˜ A set S contains the following elements: {7, 11, 15, 19, 23, x}. What is the value of x?
Two statements, labelled 1 and 2, are given below. You have to decide whether the data given
in the statements are sufficient for answering the question. Using the data given in the
statements, you have to choose the correct alternative.
Statements
I N
1. The elements are in arithmetic progression.
2. x is a prime number.

T .
(a) Statement 1 alone is sufficient but statement 2 alone is not sufficient to answer the question
asked

S
(b) Statement 2 alone is sufficient but statement 1 alone is not sufficient to answer the question
asked
O
P
(c) Both statements 1 and 2 together are sufficient to answer the question asked but neither
statement alone is sufficient to answer the question asked

I
(d) Each statement alone is sufficient to answer the question asked

A R
50. A train overtakes two persons who are walking in the same direction in which the train is
going at the rate of 2 km/h and 4 km/h and passes them completely in 9 s and 10 s,
respectively. The length (in metres) of the train is
(a) 70

R K
(b) 80 (c) 60 (d) 50

51. Mohan has coins of 50 paise, 25 paise and ` 1.50 in the ratio of 1 : 2 : 3. (Mohan stays in a

(a) 2000
S A
country where all are valid currency coins. Also, country where ` 1 equals 100 paise). How
many coins of 25 paise does Mohan have, if he got ` 6600 in all?
(b) 2200 (c) 2400 (d) 2600

Directions (Q. Nos. 52-53) Examine the information given below and answer the questions that
follow.
Nine varsity cricket players — G, H, I, J, K, L, M, N and O are to be honoured at a special
ceremony. Three of these players H, M and O are also varsity football players. Two of them K and
N are also basketball players of the varsity team. In arranging the seats, it was decided that no
athlete in two sports should be seated next to another two sport athletes.

52. Which of the following cannot sit next to M?


(a ) G (b ) J (c) G and J (d ) K

53. Before all athletes are seated, there are two vacant seats on either side of N. Which two athletes
may occupy these seats?
(a) G and K (b) G and L (c) J and H (d) L and O

PDF DOWNLOAD FROM : WWW.SARKARIPOST.IN


Page 16

16 Practice Set 1

54. Examine the following arguments.


˜ Some graduates are postgraduates. ˜ All postgraduates are professor.
˜ No scientist is a professor.
Which of the following conclusion(s) can be drawn from the above arguments?
Conclusions
1. Some scientists are graduates.
2. Some scientists are postgraduates.
3. No scientist is a postgraduate.
(a) Only 1 follows (b) Either 1 or 2 follows
(c) Either 2 or 3 follows (d) Only 3 follows
2
55. In a question, division is of the dividend and 2 times the remainder. If the remainder is 75,
3
then find the dividend.
(a) 85 (b) 145 (c) 225

I N (d) 65

56. Examine the information given below.


˜

T .
It is 8.00 pm. When can a boy get the next bus for Ranchi from Durgapur?
Two statements, labelled 1 and 2, are given below. You have to decide whether the data given

statements, you have to choose the correct alternative.


Statements
O S
in the statements are sufficient for answering the question. Using the data given in the

I P
1. Buses for Ranchi leave after every 30 min, till 10:00 pm.
2. Fifteen minutes ago, one bus has left for Ranchi.
(a )
(b )
(c ) R
Statement 1 alone is sufficient while Statement 2 alone is not sufficient to answer the question
Statement 2 alone is sufficient while Statement 1 alone is not sufficient to answer the question

A
Either Statement 1 or Statement 2 is sufficient to answer the question
(d )

K
Both Statements 1 and 2 together are sufficient to answer the question

R
Directions (Q. Nos. 57-58) Examine the information given below and answer the questions that
follow.
˜

˜
A
There is a group of 6 persons A, B, C, D, E and F seated around a circular table.

S
There are two females and four males in this group.
Both the females are married to two of the males of the same group. The remaining two males are
bachelors.
˜
A, a female, is sitting opposite to one of the bachelors, F. Her husband is not on either of her sides.
˜
Another female is sitting opposite to her husband, D.
˜
The two females are not sitting adjacently.
˜
On the immediate right of both the females is seated a bachelor.
˜
C has the same marital status as F while B is a father of two sons.

57. Who is the other female in the group?


(a ) E (b ) C
(c ) D (d) Cannot be determined

58. Moving clockwise, how many bachelors are there between A and her husband, if one starts
from the husband?
(a) Zero (b ) 2
(c ) 1 (d) Data inadequate

PDF DOWNLOAD FROM : WWW.SARKARIPOST.IN


Page 17

Stage 1 Know the Trend 17

59. I am facing South. I turn right and walk 20 m. Then, I turn right again and walk 10 m. Then,
I turn left and walk 10 m and then turn right and walk 20 m. Then, I turn right again and walk
60 m. In which direction am I with reference to the starting point?
(a) North (b) North-East (c) North-West (d) East

60. Seven men A, B, C, D, E, F and G are sitting in a row in that order from left to right and each
one is wearing a different coloured shirt among red, blue, yellow, green, violet, pink and black.
The person who wears red coloured shirt sits to the right of D but is not F. The person who
wears green coloured shirt sits to the left of C. G wears pink coloured shirt. F does not wear
yellow coloured shirt. C wears blue coloured shirt. If A wears black coloured shirt, then the
colour of the shirt worn by F is
(a) yellow (b) red (c) green (d) violet

Directions (Q.Nos. 61-67) Read the following four passages and answer the questions that

I N
follow the passages. Your answers to these questions should be based on the passages only.

B Passage 1
T .
O S
“The setting up of the Bharatiya Mahila Bank is a small step towards the economic empowerment of
women. It is also a reflection of our commitment to this cause. We are sure that the bank will fulfil the
objective with which it is being established, namely financial inclusion of women and providing them

I P
equal and easy access of financial services. Empowering the women folk of rural India has much to do
with their economic independence. A source of income for themselves is the first step towards their
liberation. The bank has tied up with Drishtee, an NGO that works with the National Skill

A R
Development Corporation, to run small shops with an initial capital of ` 25,000. It has disbursed loans
to seven small shops in Lucknow. Drishtee meanwhile has started the concept in Bihar with nine shops.

K
61. On the reading of the above passage which statement weakens the viewpoint of the author
about the setting up of Bhartiya Mahila Bank?

R
(a) This has been done in order to impart financial inclusion among women so that their

S A
empowerment can be insured.
(b) This has been a dedicated effort to provide the women especially from the rural India an easy
access to all the financial services.
(c) It has been set-up with a view to make the women financially independent which will be the
first step towards their liberalisation.
(d) This is a project that is targeting the women of the poor states in order to strengthen them.

B Passage 2
The gold monetisation schemes will tap household gold stocks and the sovereign bond scheme would
help reduce the physical demand of gold for investment. Customers can open a gold savings account
with a bank and earn tax-free interest on gold assets like bullion or jewellery. The Reserve Bank of India
has issued a circular for implementation of the gold monetisation scheme and banks are now putting in
place the requisite processes for implementation. Over the course of time this is also expected to reduce
the country’s dependence on the import of gold. Interest and principal payments on gold deposits under
the scheme will be denominated in gold. Banks can use the stock of gold they collect as part of their
Statutory Liquidity Ratio requirement, under which they have to make 21.5% of their investments in
government bonds.

PDF DOWNLOAD FROM : WWW.SARKARIPOST.IN


Page 18

18 Practice Set 1

62. The primary purpose of the gold monetisation scheme can be understood in the best way from
which of the following statements?
(a) This scheme is an approach to use the gold that is lying idle with the several household of the
country.
(b) This scheme aims to make Reserve Bank of India more responsible in terms of its function by
assigning the task of gold monetisation.
(c) This scheme targets at the good functioning of the banks and to enrich their capital so that
revenue can be geared.
(d) This scheme is meant for the buying of more government bonds by the common public and by
the financial institutions.

B Passage 3

N
To accelerate the efforts to achieve universal sanitation coverage and to put focus on sanitation, the
Prime Minister of India launched the Swachh Bharat Mission on 2nd October, 2014. The Mission

I
Coordinator shall be the Secretary, Ministry of Drinking Water and Sanitation with two Sub-Missions,

.
the Swachh Bharat Mission (Gramin) and the Swachh Bharat Mission (Urban), which aims to achieve
Swachh Bharat by 2019, as a fitting tribute to the 150th Birth Anniversary of Mahatma Gandhi, which

T
in rural areas shall mean improving the levels of cleanliness in rural areas. It has become a ‘Jan Andolan’

S
receiving tremendous support from the people. Citizens too have turned out in large numbers and
pledged for a neat and clean India. People have started to take part in it and are helping spread the
message of ‘Cleanliness is next to Godliness’.
O
I P
63. What rational and logical inference can be drawn from the reading of the above passage?
(a) People were unaware of the importance of cleanliness earlier and there was a need to make them
aware of it which leads to the mission.

A R
(b) To pay a real tribute to Mahatma Gandhi it was required that a mission like this comes into force.
(c) A kind of Jan Andolan is always a handy tool to make these kinds of projects a huge success.
(d) This mission will act as a catalyst in revolutionising the clean India mission which is an utmost

K
priority in the country.

R
S A B Passage 4
If the expansion of the universe is to stop, there must be enough invisible matter in the universe to exceed
the luminous matter in density by a factor of roughly 70. It can be calculated that the critical density of
matter needed to break the expansion and close the universe is equivalent to three hydrogen atoms per
cubic meter but the density of observable universe in the form of galaxies is only a fraction of this.
It has been known for some time that outside the bright nucleus of a typical spiral galaxy, luminosity
falls off rapidly with distance from the centre. If luminosity were a true indicator of mass, most of the
mass would be concentrated toward the centre. Outside the nucleus the rotational velocity would
decrease geometrically with distance from the centre, in conformity with Kepler’s law. Instead we have
found that the rotational velocity in spiral galaxies either remains constant with increasing distance
from the centre or increases slightly. This indicates that the fall off in luminous mass with distance from
the centre is balanced by an increase in non-luminous mass.
Our findings suggest that as much as 90% of the mass of the universe is not radiating at a wavelength
with enough intensity to be detected on the Earth. Such dark matter could be in the form of extremely
dim stars of low mass, of large planets like Jupiter or of black holes, either small or massive. While it has
not yet been determined whether this mass is sufficient to close the universe, some physicists consider it
significant that estimates are converging on the critical value.

PDF DOWNLOAD FROM : WWW.SARKARIPOST.IN


Page 19

Stage 1 Know the Trend 19

64. The authors propose all of the following as possibly contributing to the ‘missing matter’ in
spiral galaxies except
(a) massive black holes (b) small and dim stars (c) massive stars (d) large planets

65. It can be inferred from information presented in the passage that if the density of the universe
were equivalent to significantly less than three hydrogen atoms per cubic meter, which of the
following would be true as a consequence?
1. Luminosity would be a true indicator of mass.
2. Different regions in spiral galaxies would rotate at the same velocity.
3. The universe would continue to expand indefinitely.
Select the correct answer using the codes given below
(a) Only 1 (b) Only 2 (c) Only 3 (d) None of these

66. The passage suggests that the results of the author’s study, have changed their ideas about
which of the following characteristics of spiral galaxies?
1. The relative luminosity of different regions.
I N
2. The relative rotational velocity of different regions.
3. The relative distribution of matter in different regions.
Select the correct answer using the codes given below
T .
(a) Only 1 (b) Only 2
S
(c) 2 and 3

O
67. The author’s study indicates that, in comparison with the outermost regions of a typical spiral
(d) All of these

1. similar rotational velocity and higher luminosity.


2. lower rotational velocity and lower luminosity.
I P
galaxy, the region just outside the nucleus can be characterised as having

3. lower rotational velocity and higher luminosity.


4. higher rotational velocity and higher luminosity.
Select the correct answer using the codes given below A R
(a) Only 1

R K
(b) 2 and 3 (c) Only 3

Directions (Q. Nos. 68-70) Study the graph carefully to answer the questions that follow.
(d) 3 and 4

S A
Profit (in lakh) made by three companies over the years Profit = Income − Expenditure

9
Company A Company B

p
p Company C
p
Profit ( ` in lakh)

8 p
7 p p
6
5 p
4
3
2
1
0
2008 2009 2010 2011 2012 2013
Years

68. Profit made by company A in the year 2009 was what per cent of the total profit made by all
the three companies in that year?
(a) 31.25% (b) 28.24% (c) 21.43% (d) 36.25%

69. If the income of company A in the year 2012 was ` 1354300, then what was its expenditure in
that year?
(a) ` 921600 (b) ` 833500 (c) ` 648200 (d) ` 754300

PDF DOWNLOAD FROM : WWW.SARKARIPOST.IN


Page 20

20 Practice Set 1

70. What is the approximate average profit made by company A in all the years together?
(a) ` 398000 (b) ` 382000 (c) ` 483000 (d) ` 512000

Directions (Q. Nos. 71-72) Read the following information carefully and answer the questions
that follow.
1. A is mother of B. 2. C is sister of A. 3. D is father of C. 4. B is son of E.

71. Which of the following statement(s) is/are required to establish that E is a male?
(a) Only 4 (b) Both 2 and 4 (c) Both 1 and 4 (d) Both 2 and 3

72. How is D related to E?


(a) Father (b) Grandfather (c) Son-in-law (d) Father-in-law

Directions (Q. Nos. 73-76) Examine the information given below and answer the questions that
follow.

I N
Five males A, B, C, D and E and four females P, Q, R and S are to be divided into three debating
teams of three persons each. The teams are designated as the Saviours team, the Warriors team
and the Kings team.
1. C, D and E must be the captains of their teams.
T .
2.
3.
4.
A and B cannot be in the same team.
P and E cannot be in the same team.
R must be on a team with either A or D or both.
O S
5. R must be on the Saviours team.

I P
73. Which of the pairs of students could be on the same team as P?
(a) A and D

A R
(b) D and E (c) D and S

74. Any of the following pairs of students could be on the Saviours team except
(a) A and E (b) C and D (c) D and P
(d) Q and R

(d) D and S

of his team?
R K
75. If E is the only male in his team, then which of the following pairs could be the other members

S A
(a) P and Q (b) R and S (c) R and Q (d) S and Q

76. If A and E are on the Warriors team, then which of the following is not an acceptable
assignment of students for the other two teams?
Saviours team Kings team
(a ) B, D and R C, P and S
(b ) D, P and R B, C and S
(c ) R, D and Q B, C and P
(d ) Q, R and D S, C and B

77. An old person receives pension as ` 1000 in first year and after that 95% of the amount in the
preceding year. If the old man lives forever, how much pension does he get?
(a) ` 15000 (b) ` 10000 (c) ` 20000 (d) ` 1053

78. According to Sud, his age is greater than 25 year but less than 34 year but according to his
brother, his age is greater than 26 year but less than 32 year and according to his mother, his
age is less than 29 year. What is the average of all his probable ages, if all the above estimations
are correct?
(a) 27.5 yr (b) 28 yr (c) 27 yr (d) 28.5 yr

PDF DOWNLOAD FROM : WWW.SARKARIPOST.IN


Page 21

Stage 1 Know the Trend 21

79. Examine the information given below.


˜ Can Rahul retire from office A in January 2010, with full pension benefits?
Two statements, labelled 1 and 2, are given below. You have to decide whether the data given
in the statements are sufficient for answering the question. Using the data given in the
statements, you have to choose the correct alternative.
Statements
1. Rahul will complete 30 year of service in office A in April 2004 and desires to retire.
2. As per office A rules, an employee has to complete minimum 30 year of service and attain age
of 60. Rahul has 3 year to complete age of 60.
(a ) Statement 1 alone is sufficient while statement 2 alone is not sufficient to answer the question
(b ) Statement 2 alone is sufficient while statement 1 alone is not sufficient to answer the question
(c ) Either Statement 1 or Statement 2 is sufficient to answer the question
(d ) Both Statements 1 and 2 are sufficient to answer the question

80. Examine the information given below.


˜ How many New Year’s greeting cards were sold this year in a shop?
I N
T .
Two statements, labelled 1 and 2, are given below. You have to decide whether the data given
in the statements are sufficient for answering the question. Using the data given in the
statements, you have to choose the correct alternative.
Statements
1. Last year 2935 cards were sold in the same shop.
O S
(b )
P
2. The number of cards sold this year was 1.2 times that of last year.
(a ) Statement 1 alone is sufficient while Statement 2 alone is not sufficient to answer the question

I
Statement 2 alone is sufficient while Statement 1 alone is not sufficient to answer the question
(c )
(d )
R
Either Statement 1 or Statement 2 is sufficient to answer the question
Both Statements 1 and 2 are sufficient to answer the question

A
R K
S A

PDF DOWNLOAD FROM : WWW.SARKARIPOST.IN


Page 22

22 Practice Set 1

Space for Rough Work

I N
T .
O S
I P
A R
R K
S A

PDF DOWNLOAD FROM : WWW.SARKARIPOST.IN


Page 23

Stage 1 Know the Trend

Answer with Explanations


1. (d) Statement 1 and 2 best sum up the view point of the 9. (d) Let total number of goods = 100
author. The passage clearly indicates that superstitions and price per good be ` 100, then
are prevalent because people have no scientific outlook marked price = ` 120
and therefore they can’t decide what reality is and what is
Then, his total investment = v (100 × 100) = ` 10000
based on truth. But, with the advancement in science and
Price recovered = (25 × 50 + 70 × 120) = ` 9650
technology people shall be aware of it and thereby, shall
[Qdamaged 25 at 50% discount, 70 at 20% profit]
be able to decide truth.
So, his net loss = (10000 – 9650) = ` 350
2. (c) We know that science is a good servant but a bad master.
∴ Loss percentage =  × 100 = 3.5%
350
It clearly indicates that science must be used with a  1000 
conscience otherwise it will be useless and dangerous. On
this ground we can say that every new discovery is the 10. (b) Let the speed of the boat in still water be x km/h
outcome of logic and reasoning which also forms the and speed of stream be 2 km/h.
basis of innovations and civilisation both.
3. (a) The passage suggests that human mind is very inquisitive
and it always has a desire for explanation of the mysteries ∴
8
+
8
I
=1
N
Then, speed in upstream = (x − 2) km / h
Speed in downstream = (x + 2) km / h
40
=1 =
2 5
and when this is served well it gets satisfied with the notion
that the explanation was final. But the writer asserts that
what is mystery will remain a mystery ever. Therefore the


T .
x + 2 x −2 60
5x 2 − 48x − 20 = 0
5x − 50x + 2 x − 20 = 0
2
3 3

contrast lies here in this statement.


4. (c) According to the passage success is what one achieves
by the effort of one’s own but this is largely meant for the
O S ⇒ 5x (x − 10) + 2 (x − 10) = 0
⇒ (5x + 2 ) (x − 10) = 0
betterment of the society. Also there is no self or no
interest involved with the success.
5. (d) Option (d) is incorrect. Getting involved in making new
I P ⇒

∴ Required answer = 10 km / h
x = 10 or −
2
5

activities increases one’s popularity.


6. (c) Declining prosperity leads to more government
A R
friends, joining sports, student council and after school 11. (a) Time taken for one revolution by each wheel is 60 , 60 and
60
24
5 5
i.e. 1, , s, respectively.
3 2
60 36

and in this way checks liberalism.

R K
interventions in the life of people to get the things correct

7. (b) The ‘requirement’ in Statement 2 is the need for giving


5 5 5
∴ LCM of 1, , = = 5 s
3 2 1
12. (b) Number of boys in the class = 18

A
democracy a ‘fair chance’, which is what the author keeps
on reiterating.In the last sentence of the passage, he Number of girls in the class = 48 − 18 = 30

S
warns that the chance is being taken away. He is neither
arguing to limit democracy (eliminate Statement 4) nor is
he defining conditions for social order in general (eliminate
Statement 1). Statement 3 is too narrow to be the main
point of the whole extract.
13. (d)
HCF of 18 and 30 = 6
So, a row can have a maximum number of 6 students.

F
B

8. (d) Statement 1 is with reference to individuals with rights which E G C


A
have been bestowed upon the individual by nature and I
therefore these are rational animals. Hence, it is consistent
H
with ‘fair chance’ as democracy needs these individuals to
claim right according to the passage. D
Statement 2 is also consistent according to the second Number of triangles formed from single unit = 4 triangles
paragraph. Statement 3 is correct according to 1st line of Number of triangles formed from two units = 8 triangles
the 3rd paragraph as the problem is due to over organised Number of triangles formed from four units = 4 triangles
institutions. It can be inferred that organised institutions ∴ Total number of triangles = 4 + 8 + 4 = 16 triangles
would be appropriate. Statetment 4 is also consistent as it
is the basic idea of democatic institution acording to the 14. (b) From the figure, it would be 700 − (375 + 125) = 200.
1st line of the passage. Hence, all the statements are 15. (a) From the figure,
consistent.
125 + x − 200 − 675 = 225
⇒ x = 975

PDF DOWNLOAD FROM : WWW.SARKARIPOST.IN


Page 24

26 Practice Set 1

 30 × 40 +  45 × 60 + (2 × 70) Due to vote bank politics politicians are supporting
   
   60 
16. (b) Average speed = 60 reservation. So, option 3 is also correct. In the passage it
30 45 is mentioned that for 1st generation learner reservation is
+ +2
60 60 needed i.e. educational backwardness. It doesn’t talk
20 + 45 + 140 205 about economic backwardness.
= = × 60 km/h = 63.07 ≈ 63 km/h
30 + 45 + 120 195
22. (d) As mentioned in the passage, the SHG is thinking of
60 providing assistance to the persons in need. This help
17. (d) It is given that last 3 digits are randomly dialed. would come out of a mutual contribution and it will fairly be
Then, each of the digit can be selected out of 10 digits in for the mutual benefit of the society as a whole.
10 ways. 23. (d) People shop frequently online and they are quite used to it
3 but still there remains an apprehension regarding the
∴ Required probability =   =
1 1
 10  security concerns as they think that some fraudulent may
1000
do something with their credentials.
18. (a) The movement of Madhuri is as follows
N 24. (b) As obvious, police must remain within the stipulated power
14 km A (Starting point) but they must be efficient enough so that they can perform

N
their role in the best way and be able to maintain law and
6 km

6 km

14 km B 12 km C
W

S
E
society.

. I
order in the society which is the essence of the civilised

25. (d) The argument that the author thinks is worst is when the
foreign nationals are attacked for denying the payment of
26 km

In right angled ∆ ABC, by using Pythagoras theorem,


AC = (BC )2 + ( AB)2 = (12 )2 + (6)2 = 180 = 180 km
S T
bribe. Rests of the statements are also worst but abashing
a foreign national is highly undesirable.
26. (c) River linking project has been an ever cherished dream
Hence, Madhuri is 180 km far from the starting point.
19. (b) Yield rate is not zero at B and C. Despite of no fertilisers
P O and it is not a reality till the date because conflict
exists between or among the states over the water
distribution. Therefore the government must seek the help
input the yield is 20. So, both I and II statements are wrong
while III and IV are correct because yield is minimum at D

R
and the yield is neither minimum nor maximum at point C.
20. (d) Let the speed of the train during returning journey be
I from the states to resolve the conflict and then start
working on the project.
27. (a) The words ‘What distinguishes a continuum is the fact that
x km/h.
Then, speed during onward journey = x +
K 100
A
25x 5x
=
4
km/h
the space between any two points can be sub-divided into
an infinite number of smaller divisions’ indicate that one of
the significant features of continuum is the divisibility of the
interval between any two points.
Distance covered in onward jouney =

Q Total time taken =


A
Covered distance
R
800
2
= 400 km 28. (c) The whole passage revolves around the concept of
dimensions and it starts as well as ends with the mention
of four-dimensions. The last sentences of the passage

S Speed

∴ Time taken by train in onward journey =

400
400
5x / 4
clearly indicate that the fourth dimension is time.
29. (b) Given, total number of cars sold = 2.5 lakh
∴ Sales of Maruti = 40% of 2.5 lakh
and time taken in returning journey = Hence, sales of Esteem =10% of sales of Maruti
x = 10% of 40% of 2.5 lakh
According to the question, = 01
. lakh
400 400 320 400
+ = 16 ⇒ + = 16 30. (d) Given, total number of cars sold = 4 lakh
5x / 4 x x x
Total percentage of Hyundai and Daewoo sold
⇒ 16x = 720
= (25 + 15)% = 40%
∴ x = 45 km/h
Total number of cars sold together by Hyundai and
5 × 45
So, the speed of train in the onward journey = Daewoo = 40% of 4 lakh = 1.6 lakh
4
= 56.25 km/h
31. (c) Total sales of cars in India = 24 lakh
Total sales of cars in Mumbai = 5% of 24 =1.2 lakh
21. (b) Option 1 is mentioned at the beginning of passage.
Total sales of Maruti = 40% of 1.2 lakh = 0.48 lakh
Option 2 is wrong as it is ‘caste based quotas’ not
‘class-based’. ∴Total sales of Maruti 800 = 60% of 0.48 lakh
= 60% of 48000 = 28800

PDF DOWNLOAD FROM : WWW.SARKARIPOST.IN


Page 25

Stage 1 Know the Trend 27

32. (b) Total car sales in Mumbai in 2003 = 4 lakh 7


Ratio of boys and girls = x : x = 10 : 7
10
Total car sales in Mumbai in 2004 = 115% of 4 = 4.6 lakh 10
Number of boys = × 85 = 50
∴Number of cars sold by HM = 15% of 4.6 lakh 17
= 0.69 lakh = 69000 and number of girls =
7
× 50 = 35
10
33. (c) Maruti sales in the year 2003 = 40% of 4 lakh = 16
. lakh
Number of boys (playing only badminton)
Increase in Maruti sale in the year 2004 = 20% of 1.6 lakh
50
= 0.32 lakh = × 50 = 25
100
Thus, Maruti sales in the year 2004 60
Number of boys (playing badminton) = × 50 = 30
= 1.6 + 0.32 = 1.92 lakh 100
and total sale = 4 + 0.32 = 4.32 lakh ∴ Number of boys (playing badminton and table tennis)
∴Percentage share of Maruti = 
1.92 
 × 100 = 44.44% = 30 − 25 = 5
 4.32 
Number of children (playing badminton and table tennis)
34. (b) Steps covered by first person = 2 × 120 = 2 × 120 = 48 = 12
(2 + 3)

Steps covered by second person = 3 ×


120
(3 + 5)
120
5

I N
∴ Number of girls (playing badminton and table tennis)
= 12 − 5 = 7
So, children (playing only table tennis) =
40
× 85 = 34
=3×
8
= 45

∴ Total number of steps taken by both the persons


= (48 + 45) = 93
.
Number of boys (playing only table tennis)

T = 50 − 30 = 20
100

Number of girls (playing only table tennis) = 34 − 20 = 14


35. (b) ‘All the civil servants are loyal’ can be represented as

Loyal Civil servants


O S
Number of girls (playing only badminton)
= 35 − 7 − 14 = 14

‘All the loyal persons are patriotic’ can be represented as

I P
37. (a) ‘Some officers are responsible persons’ can be
represented as

Officers Responsible
Patriotic

On combining the above two, we get


Loyal

A R persons

‘No responsible person is rich’ can be represented as

Civil
R K
Loyal
Responsible
persons
Rich

servants

S A
Patriotic
From the above diagram, Conclusions (a), (c) and (d) can
On combining the above two, we get

Responsible
persons
Rich

be drawn, but as it can be seen that ‘Some patriotic are Officers


civil servants but not all’, so option (b) is a valid
(i)
conclusion.
36. (b) 5 boys Officers

Badminton Table tennis Responsible


persons Rich
25 boys 20 boys
12 (ii)
14 girls 14 girls
So, option (a), i.e. some responsible persons are officers
is valid in both the figures. Rest options (b), (c) and (d) are
7 girls not valid.
Total children = 85 38. (d) The passage above is based on the increase in smuggling
Let the number of boys be x. activities due to huge profits involved in it. Hence, the
70x 7 x passage best supports the option (d).
∴Number of girls = =
100 10

PDF DOWNLOAD FROM : WWW.SARKARIPOST.IN


Page 26

28 Practice Set 1

39. (c) 47. (c) As the passage suggests that social security is a very old
Red concept but it has kept on changing with the need and
situation as per the requirement. Hence, this is the most
suitable corollary in the context.
Red
48. (c) To find the number of backward uneducated people that
4 cm Red are employed, we have to find the common region of both
4 cm the circles, excluding the triangle. ‘5’ shows that particular
region.
4 cm
49. (c) Statement 1 The elements are in arithmetic progression.
These are 64 small cubes, n = 3 64 = 4
So, x could either be 3 or 27. So, this statement alone is
Number of small cubes having only one face coloured insufficient to answer the question.
= (n − 2 )2 × Number of faces = (4 − 2 )2 × 6 = 24 Statement 2 x is prime i.e. x could be any prime number.
40. (a) Let us first calculate LCM of 48, 72 and 108 This statement alone is also insufficient to answer the
question.
2 48, 72, 108
2
2
2
24, 36,
12, 18,
6, 9,
54
27
27
N
Combining the two statements, we know that x could take
only two values from Statement 1, i.e. 3 or 27; from

I
Statement 2, we know that x is prime.
∴ 3 is the only value that satisfies both the conditions.
3
3
3
3,
1,
1,
9,
3,
1,
27
9
3
Then,
.
50. (d) Let us assume the speed of the train be x km/h.

T
(x − 2 ) ×
9
= (x − 4) ×
10

1, 1,

∴ LCM of 48 s, 72s and 108 s


1

O S⇒

3600
9x − 18 = 10x − 40
x = 22 km/h
Thus, distance travelled = (22 − 4) ×
3600

1000
= 5 m/s
= 2 × 2 × 2 × 2 × 3 × 3 × 3 = 432s
Thus, the second time the three lights will change after
432 s = 7 min 12 s
I P and time =10 s
So, length of train = 5 × 10 = 50 m
3600

Hence, next time the three lights will change


simultaneously at

A R
08 : 20 : 00 + 00 : 07 : 12 = 08 : 27 : 12 h
51. (c) The ratio of the number of coins is 1:2:3 for the 50 paise,
25 paise and `1.50 coins, respectively. In terms of
monetary value, the ratio becomes
(1 × 0.5) : (2 × 0.25) : (3 × 1.5)

K
41. (a) Micro blogging can be described in the best way by option
(a). It is a modern tool of evacuating knowledge from the

R
user as well as it is growing with a pace. It has its own
merits and demerits. Hence, statement (a) holds well in
which equals 0.5 : 0.5 : 4.5 or 1 : 1 : 9
∴  th of the total value comes from 25 paise coins.
1
 11 
the context.

S A
42. (a) The new sources of energy should be found out soon
because it will make the environment safe for living and will
cut down the amount of greenhouse gases produced by
i.e.
1
11
× 6600 = ` 600 is in the form of 25 paise coins.

∴Total number of 25 paise coins =


600
0.25
= 2400

burning of fossil fuels thereby ensuring a balance in Solutions (Q. Nos. 52-53)
ecosystem.
From the given information, it is clear that H, M, O, K and N
43. (c) The most rational assumption is that our law system allows play two games and therefore cannot be seated next to
the adversarial system which offers an equal chance to another.
put their cases in the front.
52. (d) Since, H, M, O, K and N cannot be seated together.
44. (b) One of the major reason for inflation is black marketing Therefore, K cannot sit next to M.
and the hoarding. The Govt. is trying hard to check the
53. (b) Any two cricket players from G, I, J and L, can sit on either
menace but only this will not be enough. So, public
side of N. Hence, G and L can occupy these seats.
awareness must be there to raise a strong voice against it
just in order to check it. 54. (d) Statement two + Conversion of statement three gives the
conclusion ‘no postgraduate is a scientist’. [Q A+E = E],
45. (c) The use of the word ’always’ makes Statement 1 incorrect. which on conversion gives Conclusion 3. So, Conclusion 3
Statement 2 is not given in the passage.
follows and thus Conclusion 2 does not follow. Again,
46. (a) Statement 1 cannot be substantiated from the passage. Statement one + no postgraduate are not scientist.
Statements 2 and 3 are mentioned in the passage. [QI + E = 0]
Hence, Conclusion 1 does not follow.

PDF DOWNLOAD FROM : WWW.SARKARIPOST.IN


Page 27

Stage 1 Know the Trend 29

55. (c) Given, remainder =75 62. (a) The primary purpose of the gold monetisation scheme
2 lies with the fact that it is aimed at using the gold that is
∴ Divisor = 2 × 75 = 150 and of the dividend = Divisor lying idle with the households in the country.
3
3 63. (d) It is well understood that cleanliness is a personal
∴ Dividend = × 150 = 225
2 discipline and people are aware of it. The truth is that
people are mainly concerned with personal hygiene
56. (d) Statement 2 reveals that the previous bus had left at 7:45 pm.
and not with the community hygiene. Therefore it is
As given in the Statement 1, the next bus would leave after
required that a mission like this should be in force.
30 min i.e. at 8:15 pm. So, both statements are needed to
Hence, (d) concludes better.
answer the question.
64. (c) The authors do not include massive stars in their list of
Solutions (Q. Nos. 57-58) possible explanations for missing matter.
The complete arrangement is as follows
65. (c) An inference is drawn from the stated information. This
Here, Male question refers to the first paragraph, where the authors
A
Female explain that the critical density of matter needed to
C D break the expansion and close the universe is
Bachelors

I N
equivalent to three hydrogen atoms per cubic meter. If
the density is significantly less, then the universe will not
‘close’ but continue to expand indefinitely. So, the

E
F
B

T .
correct answer is Statement 3.
66. (c) The authors did not change ideas about luminosity.
However, the authors changed their ideas about both

57. (a) E is the other female.


O S rotational velocity and the distribution of matter. So, the
correct answer is option (c).
67. (a) The region has similar rotational velocity and higher
58. (a) If one starts clockwise from the husband B, then there is no
bachelor between A and her husband B.
59. (b) According to the given question, the following diagram will be
I P luminosity.
68. (a) Profit made by the company A in the year 2009 = ` 5
lakh
drawn as
60 m
NW
N

A R
NE
Total profit made by all the three companies in the
year 2009 = ` 16 lakh
∴Required per cent =
5
× 100 = 31.25%
20 m
10 m W

R K E
16
69. (d) Profit of company A in the year 2012 = ` 600000
∴ Required expenditure = Income − Profit
10 m

20 m

S A
Therefore, I am facing North-East direction.
SW
S
SE

60. (d) It is given that A, B, C, D, E, F and G are sitting in a row in that


70.
= ` 754300
(c) Required average = ` 

= 1354300 − 600000

3 + 5 + 4 + 5 + 6 + 6
6
 lakh

= ` 4.83 × 100000
order from left to right A B C D E F G.
= ` 483000
Given the person who wears red coloured shirt sits to the
right of D but is not F and G wears pink coloured shirt. Solutions (Q. Nos. 71-72)
Hence, E wears red coloured shirt. Given C wears blue From the given information, family tree can be made as
coloured shirt and the person who wears green coloured shirt shown below
sits to the left of C. Hence, either A or B wears green coloured (–)
D
(+) Female
shirt. If A wears black, then B wears green coloured shirt. (+) Male
Father
∴ The distribution is as shown below (–) (–) (+)
C Sister
A Couple
E
A B C D E F G Mother
Son
(+)
Black Green Blue Yellow Red Violet Pink B

Hence, option (d) is correct. 71. (c) Only Statements 1 and 4 are required to establish that
61. (d) The Bhartiya Mahila Bank has been set-up to improve the E is a male.
financial condition of the women; be it of a poor state or of a 72. (d) From the family tree, it is clear that D is the father of E’s
rich one. Hence, the last statement weakens the argument. wife. Hence, D is the father-in-law of E.

PDF DOWNLOAD FROM : WWW.SARKARIPOST.IN


Page 28

30 Practice Set 1

Solutions (Q. Nos. 73-76) 77. (c) Pension in first year = ` 1000 and thereafter 95% of the amount
Captain → C, D and E in the preceding year.
A ≠ B, P ≠ E So, the pension received by him forms an infinite GP with
R = A or D or (A + D), R → Saviours team  a 
a = 1000 and r = 0.95 QSum of infinite GP, S ∞ =
73. (c) As D, E are captains, so they can’t go in the same  1 − r 
team. So, option (b) can be eliminated. Option (d) can
So, the old man will get = [1000/(1 – 0.95)]
be eliminated on the ground that having no captain.
= ` 20000
The statement 4 states that R must be in a team with
if he remains alive forever.
either A or D or both. So, option (a) can be eliminated.
Hence, (c) is the right option as it does not violate any 78. (a) Let his age be x years. Then,
of the conditions. According to Sud: 25 < x < 34
74. (b) A team consisting of R, C and D in the Saviours team According to his brother: 26 < x < 32
would violate the condition that C and D be the According to his mother: x < 29
captains of their respective teams. a
Q Sum of infinite GP, S ∞ =
1− r
75. (d) Combination (a) is not possible as E and P cannot be
in the same team. Combination (b), E, R and S is not
possible as R must be in the team with either A or D
or both. Combination (c), E, R and Q is not possible,
I N
So, if all the above estimates are true, then his probable ages
could be 27 or 28 yr and hence, average of all probable ages
is 27.5 yr.
as R must be in a team with either A or D or both,
hence option (d) is the right choice as E, S and Q do
not violate any rule.
.
79. (d) Clearly, the facts given in 1 and 2 contain two conditions to be

T
fulfilled to get retirement and also indicate that Rahul fulfils only
one condition (first) out of them. So, both statements are
76. (d) In each option, the Saviours team and the Kings team
have exactly one captain. In no option, A and B are
together either in the Saviours or the Kings team. In
S
needed to answer the question.

O
80. (d) From both Statements 1 and 2, we find that the number of
cards sold this year = (2935 × 1.2 ) = 3522. So, both statements
each option, R and D together in the Saviours team.
Option (d) is not possible as P cannot be assigned to
the Warriors team because E is assigned to that team.
I P are needed to answer the question.

A R
R K
S A

PDF DOWNLOAD FROM : WWW.SARKARIPOST.IN


Page 29

CSAT Paper 2

Practice Set 2
I N
T .
1. There are 80 questions in this paper.
O S
There is only one answer to be selected by you.
Penalty for wrong answer
I P
2. The answer of any question you are thinking that more than two answers are true, then you must choose the nearest one.

deducted in your total marks.

A R
3. There are four alternative answers in every question. When you select a wrong answer, then 1/3rd mark of that question is

4. If any candidate gives more than one answer and one of them is true but it is treated as a wrong answer and the candidate

R K
is penalised for that and1/3rd marks will be deducted.

Directions (Q. Nos. 1-8) Read the following six passages and answer the questions that follow.

S A
Your answers to these questions should be based on the passage only.
KNOW THE TREND
B Passage 1
Illiteracy can bring down even the most powerful nations, so if we are to become a developed nation,
the government should first remove the problem of illiteracy by introducing effective programmes with
proper implementation and budget.
It is ironical that even today; our leaders and people’s representatives give literacy a very low priority.
They fail to perceive literacy as part of the development process, as an endeavour to improve the
quality of life, as the process of building awareness among the weaker sections, as part of the
democratisation of political power, as the arrangement to give their due, to bridge the gap between the
rich and the poor. Neglecting the issue of illiteracy can hurt the development of India very badly.
Stage 1

PDF DOWNLOAD FROM : WWW.SARKARIPOST.IN


Page 30

32 Practice Set 2

1. Which of the following statements is not true in the context of illiteracy of the country as per
the passage?
(a) There is a shortage of a suitable plan and the budget that can accommodate the implementation
of an illiteracy programme.
(b) People’s representatives are working on a suitable measure to curb the menace of illiteracy.
(c) People, especially in the rural India are subjected to severe illiteracy due to lack of proper
infrastructure there.
(d) Removal of illiteracy is required to bridge the gap between the rich and the poor in the country.

B Passage 2
Ninety-seven per cent of the water on the Earth is salt water. Only three per cent is fresh water; slightly

N
over two thirds of this is frozen in glaciers and polar ice. The remaining unfrozen fresh water is found
mainly as groundwater, with only a small fraction present above ground or in the air. Fresh water is a

I
renewable resource, yet the world’s supply of clean, fresh water is steadily decreasing. Water demand

.
already exceeds supply in many parts of the world and as the world population continues to rise, so too
does the water demand. Awareness of the global importance of preserving water for ecosystem service

T
has only recently emerged as, during the 20th century, more than half the world’s wetland have been

O S
lost along with their valuable environmental services for Water Education.

2. Which among the following is the major contributing factor for the scarcity of water resources
on the surface of the earth as per the passage?

I P
(a) 97% of the water is salt water and only 3% pure water make the equitable distribution of water
difficult over the surface of the earth.
(b) Most part of water is trapped in the form of glaciers and polar ice which makes the fresh water

A R
least available to the fauna over the earth.
(c) Increasing world population is a threat to the proper distribution of water over the different
parts of the world.

K
(d) The loss of the wetland has caused the acute shortage of fresh water resources across the world.

R
S A B Passage 3
The main reason for the high level of unemployment is technological progress. Don’t get me wrong;
progress is good and it makes life easier. But if every year we produce the same amount of goods with
fewer people — in a few years far less working hours are needed to produce all the goods that are
required. The historical trend has been to increase the efficiency of work force. If we do not continue this
trend, the supply of working hours is greater than the demand.
We also know that what is abundant has low value, so an oversupply of working hours means they are
worth less, wages and salaries get reduced. Also many persons are out of work; their working hours are
no longer needed. Those that are out of work have no income and therefore the demand for goods goes
down. With fewer sales, less gets produced, more persons are laid off.
This is a vicious circle that accelerates unemployment and produces crime; because some will turn to
crime to obtain income. By distributing the available work between all persons intend to work the
vicious cycle can be broken.

PDF DOWNLOAD FROM : WWW.SARKARIPOST.IN


Page 31

Stage 1 Know the Trend 33

3. Which of the given inference is most critical in connection to unemployment in India?


(a) The development of technology and the dependence over it has grown manifold causing a lot of
people workless.
(b) The production of same kinds of goods have made the consumption go down and with this job
opportunities have dipped.
(c) Adopting the criminal ways of living have been so lucrative that people don’t bother about
being jobless.
(d) The management has not been able to divide the work available with the firm equally among
the workers and hence causing a mess resulting into unemployment.

B Passage 4
The right to privacy refers to the concept that one’s personal information is protected from public

I N
scrutiny. The right to privacy most often is protected by statutory law. It enforces the right to privacy in
various privacy policies and privacy statements. The right to privacy often must be balanced against the
state’s compelling interests, including the promotion of public safety and improving the quality of life.

T .
Seat-belt laws and motorcycle helmet requirements are examples of such laws. And while many citizens
are quite aware that the government collects personal information, most say that government
surveillance is acceptable. It would doubtless be desirable that the privacy of the individual should

O S
receive the added protection of the criminal law, but for this, legislation would be required. Perhaps it
would be deemed proper to bring the criminal liability within narrower limits; but that the community
has an interest in preventing such invasions of privacy.

of privacy?
I P
4. As per the reading of the passage what rational inference can be drawn in the context of right

R
(a) The right to privacy is a Fundamental Right of people and it must be protected by the statutory
laws on a compulsory basis despite the situation people are in.

A
(b) The right to privacy is a Fundamental Right of people and it must be protected by the statutory

K
laws except some cases when social or national interest is there.
(c) The right to privacy is to be protected from the criminal laws so that the basic human right is

R
saved from the ill effects of it.

the right.

S A
(d) For the criminals there must not be the right to privacy as they are damaging the core idea of

B Passage 5
Attaining long-term food security requires the raising of incomes and making food affordable. To
ensure food security for the vulnerable section of the society a multiple pronged strategy is to be
evolved. To begin with all the existing social safety net programmes need amalgamation and should
focus on vulnerable and underprivileged regions and groups. The existing anti-poverty programmes may
be made more transparent with better government that minimises leakages and benefits from such
programmes. Simultaneously, agricultural needs to be reformed by improving incentives, increasing
investment etc so that production of traditional and high-value commodities can be increased.

PDF DOWNLOAD FROM : WWW.SARKARIPOST.IN


Page 32

34 Practice Set 2

5. Which one of the following statements is the most suitable corollary with respect to the
achievement of food security for the citizens?
(a) Attaining food security is for long-term needs a threshold income and producing enough food.
(b) A strategy is required for the vulnerable sections of the society for food security but rest of the
sections may be covered with the existing one.
(c) There is a need of a complete overhaul in the government programmes to make the food
available and affordable to ensure food security.
(d) An agricultural reform is needed in order to maintain the food production which will ultimately
pay back in the form of food security.

B Passage 6
History proved beyond doubt that every Empire that evolved and flourished across centuries created its

I N
own grave-diggers. On the same lines, historians have debated the causes of the decline of Mughal
Empire. The causes of the decline of the Mughal Empire were deterioration of land relations, emergence
of regional powers as successor states, selfish struggle of nobles at the court, lack of initiative in modern

T .
weapons, lack of control over the bankers of the state and above all, Aurangzeb's Deccan campaign.
Unlike Emperor Akbar who paid his officials’ salaries directly from the state treasury, Shahjahan and

S
Aurangzeb opted for Jagirs. There was a constant clash of interest between the nobles at the Emperor’s
court and zamindars. Consequently the main danger to law and order came from zamindars as they
refused to pay the revenue and had to be cowed down.
O
P
The politics that emerged upon the collapse of the Mughal Empire was of two kinds. In one class the
‘succession states’ like Hyderabad, Bengal and Awadh, which were really fragments of the Empire. In the

I
second category were the Maratha Confederacy, the Jats, the Sikhs and the Afghans. Their origins as

R
polities were independent of Mughal Empire. Mysore under Hyder Ali and Tipu Sultan stood outside
these two categories. At the same time, it was the first state in India to make a beginning towards

A
modernisation. In Mughal Empire, the nobles found that their careers were not linked to talent and that

R K
loyal and useful service was ‘no security against capricious dismissal and degradation’. Their selfish
struggle led to factionalism. In order to sustain their power in court, these nobles had surreptitious
relations with regional governors, zamindars and other chieftains.

S A
There was an increasing involvement of banking firms in revenue collections at regional and local levels
that brought bankers into positions of political power all over India. The Deccan Campaign became
Aurangzeb’s Waterloo as the Mughals failed to defeat the Marathas. Unlike Emperor Akbar, who
assimilated Rajputs within his kingdom, Aurangzeb was unable to effectively assimilate the regional
warrior chiefs. Failure to sustain imperial officers in the province resulted in intensified disorder and
defiance of imperial authority.

6. Identify the cause that was not responsible for fall of Mughal Empire.
1. Lack of modern weapons.
2. Infighting and factionalism of nobles.
3. Peasant and tribal revolts.
4. Wars with English East India Company.
5. Jagirs awarded by Shahjahan.
Select the answer using the codes given below
(a) 2 and 3 (b) 1, 3 and 4 (c) 3 and 4 (d) 2, 4 and 5

PDF DOWNLOAD FROM : WWW.SARKARIPOST.IN


Page 33

Stage 1 Know the Trend 35

7. The politics that emerged upon the fall of Mughal Empire consisted of
1. Succession states.
2. The Maratha Confederacy, the Jats, the Sikhs and the Afghans.
3. Hyder Ali and Tipu Sultan.
Select the correct answer using the codes given below
(a) 1 and 3 (b) 2 and 3 (c) Only 3 (d) 1 and 2

8. Consider the following statements


1. Banking firms were increasingly involved in revenue collection during Mughal times.
2. The Deccan campaign proved disastrons for Akbar.
3. The Mysore kingdom was first to modernise in India.
Which of the statements given above is/are correct?
(a) Only 3 (b) 2 and 3 (c) 1 and 3 (d) 1 and 2

I N
9. A man buys a bus for ` 500000. If the annual maintenance cost of the bus is 4% and annual tax
is ` 5000, for how much rent per month (in `), should the man rent the bus, so that he gets an
annual return of 20%?
(a) 7500 (b) 9250 (c) 12500

T . (d) 8750

S
10. While working alone, Sajid and Wajid can paint a wall in 4 days and 8 days, respectively. Sajid
goes and paints the white portion of the wall to black on the first day. On the second day, Wajid

O
paints the black portion of the wall to white. They both continue to work in this manner. What is

(a) 12 days (b) 13 days P


the maximum number of days after which the whole wall will be painted black?

I (c) 15 days (d) 16 days

R
11. The average salary of 100 employees in an office is ` 16000 per month. The management
decided to raise salary of every employee by 10% but stopped the transport allowance of

A
` 1100 per month which was paid earlier to every employee. What will be the new average

(a) ` 162000 K
monthly salary? (Assume transport allowance is not included in the salary)

R(b) ` 16500 (c) ` 17000 (d) No change in the salary

(a) 20
A
12. Six bells commence tolling together and toll at intervals of 5, 10, 15, 20, 25 and 30 s,
respectively. In 60 min, how many times do they toll together?

S (b) 15 (c) 13 (d) 11

13. Urgently, Sudeep has to travel from Delhi to Lucknow. He travels by a car and notices that, if
he increases his speed by 6 km/h, he will reach Lucknow 4 h early. Also, if he decreases his
speed by 6 km/h, he will take 6 h more. The distance between Delhi and Lucknow (in km) is
(a) 600 (b) 810 (c) 640 (d) 720

14. In a class of 80 students, 25 are studying Commerce, 15 Mathematics and 13 Physics.


3 students are studying Commerce and Mathematics, 4 students are studying Mathematics
and Physics and 2 students are studying Commerce and Physics. 1 student is studying all the
three subjects together. How many students are not studying any of the three subjects?
(a) 40 (b) 35 (c) 20 (d) 15

15. A boy walks 2 km to West and then turns North and walks 10 km. Again, he turns to West and
walks 4 km. After this he turns to South and walks 18 km. Now, how far is he from his starting
point?
(a) 10 km (b) 11 km (c) 12 km (d) 13 km

PDF DOWNLOAD FROM : WWW.SARKARIPOST.IN


Page 34

36 Practice Set 2

16. A leak in the bottom of a tank can empty the full tank in 6 h. An inlet pipe fills water at the rate
of 4 L in a minute. When the tank is full, the inlet is opened and due to the leak the tank is
empty in 8 h. Find the capacity of the tank.
(a) 5000 L (b) 5670 L (c) 5700 L (d) 5760 L

17. 720 sweets were distributed equally amongst children in such a way that number of sweets
received by each chld is 20% of the total number of children. How many sweets did each
receive?
(a) 12 (b) 14 (c) 11 (d) 15

18. A student goes to his school from his house walking at 4 km/h and reaches his school 10 min
late. Next day, starting at the same time he walks as 6 km/h and reaches his school 5 min
earlier than the scheduled time. Find the distance between school and home.
(a) 2 km (b) 3 km (c) 4 km (d) 5 km

I N
19. In a bag containing three balls, a white ball was placed and then one ball was taken out at
random. What is the probability that the extracted ball would turn on to be white, if all

equally possible?
(a) 5/8 (b) 3/4 (c) 1/2
T .
possible hypothesis concerning the colour of the balls that were initially in the bag were

(d) 3/8

O S
20. Of the 200 candidates who were interviewed for a position at a manufacturing unit, 100 had a
two-wheeler, 70 had a credit card and 140 had a mobile phone. 40 of them had both a

I P
two-wheeler and a credit card, 30 had both a credit card and a mobile phone and 60 had both a
two wheeler and mobile phone and 10 had all the three. How many candidates had none of the
three?
(a ) 0

A R
(b) 10 (c) 18 (d) 20

Directions (Q. Nos. 21-28) Read the following seven passages and answer the questions that

R K
follow each passage. Your answers to these questions should be based on the passages only.

B Passage 1

S A
Technical progression at global scale has put more pressure on developing countries to improve their
infrastructure and progress in other important areas for prosperity. In various news report and
documents it is demonstrated that the guidelines to recognise a city as the smart city will be prepared by
the department of industrial policy and promotion. The standards for being recognised as a smart city
must have three of the five infrastructure requirements such as energy management, water management,
transport and travel, safety and security and solid waste management.
21. Which of the following statements has triggered the need of developing the smart cities in the
country?
(a) The lack of basic infrastructure in the country compelled the Govt. to revitalise it and it
becomes the smart city project.
(b) Smart cities are the outcome of planned policies in order to boost the existing infrastructure of
the country.
(c) The need of different aspects of management like energy, water, communication and waste etc.
has forced the Govt. to think of smart cities.
(d) The global technical progression has pressurised the countries with fewer infrastructures to
upgrade it and ultimately it has lead to the concept of smart cities.

PDF DOWNLOAD FROM : WWW.SARKARIPOST.IN


Page 35

Stage 1 Know the Trend 37

B Passage 2
Poverty can be defined as a social phenomenon in which a section of the society is unable to fulfil even
its basic necessities of life. The problem of poverty is considered as the biggest challenge to development
planning in India. High poverty levels are synonymous with poor quality of life, deprivation,
malnutrition, illiteracy and low human resource development. There are two basic pre-requisites of a
poverty eradication programmes. First, reorientation of the agricultural relations so that the ownership
of land is shared by a larger section of the society. Second, no programme of removal of poverty can
succeed in an economy plagued by inflation and spiraling rise of price.

22. What critical inference one can draw from the reading of the above passage?
(a) Poverty alone is the biggest challenge in the country and it needs to be addressed soon by the
government.
(b) Poverty is the other name of the poor quality of life and only human resource development can
mitigate it.
N
(c) A land restructuring programme can boost the agricultural prospects and ultimately poverty
can be addressed.
I
B Passage 3 T .
(d) A composite of agricultural reform and controlling inflation will help in the mitigation of poverty.

O S
Article 24 of the Indian Constitution provides for prohibition of child labour. It says ‘No child below the
age of 15 years shall be employed to work in any factory or mine or engaged in any other hazardous

P
employment.” Despite the above mentioned constitutional provision, the child labour scenario in India
presents a disturbing picture. India tops the list of countries having the highest child labour

I
population. The main cause of the child labour in India is considered lack of education. The prevalence

R
of child labour in India is largely due to the failure of the educational system. Due to lack of compulsory
education for which constitutional provisions exist, half of the population of school age children is
either at home or in the labour force.
A
child labour in India?
R K
23. Which of the following statements is the most suitable and rational corollary with respect to

(a) The lack of strength of our constitution is the main reason for the fostering of the child labour

A
in our country.

S
(b) The lack of the manpower in the factories and at the workplace is the major cause of child
labour in India.
(c) The lack of proper education in our country and its poor implementation are the reasons of
surge in the child labour.
(d) Strict policies regarding child labour is not in India and with its less severity it has been a weak
link in the system.

B Passage 4
Panchayati Raj genesis can be seen in ancient period in texts of Panini, Buddhist text and Rig Veda in
respect to administration autonomy. In Manusmiriti, Mahabharat and even in Kautilya’s “Arthshastra”
we get talk about officials which constitute a group of village republics. During Mauryan period, Gupta
period and Sultanate period Panchayats were organised and had performed function of sanitation,
education and judiciary etc. The Britishers had created local self government, as early as 1687, the
municipal corporation of Madras was formed. Then in 1870 a resolution for decentralisation of power
was passed by Bengal Chowkidari Act 1870. It was the first step towards local self government. The
Government of India Act of 1935 also had provision for provincial autonomy and had strengthened
panchayats in the country.

PDF DOWNLOAD FROM : WWW.SARKARIPOST.IN


Page 36

38 Practice Set 2

24. Which of the following options captures the essence of the Panchayati Raj System in the best
way?
(a) The Panchayati Raj System has its roots lying in the very ancient period and therefore it is
necessary to follow.
(b) India is a country of villages and therefore the administration should lie with it to run the
things smoothly.
(c) We have adopted the British laws and therefore it is inevitable to follow the Panchayati Raj
System.
(d) The decentralisation of the administration and to strengthen the villages of the country it is
necessary to adopt the policy of Panchayati Raj System.

B Passage 5

I N
Global temperatures began to rise slowly after 1850, with the beginning of the Modern Warm Period.
There was a vast migration from Europe by land-hungry farmers and others, to which the famine caused
by the Irish potato blight contributed, to North America, Australia, New Zealand, and Southern Africa.

T .
Millions of hectares of forest and woodland fell before the newcomers’ axes between 1850 and 1890, as
intensive European farming methods expanded across the world. The unprecedented land clearance
released vast quantities of carbon dioxide into the atmosphere, triggering for the first time humanly

O S
caused global warming. Temperatures rised more rapidly in the twentieth century as the use of fossil
fuels proliferated and greenhouse gas levels continued to soar. The rise has been even steeper since the
early 1980s. The Little Ice Age has given way to a new climatic regime, marked by prolonged and steady
warming.

I P
25. Which of the following statements can’t be attributed for the rise in Global Warming and that

A R
weakens the argument of the author in the passage?
(a) Excessive usage of fossil fuels has triggered a rise in the greenhouse gases which ultimately
causes a steep rise in the global temperature.

K
(b) Due to steady and continued rise in the temperature, the Little Ice Age has been gone and in its
place a new climate has taken place.

R
(c) The excessive land clearance by the Europeans has lead to the fall in the density of the

A
woodland area and in turn it has affected the global temperature.
(d) The migration, in any form, puts a pressure on the environment and that causes severe stress on

S
the whole ecosystem.

B Passage 6
Malnutrition does not only arise because of insufficient or too much nutrient intake. The problem may
also occur because of the area where there is a population growth which may lead to insufficient food
supply for some part of the population. But these are not the only ways malnutrition can occur in social
conditions; teenage girls often have a problem with their body and go on diets and become either
anorexic or bulimic or even both. Malnutrition is also a cause of a poor economy where people do not
have the money to buy food and because of that end up having an unbalanced diet. The real problem is
money. Food is available but people can’t afford it. We need real development and income-generating
projects. Poor people live on the basic commodities such as bread, tea, sugar and rice. Vegetables and
meat are too expensive for them.

PDF DOWNLOAD FROM : WWW.SARKARIPOST.IN


Page 37

Stage 1 Know the Trend 39

26. Which of the given assumptions can be a handy tool in the process of alleviating the curse of
malnutrition?
(a) The nutritional chart of every individual should be prepared and it should be supervised
properly.
(b) There must be an opulent supply of the basic food items on affordable cost so that people can
afford it.
(c) Government should produce the food items which are basic such as grains and avoid producing
the high cost items in bulk.
(d) Government must put a check on all the social trends and conditions that make the persons opt
for dieting.

B Passage 7

N
The miseries of the world cannot be cured by physical help only. Until man’s nature changes, his
physical needs will always arise, and miseries will always be felt, and no amount of physical help will

I
remove them completely. The only solution of the problem is to make mankind pure. Ignorance is the

.
mother of evil and of all the misery we see. Let men have light, let them be pure and spiritually strong
and educated; then alone will misery cease in the world. We may convert every house in the country

T
into a charitable asylum, we may fill lend with hospitals, but human misery will continue until man’s
character changes.

O S
27. According to the passage, which of the following statements is most likely to be true as the
reason for man’s miseries?
(a )
(b )
(c )
P
The poor economic and social conditions prevailing in society

I
The refusal on the part of man to change his character
The absence of physical and material help from his society
(d )
R
Ever increasing physical needs due to changing social structure

A
28. With reference to the passage, the following assumptions have been made.

misery.

R K
1. The author gives primary importance to physical and material help in eradicating human

2. Charitable homes, hospitals, etc can remove human misery to a great extent.

(a) Only 1

S A
Which of the assumptions is/are valid?
(b) Only 2

29. How many triangles are there in this figure.


(c) Both 1 and 2 (d) Neither 1 nor 2

(a) 35 (b) 40 (c) 37 (d) 32

30. Which number replaces the question mark?


3 12 7
4 15 9
6 11 ?

(a ) 2 (b ) 3 (c ) 4 (d ) 0

PDF DOWNLOAD FROM : WWW.SARKARIPOST.IN


Page 38

40 Practice Set 2

31. A gardener increased the area of his square garden by increasing its side by 25%. The area of
the new garden
(a) has increased by 50% (b) has increased by 56.50%
(c) has increased by 56.25% (d) is exactly the same as the old area

32. In this question, from the given answer figures, select the one in which the question figure is
embedded.
Question figure

Answer figures

(a) (b) (c) (d)


I N
33. In a row of children, Varun is seventh from the left and Neetu is fifth from the right. When
Varun and Neetu exchange their positions, Varun will be sixteenth from the left. Then, how
many students are there in a class? T .
(a) 18 (b) 19

O
Directions (Q. Nos. 34-38) Examine the graph carefully and answer the questions that follow.
S(c) 20 (d) 21

400
I P
Number of employees working in different departments in a company

375
350

A R 350
Number of employees

300 275

R K
250
200
150
225
200
175

S A 100
50
0
HR Advertising IT Finance Production Marketing
Department
Male-Female Ratio
Department Male Female
HR 9 16
Advertising 3 2
IT 9 31
Finance 2 3
Production 11 4
Marketing 4 3

34. What is the total number of male employees working in all the departments together?
(a) 755 (b) 925 (c) 836 (d) 784

PDF DOWNLOAD FROM : WWW.SARKARIPOST.IN


Page 39

Stage 1 Know the Trend 41

35. What is the total number of female employees in HR department?


(a) 158 (b) 128 (c) 140 (d) 144

36. What is the ratio of the total number of employees working in the production department to
that of the marketing department?
(a) 15 : 14 (b ) 8 : 7 (c) 14 : 15 (d ) 7 : 8

37. In which department, is the number of female employees the least?


(a) Advertising (b) Production (c) HR (d) Finance

38. What is the total number of employees in all the departments of the company?
(a) 1500 (b) 1575 (c) 1582 (d) 1600

39. A dice is thrown four times and its four different positions are given below. Find the number
on the face opposite the face showing number 2.

2 1 3 6

I N
(a ) 4
3 4

(b ) 5
2 3 5 4

(c ) 6
2 4

T .
(d ) 3

O S
40. A scooter before overhauling requires 2/3 h service time every 45 days, while after
overhauling, it requires 2/3 h service time every 60 days. What fraction of pre-overhauling
service time is saved in the latter case?
(a) 4/3 (b) 1/3

I P (c) 3/4 (d) 1/4

R
Directions (Q. Nos. 41-47) Read the following six passages and answer the questions that follow
each passage. Your answers to these questions should be based on the passage only.

A
R K B Passage 1

S A
Skilling and entrepreneurship complete each other! Skills are fundamental to, but not sufficient for,
gaining decent jobs. Improved productivity through skill development must be complemented by
economic growth and employment opportunities.
They are, collectively, a prerequisite to the government’s pursuit of holistic development of the nation.
Unleashed entrepreneurship, besides employment generation, will also utilise the youth power for
productivity improvement and wealth creation; thus, stimulating equitable development and a
sustainable path for decent livelihood creation for the country. Technology and resources have to be
judiciously harnessed to achieve the same. Furthermore, endeavours need to be aligned to ensure the
outreach of opportunities and services to the bottom of the economic pyramid, facilitated by
enterprises.

41. The author’s view point can be best summed up in which of the following statements —
(a) Skilling and entrepreneurship are complementary to each other.
(b) More emphasis is required on generating employment opportunities.
(c) Emphasis on entrepreneurship will help in employment generation as well as stimulate equitable
development and a sustainable path for decent livelihood.
(d) There is no solution in sight for the problem of providing opportunities to the very poor people
of the nation.

PDF DOWNLOAD FROM : WWW.SARKARIPOST.IN


Page 40

42 Practice Set 2

B Passage 2
India has welcomed the Paris Agreement on Climate Change. An important pact concluded by 196
countries to put the world on the track to sustainable development.
Will it now leave the ‘‘fake horse race’’, as many see it, in which climate responsibility and development
are treated as separate competing entities, and move to a green growth trajectory away from fossil fuels?
That is the central question for national policy posed by the agreement, one of the most anticipated
international pacts in decades, and one that could bring about a fundamental shift in the development
paradigm. Important answers will naturally depend on how willing the West is to pay for clean
development in India and around the world, since everyone, not just the poor, stands to lose from global
warming.

42. What is the most logical, rational and crucial message that is implied in the following passage?
(a) India, would do well to look at the Paris Agreement on climate change through the prism of
opportunity not adversity.

I N
(b) Central question faced by any nation who were involved in this Agreement is ‘how to move to a
green Growth Trajectory away from fossil fuels.’

T .
(c) India’s agreement on the issue of climate change rests primarily on the principle of equity and
common but differentiated responsibilities of developed and developing nation.
(d) Developed nations are not willing to take the whole responsibility of making the world free from
global warming.

O S
B

I P
Passage 3
BRIC stands for four emerging nations, Brazil, Russia, India and China. It is a coalition of emerging

R
superpowers proposed by Russian President Vladimir Putin.
The coalition encompasses over 40 per cent of the world's population and holds a combined GDP of

A
$12.14 trillion. The idea was proposed because of growing concerns regarding the domination of the US

K
and the European Union over the world economy and political scene. As all the members have
experienced enormous growth in the past few years and are expected to continue this, the coalition will

R
likely grow to be a larger threat to developed countries. Unlike other developed nations, BRIC

S A
economies do not want to dominate the world economy. What they want is an end to their poverty and
an end to the global and unilateral leadership of the United States.
The major driving force behind the soaring economy is the GDP growth. China will continue to witness
a GDP growth of over 7 per cent till 2010 and over 4 per cent till 2035. At the same time, India will grow
at a rate of 5-6 per cent. In the case of Russia and Brazil, the growth rate will vary between 3-4 per cent.
Both rising currencies and faster growth will narrow the gap between BRIC and developed economies.

43. Which among the following is the most rational and critical inference that can be made from
the above passage.
(a) U.S. and other European countries will dominate the world economy and political scene in
coming future also.
(b) The BRIC countries (Brazil, Russia, India and China) are going to be a major thereat to the
domination of existing countries over world economy.
(c) Main reason of the BRIC countries going to complete with U.S and other European. economy is
their rising currencies and subsequent faster GDP growth.
(d) Main purpose of BRIC economies is not to dominate the world economy but to make concerted
efforts to end the poverty of their regions. Unilateral leadership of this U.S. will automatically
dissolve due to this purpose.

PDF DOWNLOAD FROM : WWW.SARKARIPOST.IN


Page 41

Stage 1 Know the Trend 43

B Passage 4
Modern economy can be called as ‘Bank economy’. Banking system transact business by means of ever
negotiable bank accounts and help in attainment of domestic security.
Over several decades the banking has been the target of vicious political attack irrespective of any
ideology. In fascist and National socialist countries of yesterdays also banks functioned in spite of their
repeated denunciation of “Interest Slavery”. In communist countries where bank directors were
compared to leaches, the banking is operated.
Unaccounted of the political complexion, racial composition or social philosophy of the government,
banks are indispensable for the administration of an up-to-date economic system.
Banks are the coordinating centres and observation posts of economic activities, transcend national as
well as ideological frontiers in this modern age. At present stage of development, banks alone are capable
of ushering the surplus production into productive investment and thus promote technical progress and
human welfare.

I N
44. Which one of the following is the most logical assumption that can be made from the above
passage?

T
(a) Bank economy works differently in different political systems. .
(b) In spite of continued criticism banking is operational is all sorts of political scenario -
democracy, communism or federalism.
S
(c) Banks are indispensable for the administration of an update economic system in any part of the
world.
O
post of economic activities.

I P
(d) Any nation can make a headway when banks exist there as coordinating centres and observation

B
A R
Passage 5
Once a conflict gets over, aid agencies sanctioned by the World Bank send study groups instead of

R K
requisite personnel. There is a gap of several years before moving from humanitarian relief to economic
development and hence political stability. By the time such help arrives the war has restarted. It is
possible to restart economic development through targeted ‘quick impact’ initiatives. Most economies

S A
in post-conflict countries are based on agriculture. Providing free packages of seeds, fertilizers and
low-cost agricultural tools will ensure that former soldiers will return to their farms and establish their
livelihood. But the window of opportunity closes quickly and one has to implement these measures
almost immediately.

45. How can economic development be restarted in an impoverished country?


1. Deploying peacekeepers in the country.
2. Restricting number of aid agencies to avoid waste.
3. Long-term studies should be commissioned.
4. Focusing on agricultural initiatives.
Select the correct answer using the codes given below
(a) Only 1 (b) 2 and 4 (c) Only 3 (d) Only 4

46. What is the benefit of ‘Quick Impact’ aid?


1. Providing alternative livelihoods to soldiers before war can restart.
2. Free land is given to soldiers.
3. Soldiers earn income from their farms.
Select the correct answer using the codes given below

PDF DOWNLOAD FROM : WWW.SARKARIPOST.IN


Page 42

44 Practice Set 2

(a) Only 1 (b) 2 and 3 (c) Only 3 (d) 1 and 2

B Passage 6
Water conflicts in India now reach every level; divide every segment of our society-political parties,
states, regions and sub-regions within states, districts, castes and groups and individual farmers. Water
conflicts within and among many developing countries are also taking a serious turn. Fortunately, the
‘water wars’ have not yet materialised. War has taken place, but over oil, not water. Water is radically
altering and affecting political boundaries all over the world, between as well as within countries. In
India, water conflicts are likely to worsen before they begin to be resolved. Till then they pose a
significant threat to economic growth, security and health of the ecosystem and the victims are likely to
be the poorest of the poor as well as the very sources of water rivers, wetlands and aquifers.

1. Trans-border conflicts between developing countries.

I N
47. According to the author, which of the following is/are consequences of water conflicts?

2. Water bodies will remain unused and unaffected till the conflict is resolved.

Select the correct answer using the codes given below


(a) Only 1 (b) Only 2
T .
3. Water conflicts have altered the political boundaries within countries.

(c) Only 3 (d) 1 and 3

O S
48. In the following question, two statements numbered 1 and 2 are given. There may be cause
and effect relationship between the two statements. These two statements may be the effect of

P
the same cause or independent causes. These statements may be independent causes without
having any relationship. Read both the statements and mark your answer.
Statements
I
R
1. The police authority has recently caught a group of house breakers.

A
2. The citizens group in the locality have started night vigil in the area.
(a ) Statement 1 is the cause and Statement 2 is its effect
(b )
(c )
(d )
K
Statement 2 is the cause and

R
Both the Statements 1 and 2
Both the Statements 1 and 2
Statement 1 is its effect
are independent causes
are effects of some common cause

S A
49. In the Lok Sabha elections, there are two candidates – A and B contesting from North Delhi
seat. 3/5th of the people promise to vote for A and the rest promise to vote for B. If 10% of the
people, who promised to vote for A did not vote and 20% of the people, who promised to vote
for B did not vote. Also, if B got 6400 votes, then how much was the margin of victory for A?
(a) 5500 (b) 4000
(c) 4400 (d) 8800

Directions (Q. Nos. 50-53) Examine the information given below and answer the questions that
follow.
A railway station, R, located in India has only two platforms, one for trains going from North to
South and other those going from South to North. Details about the six daily trains—Shatabdi,
Simadri, Gautami, Godavari, Rajdhani and Ratnachal at ‘R’ are known. These are the only trains
that stop at station R, while going from South to North. Shatabdi is either the first or the last train
to arrive at station R. No two trains, having their names starting with the same letter, arrive

PDF DOWNLOAD FROM : WWW.SARKARIPOST.IN


Page 43

Stage 1 Know the Trend 45

immediately one after the other. The name of the train that arrives just before Simadri has the same
starting letter as that of the train that arrives immediately after Simadri. Rajdhani Express comes
immediately after Shatabdi Express.

50. Which is the last train to arrive at station R?


(a) Shatabdi (b) Godavari (c) Rajdhani (d) Ratnachal

51. Which of the following trains comes definitely just before Godavari?
(a) Simadri (b) Gautami (c) Rajdhani (d) Shatabdi

52. Gautami may come before which of the following trains?


(a) Rajdhani or Ratnachal (b) Simadri or Ratnachal
(c) Shatabdi or Rajdhani (d) Rajdhani or Simadri

53. If the train that arrives first has 18 compartments and the trains that arrive second, third,
fourth, fifth and sixth have 15, 12, 9, 6, 3 compartments respectively, then which of the
following will have the least number of compartments?
(a ) Trains whose names start with ‘S’
I N
(b )
(c )
(d )
Trains
Trains
Trains
whose
whose
whose
names
names
names
start
start
start
with
with
with
‘G’ or ‘R’
‘G’ and ‘R’ both
‘S’ and ‘G’ both
T .
54. Identify the missing figure to complete the matrix.
Question figure
O S
I P
?

Answer figures
A R
(a)

R K
(b) (c ) (d)

Statements
A
55. Examine the statements given below.

S
Some carrots are brinjals.
Some brinjals are apples.
All apples are bananas.
Conclusions
1. Some apples are carrots. 2. Some bananas are brinjals.
3. Some bananas are carrots.
Which of the conclusions above is correct?
(a) Only 1 (b) Only 2 (c) Only 3 (d) Either 2 or 3

56. Examine the statement given below.


Statement Local administration made elaborate security arrangements and alerted the local
hospitals to be in readiness during the ensuing festival days.
Assumptions
1. A very large number of devotees may assemble in the city during the festival days.
2. Security personnel may not be able to control the crowd.

PDF DOWNLOAD FROM : WWW.SARKARIPOST.IN


Page 44

46 Practice Set 2

(a ) If only Assumption 1 is implicit


(b ) If only Assumption 2 is implicit
(c ) If either Assumption 1 or 2 is implicit
(d ) If neither Assumption 1 nor 2 is implicit

57. There are 4 villages— A, B, C and D nearby each other. B is to the East of A. Village C is in the
North of B and D is to the right of C. In which direction of D is village A?
(a) South-West (b) North (c) North-East (d) West

Directions (Q. Nos. 58-60) Examine the information carefully and answer the questions that
follow.
A, B, C, D, E, F, G and H are sitting around a circular table facing the centre. D is the neighbour of A
but not of H. B is the neighbour of F and fourth to the left of D. E is the neighbour of H and third to
the right of F. C is neither the neighbour of A nor of G.

58. Which of the following is correct?


(a ) D is to the immediate left of G
I N
(b )
(c )
(d )
A is between C and D
F is fourth to the right of D
None of these
T .
59. Which of the following is wrong?
(a ) B is to the immediate left of H
O S
(b )
(c )
(d )
H is to the immediate left of E
D is fourth to the right of F
C is to the immediate right of E
I P
(a) AFC R
60. Which of the following groups has the second person sitting between the first and the third?

A
(b) GAD (c) BEH (d) HFB

R K
Directions (Q. Nos. 61-67) Read the following two passages and answer the questions that follow.
Your answers to these questions should be based on the passages only.

S A B Passage 1
Recent studies by the Intergovernmental Panel on Climate Change and the International Energy Agency
have suggested that, in order to achieve the international goal of limiting global warming to 2ºC, the
world will need to live within a set carbon budget, and a significant portion of proven global fossil fuel
reserves will need to be left in the ground. The world is currently, however, on a path towards global
warming of 4ºC or more, which the World Bank warned must be avoided in order to prevent
catastrophic climate change impacts.

61. Which among the following is the most logical corollary to the above passage?
(a) The world has to follow certain limitations in order to achieve international goal of limiting
global warming.
(b) The world is currently on a path towards global warming of 4°C or more which must be
avoided.
(c) The restrictions required to contain global warming include within a set carbon budget and
global fossil fuel reserve will need to be left in the ground.
(d) The world has reached on the edge of catastrophic climate change impacts.

PDF DOWNLOAD FROM : WWW.SARKARIPOST.IN


Page 45

Stage 1 Know the Trend 47

B Passage 2
Citizenship today requires individuals be knowledgeable of public problems but, more important, have
the capacity to act together toward their solutions (Morse 1989). Voluntary actions by private citizens
working together to right injustices, change directions and pursue benefits for the common good are
noted throughout.

62. Which one of the following expresses the view implied in the above passage?
(a) Responsible citizens are required to act together to find a solution to a common problem.
(b) There are many examples of good citizens acting voluntarily to fight any injustices and bringing
much required change in the system.
(c) Citizens of today are least bothered about common problems and good of the people.
(d) People all over the world have tried to act together for a common cause and bring justice to the
affected people as they have become more knowledgeable about the existing problems.

B Passage 3 I N
T .
Human rights are the rights relating to life, liberty and equality and dignity of individuals guaranteed by
the Constitution or embodied in international covenants and enforceable by the courts in India.

S
Human Rights have a projection of universality but as a matter of ground reality they are not the same
for all people and societies. For some, especially the well-advanced West, they are predominantly

O
matters of civil and political liberty while for others, the developing and under-developed Asian, Africa

P
and Latin America, predominantly they are also matters of survival. Human Rights are therefore about
being citizen of the earth, being part of an earth family. Human Rights in my view are exercised to their

I
fullness through participating in earth democracy-the democracy of all life. And as earthlings, our

A R
human duties to protect the earth and all her beings are the ground from Human Rights emerge.
‘‘All humanity is one undivided and indivisible family, and each one of us is responsible for the misdeeds
of all others. I cannot detach myself from the wickedest soul.’’ Mahatma Gandhi -

R K
63. With reference to the above passage which of the following is the most valid assumption?
(a) Human Right are common to all irrespective of nationality or region.

S A
(b) In reality Human Rights in developed countries are related to different freedom but in
Underdeveloped and developing countries the Human Rights pertain to basic survival.
(c) Human rights are the prerogative of all individuals and must be protected everywhere in each
country.
(d) If human beings care about the mother earth, their rights will be automatically safe-guarded.

PDF DOWNLOAD FROM : WWW.SARKARIPOST.IN


Page 46

48 Practice Set 2

B Passage 4
Too much heat may lead to high rate of exhaustion that limits the productiveness of human beings. Low
productivity would in turn lead to lower yield hence, slowing down the economic growth. High
temperatures lead, to a lot of heat being generated that may cause Sun burns in most individuals.
Human beings would also face difficulty in the production of food. The ever changing climatic condition
emanating from global warming serves as a setback in the agricultural economy. The weather condition
would be unpredictable and farmers may suffer a big blow due to climatic variation changes. The
elevated temperatures may lead to cracking of runways and sunken foundations in buildings. This may
act as a negative impact towards the economic growth of a given nation.
In conclusion, global warming serves as a major setback in the development of the economy of a given
nation. Industries are eloping up at a high rate and are posing a great challenge in environment
ecosystem. The amount of pollution from industries is increasing which has left government agencies

N
with strategic planning to be conducted. The increases in vehicles have also contributed to global
warming and it has posed a challenge on better mitigation measures. The consequences of global

I
warming may lead to the reduction in progress in the economy of the world, if proper mitigation

.
strategies are not channelled in faster. Although some of the consequences about global warming seem to
be favourable such as, summer hiking and high productivity due to high carbon dioxide present, the

T
demerits seem to be severe. This has called for stake holders from various spheres of world to find ways
of dealing with this crisis of global warming.

64. Consider the following statements


O S
P
1. Excessive heat causes Sun burns in most individuals.
2. Too much heat hampers the prodcutiveness of human beings.

I
3. Much heat enriches the productivity of environment.

(a) 1 and 2
R
Which of the statement(s) given above is/are correct?

A
(b) 2 and 3 (c) Only 3 (d) All of these

K
65. Consider the following statements
1. Global warming serves as a major setback in the development of the economy of a nation.

R
2. The increase in vehicles has also contributed to global warming.

S A
Which of the statement(s) given above is/are correct?
(a) Only 1 (b) Only 2 (c) Both 1 and 2

66. With reference to the passage, consider the following statements


(d) Neither 1 nor 2

1. Global warming may cause a threat to agricultural economy.


2. Climatic variation changes are also a crucial factor in agriculture.
3. Government’s policy is falacious in this regard.
Which of the statement(s) given above is/are correct?
(a) 1 and 2 (b) 2 and 3 (c) 1 and 3 (d) None of these

67. The passage is mainly/thematically centered on


(a ) the rising temperature and its consequences
(b ) the government initiative in disaster mitigation
(c ) people’s approaches and involvement in environmental protection
(d ) None of the above

PDF DOWNLOAD FROM : WWW.SARKARIPOST.IN


Page 47

Stage 1 Know the Trend 49

68. Examine the following statement.


˜ Should import duty on all the electronic goods be dispensed with?
Two arguments numbered 1 and 2 are given. You have to decide which of the arguments is a
‘strong’ argument and which is a ‘weak’ argument.
Arguments
1. No, this will considerably reduce the income of the government and will adversely affect the
developmental activities.
2. No, the local manufacturers will not be able to compete with the foreign manufacturers who
are technologically far superior.
(a) Only 1 is strong (b) Only 2 is strong
(c) Either 1 or 2 is strong (d) Neither 1 nor 2 is strong

69. Based on the following statements, find who is the uncle of P.


1. K is the brother of J.
3. P is the brother of N.
(a ) K (b ) J
2. M is the sister of K.
4. N is the daughter of J.
(c ) N
I N
(d ) M

T .
70. Ajay is lighter than Vipul and Priyanka is heavier than Ajay, while Pramod is lighter than
Priyanka and heavier than Vipul. Then who among them is in the second place from the top
when they are arranged in descending order of heavier weights?
(a) Vipul (b) Priyanka

O
(c) Pramod S (d) Ajay

6
9
1
Women

5
I P
71. In the following figure, which number represents the women doctors who are not employed?

R
Employed
3 2 4

(a ) 7
K
(b ) 3
A
7
8
Doctors

(c ) 1 (d ) 8

A R
72. A question followed by information given in three Statements 1, 2 and 3. You have to study
the question along with the information in three statements and decide that the information in

S
which of the statement(s) is/are necessary to answer the question?
What is the area of the rectangular garden?
1. Perimeter of the garden is 220 m.
2. Length and the breadth of the garden are in the respective ratio of 7 : 4.
3. Length is less than twice the breadth by 10 m.
(a) 1 and 2 (b) 1 and 3
(c) 1 and either 2 or 3 (d) 2 and 3

73. A, B and C started a business with their investments in the ratio of 1 : 2 : 4. After 6 months,
A invested the half amount more as before and B invested the twice the amount as before while
1
C withdrew of their investments. Find the ratio of profits at the end of the year.
4
(a) 5 : 12 : 13 (b) 12 : 5 : 14
(c) 5 : 12 : 14 (d) 5 : 12 : 17

PDF DOWNLOAD FROM : WWW.SARKARIPOST.IN


Page 48

50 Practice Set 2

74. A die is rolled and a coin is tossed. Find the probability that the die shows an even number and
the coin shows a tail.
(a) 3/4 (b) 1/2 (c) 1/4 (d) 1/5

75. A sum is invested as SI at certain interest rate for 2 yr. If the rate of interest had been invested
at 3% higher rate, it would have fetched ` 90 more. The sum (in `) was
(a) 1500 (b) 1200 (c) 2000 (d) 1000

76. Examine the statement given below.


˜ The government has decided not to provide financial support to voluntary organisations
from the next Five Year Plan and has communicated that all such organisations should
raise funds to meet their financial needs.
The statement given above is followed by two courses of action numbered 1 and 2. You have to

N
assume everything in the statement to be true and on the basis of the information given in the
statement, decide which of the suggested courses of action logically follow(s).
Courses of Action
I
2. They should explore other sources of financial support.
(a) Only 1 follows
.
1. Voluntary organisations should collaborate with foreign agencies.

T
(b) Only 2 follows
(c) Either 1 or 2 follows

O S(d) Neither 1 nor 2 follows

77. A tells B “if you give me ` 400, then I shall have 25% more than what you have”. B tells A, “if

does B have?
(a) ` 1200 (b) ` 900
I P
you give me ` 200, then I shall have three and a half times as much as you have”. How much

(c) ` 800 (d) ` 1600

A R
78. A family made a down payment of `750 and borrowed a television set that cost `4000. The
balance with interest was paid in 23 monthly instalments of `160 each and a final payment of

(a) 12%

R K
`90. The amount of interest paid was what per cent of the amount borrowed?
(b) 15% (c) 15.5% (d) 16%

79. A producer of tea blends two varieties costing `18 per kg and another `20 per kg in the ratio of

S A
5:3. If he sells the blended variety at `21 per kg, then what is his gain percentage?
(a) 12% (b) 10% (c) 14% (d) 15%

80. A shopkeeper had to arrange a certain number of toys in rows. When he put 5 toys in each row,
he was left with 1 toy. Then, he tried 6 in a row, then 8, then 9, then 12 but always he was left
with 1 toy. Next, he put 13 toys in a row and then he was left with none. What is the smallest
number of toys?
(a) 3491 (b) 3600 (c) 3601 (d) 4601

PDF DOWNLOAD FROM : WWW.SARKARIPOST.IN


Page 49

Stage 1 Know the Trend

Answers with Explanations


1. (b) On reading of the passage we can conclude that the 11. (b) Total salary of 100 employees = 100 × 16000
representatives of the public are not very much concerned = ` 1600000
about alleviating illiteracy. Hence, it is not true. Increase in salary = 10% of 16000 = ` 1600
2. (c) The major contributing factor for the scarcity of the water Decrease in salary due to withdrawal of transport
resources is the increasing population of the world. With the allowance = ` 1100
increase in the population the demand for water grows as Net effect = 1600 − 1100 = ` 500
more water is required for agriculture, industry and for So, new salary = ` (16000 + 500) = ` 16500
domestic uses. With the limited resources of water,
population becomes really a contributing factor. 12. (c) LCM of 5, 10, 15, 20, 25 and 30 is 300.
So, the bell will toll together after every 300 s (5 min).
3. (a) The most critical inference in connection to unemployment in 60
India is the growth and development of technology and the Hence, number of times they toll together = + 1 = 13
5
dependence over it which has lead to such a situation where
a firm requires less working hours of the human population.
4. (b) The passage clearly states that the right to privacy of every
1
I N
13. (d) Let the distance between Delhi and Lucknow be d km
and the speed of Sudeep be s km/h, then
1 
citizen must be protected by the common law but if there is
any social or national interest only then the right may be
considered to be cut short.
and
T .d −

 1
d
 =4
 s (s + 6)
1
−  =6
 (s − 6) s 
…(i)

…(ii)
5. (c) The passage suggests that there is a need of transformation
in the government programmes in order to achieve food
security by making the food affordable and available to the
various sections of the society especially the vulnerable ones.
O S On dividing Eq. (i) by Eq. (ii), we get
d (6)
s (s + 6) 4
Hence, option (c) gives the best corollary in this context.
6. (c) Though English East India Company was responsible for
downfall of Mughal Empire, it has not been mentioned in the
I P ⇒
d (6)
(s − 6) s
=

(s − 6) 4
6

the passage.

A R
passage. Peasant and tribal revolts are also not mentioned in

7. (d) The politics that emerged upon the collapse of the Mughal


=
(s + 6) 6
6 s − 36 = 4 s + 24
2 s = 60

K
Empire was of two kinds. In one class the succession states
like Hyderabad, Bengal and Awadh. In the second category

R
were the Maratha Confederacy, the Jats, the Sikhs and the
⇒ s = 30 km
On putting the value of s in Eq. (i), we get
d 
1
−  = 4
1

A
Afghans.
 30 36 
8. (c) The Deccan campaign was conducted by the Aurangzeb and
not Akbar.

S
9. (d) Here, total cost of bus per year
= (` 500000 + 4% of ` 500000 + ` 5000) = ` 525000
For getting a return of 20%, he has to get 20% of ` 525000 14. (b)


C
d 

M
6 
 =4
 1080 
d = 720 km

= ` 105000
2 9 C : Commerce
Hence, rent per month = ` (105000/12) = ` 8750 21
M : Mathematics
10. (b) Wajid paints the black portion painted by Sajid to white on 1 1 3
each day. P : Physics

So, from first day onwards in every 2 days 8


1 1 1
= − = th of the wall gets painted black. P
4 8 8
1 3 Let the sets C, M and P represent the students who are
In 12 days = 6 × = th of the wall gets painted black. studying Commerce, Mathematics and Physics,
8 4
1 respectively.
The next day Sajid will paint the rest th part to black.
4 Given, n(C ∩ M ∩ P) = 1,
Hence, the wall will get painted black in (12 + 1) days n(C ) = 25, n(M ) = 15, n(P) = 13
i.e. 13 days. n(C ∩ M ) = 3, n(C ∩ P) = 2, n(M ∩ P) = 4

PDF DOWNLOAD FROM : WWW.SARKARIPOST.IN


Page 50

54 Practice Set 2

1
Since, n(C ∪ M ∪ P) = n(C ) + n(M ) + n(P) − n(C ∩ M ) 1+ + + 
3 1 1
∴ Required probability =
− n(C ∩ P) − n(M ∩ P) + n(C ∩ M ∩ P) 4  4 2 4 
1 4 + 3 + 2 + 1 5
= 25 + 15 + 13 − 3 − 2 − 4 + 1= 54 − 9 = 45 =  =
∴Number of students who are not studying any of the 4  4  8
three subjects = 80 − 45 = 35 20. (b) Let A, B and C denote the set of the two wheeler, credit
15. (a) The movement of boy is as follows: card and mobile phone.
P 4 km N ∴ n( A) = 100, n(B) = 70n(C ) = 140
n( A ∩ B) = 40, n(B ∩ C ) = 30
10 km

N and n( A ∩ C ) = 60
∴ n( A ∪ B ∪ C ) = n( A) + n(B) + n(C )
18 km

4 km 2 km
A − n( A ∩ B) − n(B ∩ C ) − n( A ∩ C ) + n( A ∩ B ∩ C ))
B M (Starting W E
= 100 + 70 + 140 − 40 − 30 − 60 + 10 = 190
8 km

point)
∴ n( A ∩ B ∩ C ) = 200 − 190 = 10

N
S 21. (d) According to the passage, the need of smart cities has
C
been triggered by the global technical progression which is
Starting from the point A, passing through the points M, N
and P, he reaches C.
In right angled ∆ ABC, by Pythagoras theorem,
country.

. I
the root cause of the development of the smart cities in the

22. (d) It can be inferred from the passage that poverty alleviation

16.
AC = (BC )2 + ( AB)2 = (8)2 + (6)2 = 100 = 10 km

(d) Efficiency of leak =


1
6
S T
programme needs a composite effort of agricultural reform
as well as a check on the soaring prices.
23. (c) The flourishing of child labour in India is due to the lack of

1 1 4− 3
So, the efficiency of inlet pipe = − =
6 8 24
1
Combined efficiency of leak and inlet pipe =
8
=
1
24
P O proper education and its failure in the process of
implementation. This is the main reason of an increase n
the child labour.

Inlet pipe can fill the tank in 24 h, at the rate of 4 L /min.


∴Capacity = 24 × 60 × 4 = 5760 L

R I24. (d) The essence of the Panchayati Raj System lies with the
concept of the decentralisation of the administration that
will strengthen the villages. Hence, it is suitable in the
context.
17. (a) Let total number of the children be x.
Then, x ×
20
100
x = 720 ⇒
x2
5
= 720

K A 25. (d) All the form of migration is not severe to the environment
but those migrations may be severe that aims at the
downfall of the flora. Loss of flora results into excess


⇒ x 2 = 5 × 720 = 5 × 5 × 144
x= 5 × 5 × 144 = 60

A
Number of sweets received by each child =
R 20
× 60 = 12
carbon emission which ultimately harms the ecosystem.
Hence, (d) is the suitable option.
26. (b) Most of the people are unable to afford even basic food

18. (b) Let the distance be x km.


Then, Time =
Distance
Speed
S x
100

, 1st day time taken = h


4
items because they are so costly. Also, there needs to be
an enough supply of it to procure food items to the people.
So, food items should be affordable and available both to
alleviate malnutrition.

x x − x h 27. (b) That option (b) is the right answer. It is clear from the
2nd day time taken = h, Difference in time =  
6  4 6 second sentence of the passage which mentions that
miseries cannot be ended unless man changes his
1
Actual difference between these two times = 15 min = h character. On the other hand the passage makes the point
4
that even if the problems mentioned in the other options
x x 1
According to the question, − = are solved the miseries will still remain.
4 6 4
3x − 2 x 1 28. (d) Option 1 is incorrect. The passage infact mentions that
⇒ = ⇒x = 3 providing physical and material help alone will not help in
12 4
eradicating human misery until man changes his
Hence, the required distance is 3 km.
character. Option 2 is also incorrect. The last sentence of
19. (a) Since, all possible hypothesis regarding the colour of the the passage clearly mentions that opening up of hospitals
balls are equally alike, therefore there could be 3 white and charitable homes will not lead to removal of human
balls, initially in the bag. misery until the change of man’s character.

PDF DOWNLOAD FROM : WWW.SARKARIPOST.IN


Page 51

Stage 1 Know the Trend 55

29. (d) Number of triangles 34. (c) Total number of male employees working in all the
A G B departments together
9 3 9 2
= × 225 + × 275 + × 200 + × 175
J 25 5 40 5
F H
11 4
+ × 375 + × 350
K I L 15 7
E C
= 81 + 165 + 45 + 70 + 275 + 200 = 836
D
35. (d) Number of female employees in HR department
∆ABD ∆FHD ∆FGH ∆FGD 16
= × 225 = 144
∆GHD ∆GJH ∆HJD ∆JDF 25
∆GJF ∆KID ∆LID ∆EID 36. (a) Required ratio
∆CID ∆CLD ∆EKD ∆AFE =
Total number of employees in production department
∆AFG ∆EFK ∆AGE ∆EAK Total number of employees in marketing department
∆EAD ∆EFD ∆EGC ∆GHB =
375
= 15 : 14
∆BHC
∆BDC
∆GIC
∆CHL
∆KLD
∆AGD
∆GCB
∆HCD
∆BGD
∆BLC
∆EGI
350
37. (b) Number of female employees in
16
HR =
I
× 225 = 144
N
So, there are 35 triangles in the given figure.
30. (b) In each row of the diagram, add the left and right hand
25
2

T .
Advertising = × 275 = 110
5
digits together and add 2 to give the result in centre i.e.
In row 1, 3+7+2=12
In row 2, 4+9+2=15
O S IT =
31
40
3
× 200 = 155

Finance = × 175 = 105


In row 3, 6+?+2=11
∴ ?=3
31. (c) Let the side of the square be denoted by s.
I P Production =
5
4
15
3
× 375 = 100

Marketing = × 350 = 150


New side after 25% increase = 125% of s = 1.25 s
Area of square = s 2
∴New area = (1.25 s )2 = 15625
. s2
A R 7
So, the number of female employees is least in production
department.

Now, required per cent increase in area

=
.
15625 s2 − s2
2
× 100 =
R K
0.5625 s 2
2
× 100
38. (d) Total number of employees
= ( 225 + 275 + 200 + 175 + 375 + 350) = 1600
s
= 56.25%

S A s

32. (a) It is clearly seen in the option (a), that question figure is
embedded in it.
39. (b) The numbers 1, 3, 4 and 6 lie on the faces adjacent to the
number 2. Therefore, the number 5 lies on the face opposite
to the number 2.
40. (d) Time required for service befoer overhauling
2
= h = 40 min in 45 days
3
and time required for service after overhauling
2
33. (c) According to the question, = h = 40 min in 60 days
3
Varun Neetu Now, LCM of 45 and 60 = 180 days
7th 5th ∴Time required for service before overhauling in 180 days
L R = 40 × 4 = 160 min
On exchanging their positions, and time required for service after overhauling in 180 days
= 40 × 4 = 120 min
Neetu Varun 160 − 120 40 1
16th ∴Fraction of time saved = = =
160 160 4
L R 41. (c) Is the appropriate answer. The government wants to focus
Now, total number of students = L + R − 1 more on entrepreneurship as it will expedite employment
= 16 + 5 − 1= 21 − 1= 20 generation. Besides more wealth creation and improvement
in productivity can be achieved by motivating people to be

PDF DOWNLOAD FROM : WWW.SARKARIPOST.IN


Page 52

56 Practice Set 2

entrepreneurs. Skill development is the basic needed to But Godavari and Gautami cannot come immediately after one
get a decent job but unless job opportunities are there. another as both their names start with the same alphabet ‘G’.
Skill development alone will not bring any change in the So, Simadri cannot be the third train.
economic development of the country. Also, Simadri cannot be the fifth train to arrive as then Ratnachal
would be the third train to arrive as fourth and sixth will be either
42. (b) It has been agreed by almost every country to adopt to Godavari or Gautami and both Rajdhani and Ratnachal cannot
clean energies for future development. Use of excessive arrive immediately one after the other.
fossil fuel has been one of the major reasons of global
warming which was adopted by developed countries at So, Simadri is the fourth train to arrive and third and fifth are
either Godavari or Gautami. Ratnachal is the last train to arrive
the time of ‘Industrial Revolution.’ Poor or developing
at station R. So, the order is
countries should be helped financially to adopt to safer
clean energies for the purpose of development. 1. Shatabdi 2. Rajdhani
3. Godavari / Gautami 4. Simadri
43. (d) BRIC countries are experiencing rising currencies and
impressive GDP growth. Their enormous growth will 5. Gautami / Godavari 6. Ratnachal
definitely theatern the domination of existing world 50. (d) Ratnachal is the last train to arrive at the station R.
economies and also end the poverty of their regions.
51. (c) Rajdhani’s arrival is definitely just before Godavari.
44. (c) Banks have been and are an integral part of any
nation’s economy. They alone can help any country to
promote technical progress and human welfare. Any
I N
52. (b) Gautami may come before either Simadri or Ratnachal.
53. (c) Number of compartments in trains whose names start with
area devoid of banking facilities will suffer and will not
be able to mobilise the untapped financial resources.
45. (d) In the 4, 5 and 6 sentences of the passage clearly give the .
S = Shatabdi + Simadri = 18+9 = 27 compartments
R = Rajdhani + Ratnachal = 15+3 = 18 compartments

T
G = Godavari + Gautami = 6+12 = 18 compartments
reason for focusing on agricultural initiatives, i.e. to
establish the livelihoods of the people. The other
statements are neither mentioned nor implied in the
passage. 54. (a)
S
So, both the trains whose names start with ‘G’ and ‘R’ have
minimum total number of compartments.

O
46. (a) In the sixth sentence of the passage bring this out
clearly as the correct choice. The other statements are
not specifically mentioned in the passage.
I P
47. (c) Water conflicts are radically altering and affecting

A R
political boundaries all over the world, between as well
as within countries.
55. (b)

R K
48. (d) Both the statements are clearly backed by a common
cause, which is clearly an increase in the number of
thefts in the locality.
Carrots
Brinjals Apples

Bananas

49. (c) Let total votes be x

S A
Then, people who voted for A
= 60% of x − 10% of 60% of x = 0.54x
And people who voted for B
Conclusions
1. O 2. P 3. O
56. (a) During the festival days, a large number of devotees may
assemble in the city for which the local administration is
= 40% of x − 20% of 40% of x = 0.32 x, making appropriate security arrangement. So, Assumption 1
Margin of victory for A = 0.54 x − 0.32 x = 0.22 x is implicit. Security personnel are able to control the crowd.
6400
Now, 0.32 x = 6400 ⇒ x = = 20000 So, Assumption 2 is not implicit.
0.32
57. (a) According to the question, following diagram is drawn.
Margin of victory for A = 0.22 × 20000 = 4400 votes N
Solutions (Q. Nos. 50-53) C D
According to the information 1 and 4, we can deduce that W E
Shatabdi is the first train and Rajdhani is the second train A B
to arrive.
SW
Simadri should be the third or fourth or fifth train to arrive. If S
Simadri is the third train, then Ratnachal should be the
fourth. Fifth and sixth would be either Godavari or Gautami. Hence, A is in the South-West direction of D.

PDF DOWNLOAD FROM : WWW.SARKARIPOST.IN


Page 53

Stage 1 Know the Trend 57

Solutions (Q. Nos. 58-60) 60. (b) A is sitting between G and D.


As it is given that B is the neighbour of F and fourth to the 61. (c) Some restrictions are to be followed. Firstly, a limited
left of D. So, the arrangement is carbon budget has to be followed by many countries and
D D reserves of fossil fuels are to be kept underground to avoid
or prevent catastrophic climate change impacts.
62. (d) It is really a pleasant thing to note that knowledgeable
people have joined hands and tried to bring a change in the
OR system. The concerned person has been able to get some
justice at least.

F F 63. (c) Every Individual’s right of equality and dignity must be


protected irrespective of nationality or soil. If we all follow
B B
the basic rule that any one who lives on this earth, has the
Case I Case II right to lead a dignified life, the problem of human rights will
Now, E is third to the right of F. This information rejects the not show its ugly face. Only when rights of people are
violated, problems and issues crop up is any society.
Case I.
Also, it is given that E is the neighbour of H and third to the
right of F, so the following two possibilities are there

I N
64. (a) The passage itself implies the impact of excessive
temperature. Too much heat may lead to sun burns and
inhibition in productiveness in most individuals. Actually
D

H
D

T .
heat is necessary for existence of life on the Earth, but not
in excessive amount, but in turn it causes harm to
individuals.

E Or E

O S
65. (c) The passage clearly signifies the increasing heat and its
impacts on living beings. The excessive heat causes global
warming which in turn affects the economy of a nation.
F
B
Case II (a)
F
B
Case II (b)
H

I P While increasing vehicles have also contributed in the


enhancement of global warming.
66. (a) The given passage explicitly talks about the impacts of heat

Now, from the information that D is the neighbour of A but


not of H. Hence, Case II (a) can be rejected. So, the
following two possibilities are there
A R in excessive amount. But in passage, nowhere is
mentioned about the government policies.
67. (a) The central theme of the passage is to show the causes of

G/C
D
A

R
A K D
C/G
excessive heat and the rising temperature. The uncontrolled
heat radiation causes the inhibition in productivity, sun burn
and lowering down in agricultural output.

C/G

F S
H
A E Or C/G

F H
E
68. (b) Abolishing the import duty on electronic goods shall reduce
the cost of imported goods and adversely affect the sale of
the domestic products, thus giving a setback to the Indian
electronics industry. So, Argument 2 holds strong.
Argument 1 does not provide a convincing reason.
B B
69. (a) According to the question, blood relation diagram is as
Case III (a) Case III (b)
follows
Also, it is given that, C is neither the neighbour of A nor of G, (+) Brother (+)
K J
so Case III (a) can be rejected.
So
Da

Hence, the final seating arrangement will be as follows


Sister

n
ug

D
hte

A C
r

Brother (+)
(–)
M (–)
N P
G E
Here, (+ ) = Male, (− ) = Female
F H
B (—) = linking line
As, P is the son of J.
58. (d) None of the given options is correct.
Given, K is the brother of J.
59. (c) D is fifth to the right of F.
Therefore, K is the uncle of P.

PDF DOWNLOAD FROM : WWW.SARKARIPOST.IN


Page 54

58 Practice Set 2

70. (c) Q Vipul > Ajay 75. (a) Let the sum be ` x and the interest rate be r % per annum.
2 2
Priyanka > Ajay Then, x × (r + 3) × − x × (r ) × = 90
100 100
Priyanka > Pramod
= 2 x(r + 3) − 2 xr = 90 × 100 ⇒ 6x = 9000
Priyanka > Pramod > Vipul > Ajay
∴ x = ` 1500
Hence, Pramod is at second place from top, when arranged
in descending order of heavier weights. 76. (b) The problem arising is shortage of funds. So, alternative
sources of financial support need to be worked out first.
71. (a) ‘7’ represents the women doctors who are not employed. Thus, only course 2 follows.
72. (c) From Statement 1, 77. (a) Let the money with A be a
Perimeter of the garden = 220 m and money with B be b.
2(l + b ) = 220 …(i) According to the question,
l 7 7b
From Statement 2, = ⇒ l= 1.25(b − 400) = a + 400
b 4 4
⇒ 1.25b − 500 = a + 400
On putting the value of l in Eq. (i), we get
⇒ 1.25 b − a = 900 … (i)


2 
7b
 4
+ b  = 220

7 b + 4b 220
= ⇒
11b
= 110

N
Again, 3.5(a − 200) = b + 200

I
3. 5 a − 700 = b + 200
3.5a − b = 900 … (ii)

4
b=

l=
2
110 × 4
11
4
= 40 m

7 b 7 × 40
= = 70 m
and (ii),
we get
T .
Multiplying Eq. (ii) by 1.25 and then adding Eqs. (i)

and
4 4
∴ Area = l × b = 70 × 40 = 2800 sq m
From Statement 3, l + 10 = 2b ⇒ l = 2b − 10
O S ⇒
1.25 b – a =900
– 1.25 b + 4.375 a =1125
0.375 a = 1125 − 900
Again putting the value of l in Eq. (i), we get


2 (2 b − 10 + b ) = 220
3b − 10 =
220
I P ⇒ 0.375 a = 255 ⇒
Putting the value of a in Eq. (i),
we get
a = 600



3b = 110 + 10 =120
b=
120
2

= 40
A R ⇒

. b − 600 = 900 ⇒ 125
125
b = 1200
a = 600 and b = 1200
. b = 1500


3
b = 40 m and l = 70 m

R
Now, area = l × b = 70 × 40 = 2800 sq mK So, B have ` 1200.
78. (d) Down payment made = 750

S A
So, we can easily find the area of the rectangular garden
with the help of the Statements 1 and either 2 or 3.
73. (c) Let us assume their initial investments be x, 2x and 4x,
respectively.
∴Balance = 4000 − 750 = 3250
Total amount paid in 23 months = 23 × 160 = 3680
Total amount paid = 3680 + 90 = 3770
∴Extra amount paid = 3770 − 3250 = 520
Therefore, ratio of their investments during the whole year 520
∴Required percentage = × 100 = 16%
=  x × 6 + x × 6 : (2 x × 6 + 4x × 6) : (4x × 6 + 3x × 6)
3 3250
 2 
79. (a) Let the quantities of varieties be 5 kg and 3 kg.
=15x : 36x : 42 x = 5x :12 x :14x ∴Total cost = 5 × 18 + 3 × 20 = 90 + 60 = 150
∴Ratio of their profits = 5:12 :14 Selling price = 21per kg
74. (c) The sample space S of the experiment described in the ∴Revenue = 21 × 8 = 168
question is as follows 168 − 150 18
∴Proft percentage = × 100 = × 100 = 12%
S ={(1, H), (2, H), (3, H), (4, H), (5,H), (6, H), (1, T), 150 150
(2,T), (3, T), (4, T), (5, T), (6, T)} 80. (c) LCM (5, 6, 8, 9, 12) = 360
So, n(s ) = 12 So, smallest such number which will leave 1 remainder
Let E be the event that ‘the die shows an even number and in each case = 360 + 1 = 361
the coin shows a tail’. and series = 360 N + 1
So, E = {(2, T), (4, T), (6, T)}
⇒ n(E ) = 3 After putting N = 10, we get
3 1
∴ Required probability, P(E ) = = 360 × 10 + 1 = 3601
12 4
which is completely divisible by 13.

PDF DOWNLOAD FROM : WWW.SARKARIPOST.IN


Page 55

CSAT
C i v i l S e r v i c e s A p t i t u d e Te s t
Paper 2

Practice Set 3
I N
Time : 2 hrs
T . MM : 200

Instructions

1. There are 80 questions in this paper.


O S
There is only one answer to be selected by you.
Penalty for wrong answer
I P
2. The answer of any question you are thinking that more than two answers are true, then you must choose the nearest one.

deducted in your total marks.

A R
3. There are four alternative answers in every question. When you select a wrong answer, then 1/3rd mark of that question is

4. If any candidate gives more than one answer and one of them is true but it is treated as a wrong answer and the candidate

K
is penalised for that and1/3rd marks will be deducted.

R
S A
Directions (Q. Nos. 1-13) Read the following five passages and answer the questions that follow
each passage. Your answers to these questions should be based on the passages only. KNOW THE TREND

B Passage 1
‘‘Rural development implies both the economic betterment of people as well as greater social
transformation. In order to provide the rural people with better prospects for economic development,
increased participation of people in the rural development programmes, decentralisation of planning,
better enforcement of land reforms and greater access to credit are envisaged.’’
Initially, main thrust for development was laid on agriculture, industry, communication, education,
health and allied sectors but later on it was realised that accelerated development can be provided only
if governmental efforts are adequately supplemented by direct and indirect involvement of people at
the grass root level. Keeping in view the needs and aspirations of the local people, Panchayati Raj
Stage 1

Institutions have been involved in the programme implementation and these institutions constitute
the core of decentralised development of planning and its implementations.

PDF DOWNLOAD FROM : WWW.SARKARIPOST.IN


Page 56

60 Practice Set 3

1. In the light of the passage above which one of the following statements is the most convincing
explanation?
(a) Rural development means a larger transformation of people which is possible with the help of
Panchayati Raj Institutions.
(b) India’s villages are already developed they do not need any help.
(c) Rural development includes enforcement of land reforms in an effective manner and easy access
to credit, and involvement of rural people at grass root level in all development activities.
(d) Role of Panchayati Raj Institutions in the overall economic development of rural areas is very
crucial and much needed.

B Passage 2
Deontological pacifism decrees that moral agents have an absolute duty to avoid aggression or waging

N
war against others. Duties are moral actions that are demanded in all pertinent circumstances. The
first problem for deontological pacifism is the potential collision of duties. What if force is to be used

I
to halt an aggressor who endangers the pacifist’s life or the life of an innocent? Regarding the pacifist’s

.
own life, it can be argued that he or she possesses no right of self-defence. Among such adherents are
absolute pacifists. Another example: does the duty to respect others outweigh the duty to respect

T
oneself? Those pacifists who admit the right to defend the self against a threat can admit the use of

S
restraining or disabling force and even, if the threat is deadly, the right to kill an assailant.
Deontological pacifists can claim that others’ rights to life are of a higher order duty than the duty to

O
intervene to save oneself. But that hinges upon a moral evaluation of the self compared to others and it

P
is not clear why others should be accorded a higher moral evaluation; for after all, the self is in turn one
amongst many others from a different subject’s point of view. If the pacifist argues that his life is his own

I
to lay down in the face of aggression, the problem intensifies when the life of another is threatened.

A R
The pacifist who claims that he has no duty to intervene in saving others’ affairs treads a precarious
moral path here; the immediate retort is why should the moral life of the pacifist be morally more
important than the life of a threatened innocent? For the sake of his own beliefs, could the pacifist
consistently ignore the violence meted upon others? Yes, from two possible perspectives. The first is that

R K
the ideal of pacifism retains a supremacy over all other ideals. The second is that the life of the pacifist is
morally superior to the life of the threatened innocent. Deontologists argue that certain kinds of moral
actions are good in themselves, hence deontological pacifists claim peace to be a duty to be categorically
upheld.

S A
2. Which of the following, according to the passage, would the deontological pacifist consider
idealistic?
1. Resorting to self defence in the face of mortal danger.
2. Using a tool to save an innocent’s life.
3. Devaluing one’s life in favour of an ethical conduct.
Select the correct answer using the codes given below
(a) 1 and 2 (b) 2 and 3 (c) Only 3 (d) None of these

3. Which of the following is the author unlikely to agree with?


1. It is not incumbent on the pacifist to perform duties in all pertinent circumstances.
2. The notion that there is a potential collision of duties is non-existent.
3. Self also should be given the same moral evaluation as any other.
4. The ideal of pacifism should not gain supremacy over all other ideals.
Select the correct answer using the codes given below
(a) 1 and 2 (b) Only 2 (c) 2 and 4 (d) Only 4

PDF DOWNLOAD FROM : WWW.SARKARIPOST.IN


Page 57

Stage 1 Know The Trend 61

4. What according to the passage is not implied by ‘collision of duties’?


1. Duty to protect others from an assailant or the virtue of pacifism.
2. Duty to forfeit one’s life or the duty to respect another’s life.
3. Duty to accord a higher moral value to the lives of others rather than to oneself.
Select the correct answer using the codes given below
(a) Only 1 (b) Only 2 (c) 1 and 2 (d) Only 3

B Passage 3
Mathematician Edward Lorenz found unexpected behaviour in apparently simple equations
representing atmospheric air flows. Lorenz realised that tiny rounding errors in his analog computer
mushroomed over time, leading to erratic results. His findings marked a seminal moment in the

I N
development of chaos theory, which, despite its name, has little to do with randomness. To understand
how unpredictability can arise from deterministic equations, which do not involve chance outcomes,
consider the non-chaotic system of two poppy seeds, placed in a round bowl. If the bowl is flipped over,

enlarges initial differences in position.

T .
two seeds placed on top will roll away from each other. Such a system, while still not technically chaotic,

Chaotic systems, such as a machine mixing bread dough, are characterised by both attraction and

O S
repulsion. As the dough is stretched, folded and pressed back together, any poppy seeds sprinkled in are
intermixed seemingly at random. In fact, the poppy seeds are captured by ‘strange attractors,’
staggeringly complex pathways, whose tangles appear accidental, but are in fact determined by the
system’s fundamental equations.

I P
During the dough-kneading process, two poppy seeds positioned next to each other eventually go their

R
separate ways. Any early divergence or measurement error is repeatedly amplified by the mixing until
the position of any seed becomes effectively unpredictable. It is this ‘sensitive dependence on initial

A
conditions’ and not true randomness that generates unpredictability in chaotic systems, of which one

K
example may be the Earth’s weather. According to the popular interpretation of the ‘Butterfly Effect,’ a
butterfly flapping its wings causes hurricanes. A better understanding is that the butterfly causes

R
uncertainty about the precise state of the air.

(a )
(b )
A
5. The future development basis of chaos theory was provided by

S
the unpredictable atmospheric flow of air
mathematical equations of atmospheric flows
(c ) random results given by analog computers
(d ) mathematical findings of Edward Lorenz
6. In the example discussed in the passage, what is true about poppy seeds in bread dough, once
the dough has been thoroughly mixed?
1. They have been individually stretched and folded over, like miniature versions of the entire
dough.
2. They are scattered in random clumps throughout the dough.
3. They are accidentally caught in tangled objects called strange attractors.
4. They are in positions dictated by the underlying equations that govern the mixing process.
Select the correct answer using the codes given below
(a) Only 1 (b) Only 2 (c) Only 3 (d) Only 4

PDF DOWNLOAD FROM : WWW.SARKARIPOST.IN


Page 58

62 Practice Set 3

7. The passage mentions each of the following as an example or potential example of a chaotic or
non-chaotic system except
1. a dough-mixing machine. 2. atmospheric weather patterns.
3. fluctuating butterfly flight patterns.
Select the correct answer using the codes given below
(a) Only 1 (b) Only 2 (c) 1 and 2 (d) None of these

8. It can be inferred from the passage that which of the following pairs of items would most likely
follow typical pathways within a chaotic system?
(a ) Two particles ejected in random directions from the same decaying atomic nucleus
(b ) Two stickers affixed to a balloon that expands and contracts over and over again
(c ) Two avalanches sliding down opposite sides of the same mountain
(d ) Two baseballs placed into an active tumble dryer

B Passage 4 I N
T .
There are limits to the population that our environment can sustain in the long-run, particularly in view
of our desire to achieve higher standards of living. There is widespread consensus that population

S
stabilisation entails a holistic, comprehensive approach towards education and health care. We need to
invest in educating the girl child and improving her health and social status. In a country where credible

O
social safety nets are absent, it is children who form the main pillar of support to their parents in their

P
old age. This, coupled with high infant mortality rates, compels families to have more children. There
has to be a concerted campaign to improve the health and educational status of the entire population in
general. Regional imbalances exist.
I
A R
It is necessary to integrate our population policy with our health and education policy as well as a social
empowerment policy. We have to pursue a more balanced policy of regional development so that
adequate employment opportunities are made available in the more populated regions of the country.
The approach of focused attention on women and girls, a general enhancement of educational and

R K
health levels and balanced regional development must be the core of any population stabilisation
strategy. For far too long has the health sector been addressed through a series of vertical, disease based
programmes. The vertical and horizontal fragmentation of health programmes, coupled with a target

A
driven approach to population stabilisation has distanced the entire gamut of health services from the

S
common man. Investing in human capabilities converts human liabilities into human assets. Therefore,
even as our population policy focusses on population stabilisation, it must also focus on altering the skill
profile of our population.

9. Which of the following does the author cite as being responsible for population explosion?
1. Lack of educational and health facilities for the masses especially for women.
2. High infant mortality rate that forces the poor to beget more children out of fear.
3. Regional development policies that failed to achieve their targets.
Select the correct answer using the codes given below
(a) Only 1 (b) Only 2 (c) Only 3 (d) All of these

10. By creating new employment opportunities in less developed regions, our Government aims to
1. bring about all-round progress in areas like health, education, etc.
2. increase the participation of women in work to facilitate a better standard of living.
3. empower the poor to make better informed choices.
Select the correct answer using the codes given below
(a) Only 1 (b) 2 and 3 (c) Only 3 (d) None of these

PDF DOWNLOAD FROM : WWW.SARKARIPOST.IN


Page 59

Stage 1 Know The Trend 63

11. Apart from population stabilisation, the population policy must also focus on
1. empowerment of people so that they become assets and not liabilities.
2. ensuring a decent standard of living to everybody.
3. programmes that help better the rate of achievement of targets.
4. evolving a development strategy that focuses on elimination of poverty.
Select the correct answer using the codes given below
(a) Only 1 (b) 2 and 3 (c) 3 and 4 (d) None of these

B Passage 5
Encompassing an increasing range of economic activities such as retail, travel, tourism, food and
beverages, e-commerce has emerged as India’s new sunrise industry, set to cross in business worth $ 16

I N
billion by the end of 2015. The greater adoption of Internet and smartphones is the biggest driver of
e-commerce in India. Internet penetration is rapidly increasing with around 300 million users in 2014.
The smartphone is steadily growing and consists of 35 per cent of the overall mobile phones market in

e-commerce.

T .
the country and success rate of some of the technologies is directly connected to the success of

The e-commerce companies are concentrating their efforts on increasing the penetration of their mobile
apps for higher growth.

O S
The ‘Future of e-Commerce: Uncovering Innovation’ study reveals that the digital commerce market in

P
India has grown steadily from $ 4.4 billion in 2010 to $ 13.6 billion in 2014 and likely to touch $ 16
billion by the end of 2015 on the back of growing Internet population and increased online shoppers.

I
12. With reference to the above passage, consider the following statements.

billions by the end of 2015.


A R
1. E-commerce has emerged as India’s new sunrise industry, set to cross business worth $ 16

2. The biggest driver of e-commerce is the greater adoption of internet and smartphones.

(a) Only 1
K
Which of the statements given above is/are correct?

R (b) Only 2 (c) Both 1 and 2 (d) Neither 1 nor 2

(a )
(b )
A
13. What is the logical assumption that can be made from the above passage?

S
Government is promoting e-commerce
E-commerce has great scope in India due to internet and smartphones
(c ) E-commerce companies 50 per cent revenue comes from mobile apps
(d ) None of the above statements is a logical assumption that can be made from the passage

14. Examine the information given below.


˜
No student is lazy. All the lazy persons are unsuccessful.
Which of the following is valid conclusion regarding the above argument?
(a ) No student is unsuccessful
(b ) No unsuccessful persons are students
(c ) Some unsuccessful persons are lazy
(d ) All the unsuccessful persons are lazy

15. In the following question, two statements numbered 1 and 2 are given. There may be cause
and effect relationship between the two statements. These two statements may be the effect of
the same cause or independent causes. These statements may be independent causes without
having any relationship. Read both the statements in each question and mark your answer.

PDF DOWNLOAD FROM : WWW.SARKARIPOST.IN


Page 60

64 Practice Set 3

Statements
1. The Reserve Bank of India has recently put restrictions on a few small banks in the country.
2. The small banks in the private and cooperative sectors in India are not in a position to
withstand the competition of the bigger ones in the public sector.
(a ) Statement 1 is the cause and Statement 2 is its effect
(b ) Statement 2 is the cause and Statement 1 is its effect
(c ) Both the Statements 1 and 2 are independent causes
(d ) Both the Statements 1 and 2 are effects of independent causes
16. Which of the following diagrams indicates the best relation among the Factory, Product and
Machinery?

(a) (b) (c) (d)

17. Examine the following arguments.


I N
Some students are engineers. Some engineers are bureaucrats. Some bureaucrats are

.
˜

honest. Some honest persons are loyal.


Which of the following conclusions follow(s) the above arguments?
Conclusions
1. Some loyal persons are bureaucrats.
S T
2. Some bureaucrats are students.
3. Some students are loyal.
(a) None
P
(b) Both 1 and 2 O (c) Both 2 and 3 (d) Both 1 and 3

below. I
18. Six squares are marked A, B, C, D, E and F and are hinged together as shown in the figure given

R A B

K A C D
F
E

(a) B, C, D and ER
If they are folded to form a cube, then what would be faces adjacent to face A?

A (b) B, C, D and F (c) C, D, E and F (d) B, C, E and F

(a ) 4
S
19. A one-rupee coin is placed on a plain paper. How many coins of the same size can be placed
round it, so that each one touches the central and adjacent coins?
(b ) 3 (c ) 7 (d ) 6

20. The charges of a hired car are ` 4 per km for the first 60 km and ` 5 per km for next 60 km and
` 8 for 5 km for further journey. If the balance amount left over with Suman is one-fourth of
what he paid towards the charges of the hired car for travelling 320 km, then how much
money did he have initially with him?
(a) ` 1075 (b) ` 1255 (c) ` 1540 (d) None of these

21. Two chairs and three tables cost ` 1025 and three chairs and two tables cost ` 1100. What is
the difference between the cost of one table and one chair?
(a) ` 75 (b) ` 35 (c) ` 125 (d) None of these

22. Three persons A, B and C are running on a circular track with a speed of 10 m/s, 15 m/s and
20 m/s, respectively. If they start simultaneously from point P in the same direction, after how
much time will they meet for the first time?
(a) 100 s (b) 40 s (c) 66.66 s (d) 200 s

PDF DOWNLOAD FROM : WWW.SARKARIPOST.IN


Page 61

Stage 1 Know The Trend 65

Directions (Q. Nos. 23-24) Examine the following information and answer the questions that
follow.
Five persons Amita, Babu, Chanda, Diwakar and Eshoo are travelling in a train. Amita is the
mother of Chanda who is the wife of Eshoo. Diwakar is the brother of Amita and Babu is the
husband of Amita.

23. How is Babu related to Eshoo?


(a) Father (b) Mother-in-law (c) Brother-in-law (d) Father-in-law

24. How is Amita related to Eshoo?


(a) Sister (b) Mother (c) Mother-in-law (d) Niece

25. A 40 L mixture of milk and water contains milk and water in the ratio of 3 : 2. 10 L of the
mixture is removed and replaced with pure milk and the operation is repeated once more. At

(a) 2.5 : 1 (b ) 3 : 1 (c) 31 : 9


N
the end of the two operations, what is the ratio of milk and water in the resultant mixture?
(d) 3.5 : 1

I
26. A shopkeeper offers his customers 10% discount and still makes a profit of 26%. What is the
actual cost to him of an article marked at ` 280?
(a) ` 200 (b) ` 100 (c) ` 150

T . (d) ` 180

then what was his speed in the final hour?


(a) 44 km/h (b) 45 km/h
O S
27. A car driver completes a 180 km trip in 4 h. If he averages 50 km/h during the first 3 h of trip,

(c) 35 km/h (d) 30 km/h

I P
28. Roshan is taller than Hardik, who is shorter than Susheel. Niza is taller than Harry but shorter
than Hardik. Susheel is shorter than Roshan. Who is the tallest?
(a) Roshan (b) Susheel

A R (c) Hardik (d) Harry

29. In a carromboard game competition, m boys and n girls (m > n > 1) of a school participate. In
which every student has to play exactly one game with every other student. Out of the total games

R K
played, it was found that in 221 games, one player was a boy and the other player was a girl.
Consider the following statements.

A
1. The total number of students that participated in the competition, is 30.
2. The number of games in which both players were girls, is 78.

S
Which of the statements given above is/are correct?
(a) Only 1 (b) Only 2 (c) Both 1 and 2 (d) Neither 1 nor 2

Directions (Q. Nos. 30-38) Read the following three passages and answer the questions that
follow each passage. Your answers to these questions should be based on the passages only.

B Passage 1
If globalisation’s perils tend to be exaggerated in the ways I just discussed, they are also understated by
many who say, ‘Well, we have always had globalisation and it is no big deal.’ True, rapid integration of
the world economy occurred in the late 19th and early 20th centuries. If multinationals bother you,
then just think of the great East India Company, which virtually paved the way for the British conquest
of India and the Dutch East Indies Company. Trade grew rapidly along with European outward
expansion. But all this misses the fact that there are fundamental differences that give globalisation
today a special and at times, sharp edge.

PDF DOWNLOAD FROM : WWW.SARKARIPOST.IN


Page 62

66 Practice Set 3

First, the earlier integration of the world economy was driven more by technological developments in
transportation and communications than by policy changes. It’s true that British Prime Minister Robert
Peel repealed the Corn Laws in 1846, bringing free trade unilaterally to England in the first dramatic
move away from mercantilism. But none of these policy changes did as much to integrate the world
economy in the latter half of the century as did emerging technological revolutions in transportation by
railways and in the oceans. But today’s most dramatic change is in the degree to which governments
have intervened to reduce obstacles to the flow of trade and investments worldwide.
The novel fear today is that globalisation places limits on the freedom to discharge this critical
responsibility and so the complacent view that there is nothing new about globalisation is simply wrong.
We do need to look at the phenomenon closely, seeking to analyse and address the fears that are novel
and indeed appear to be plausible at first blush.

30. Regarding globalisation, the author is of the opinion that


1. the second wave of globalisation is not as distinct as the first one was.
2.
3.
4.
it is nothing new.
it is a difficult but foolproof method of achieving prosperity.
I N
the apprehensions about this phenomenon may not be entirely justified.
Select the correct answer using the codes given below
(a) 1 and 2 (b) 2 and 3
T .
(c) Only 3 (d) Only 4

1.
O S
31. In the process of globalisation, the feature that is visible more now than in the last century is
the rapid technological change and its widespread applications.
2.
3.
4. P
the emergence of the concept of a welfare state.
the proactive approach being adopted by Governments in the light of inevitable globalisation.

I
the type of policy changes that will facilitate rapid desegregation, in order to protect local
markets.

(a) 1 and 2
R
Select the correct answer using the codes given below

A
(b) Only 1 (c) Only 3 (d) 2 and 4

R K
32. The case of the Corn Laws illustrates that
1. many European countries followed the example of Great Britain.

A
2. the policy changes made in one country did not benefit its trading partner.
3. policy changes did have some role in ushering free trade in England.

S
Select the correct answer using the codes given below
(a) Only 1 (b) Only 2 (c) 1 and 2 (d) Only 3

B Passage 2
According to the reports in Economic Survey, the condition of the macro-economy is positive but it has
not glossed over the challenges. The resilience of our economy is seen in its ability to withstand two shocks
in quick succession. The ripple effects of the global economic crisis that devastated world growth, trade
and finances have persisted in the form of the European fiscal crisis. On the domestic front, the farm
sector that saw a negative growth two years back was further hit by erratic monsoons, severe drought and
unseasonal rains in two successive years. Despite this, the economy is growing at rates seen during the
pre-crisis period. The economy is projected to grow at 9% during the year 2011-12. The services sector, for
long ‘the power house of the economy’, with a more than 60% share of the GDP has started gaining
momentum. Another favourable feature is that India’s demographic dividend is yet to peak. The growing
trend in savings and investment rates should benefit from the gradual withdrawal of stimulus measures by
the government. The survey notes that once the economy operates around full capacity, it is not the saving
and investment rates that will drive growth but skill development and innovation.

PDF DOWNLOAD FROM : WWW.SARKARIPOST.IN


Page 63

Stage 1 Know The Trend 67

33. Which of the following are among the favourable indicators noted by the Economic Survey?
1. The growing trend in saving and investment.
2. India’s favourable population statistics.
3. The gain in the momentum of the service sector.
Select the correct answer using the codes given below
(a) Only 3 (b) 1 and 3 (c) 1 and 2 (d) All of these
34. According to the passage, the Economic Survey can be described most aptly by
(a ) it warns of the challenges the country will face on the economic front
(b ) it points to the positive aspects of the economy
(c ) it is cautiously optimistic, pointing to the favourable aspects without ignoring the challenges
(d ) it emphasizes the important role of foreign investments in the country

35. The strength of the Indian economy is evident from


(a )
(b )
(c )
not falling prey to the ripple effects of the global economic crisis

I N
the growth it has posted despite global economic crisis and negative growth in the farm sector
the service sector being the power house of the economy
(d ) None of the above

T .
B Passage 3
O S
A sanctuary may be defined as a place where man is passive and the rest of nature active. Till quite

I P
recently, nature had her own sanctuaries, where man either did not go at all or only as a tool-using
animal in comparatively small numbers. But now, in this machinery age, there is no place left where man
cannot go with overwhelming forces at his command. He can strangle to death all the nobler wildlife in
the world today. Tomorrow he certainly will have done so, unless he exercises due foresight and
self-control in the meantime.

A R
There is not the slightest doubt that birds and mammals are now being killed off much faster than they
can breed. And it is always the largest and noblest forms of life that suffer most. The whales and

R K
elephants, lions and eagles, go. The rats and flies, and all mean parasites, remain. This is inevitable in
certain cases. But it is wanton killing off that I am speaking of tonight. Civilised man begins by
destroying the very forms of wildlife he learns to appreciate most when he becomes still more civilised.

A
The obvious remedy is to begin conservation at an earlier stage, when it is easier and better in every way,

S
by enforcing laws for close seasons, game preserves, the selective protection of certain species and
sanctuaries. I have just defined a sanctuary as a place where man is passive and the rest of nature active.
But this general definition is too absolute for any special case. The mere fact that man has to protect a
sanctuary does away with his purely passive attitude. Then, he can be beneficially active by destroying
pests and parasites, like botflies or mosquitoes and by finding antidotes for diseases like the epidemic
which periodically kills off the rabbits and thus starves many of the carnivora to death. But, except in
cases where experiment has proved his intervention to be beneficial, the less he upsets the balance of
nature the better, even when he tries to be an earthly providence.

36. Consider the following statements with regard to the passage


1. Parasites have an important role to play in the regulation of populations.
2. The elimination of any species can have unpredictable effects on the balance of nature.
3. Botflies and mosquitoes have been introduced to the area by human activities.
4. Elimination of these insects would require the use of insecticides that kill a wide range of
insects.
Which of the above statement(s) will weaken the author’s argument that destroying botflies
and mosquitoes would be a beneficial action?
(a) 1 and 3 (b) 2, 3 and 4 (c) 1, 2 and 4 (d) All of these

PDF DOWNLOAD FROM : WWW.SARKARIPOST.IN


Page 64

68 Practice Set 3

37. What is the purpose of the last paragraph?


1. Sum up the main points of the author’s argument.
2. Qualify the author’s definition of an important term.
With reference to the above question, which of the statement(s) given above is/are correct?
(a) Only 1 (b) Only 2 (c) Both 1 and 2 (d) None of these

38. It can be inferred that the passage is


(a ) part of an article in a scientific journal
(b ) a speech delivered in a court of law
(c ) part of a speech delivered to an educated audience
(d ) extracted from the minutes of a nature club

39. Examine the information given below.


˜ How many employees of Company P opted for Life Insurance?
Two statements, labelled 1 and 2 are given below. You have to decide whether the data given
in the statements are sufficient for answering the item. Using the data given in the statements,
I N
you have to choose the correct alternative.
Statements
1. 18% of the 950 officer cadre employees and 6% of the 1100 other cadre employees opted for
T .
Life Insurance.

O S
2. 28% of the employees in the age group of 51 to 56 and 17% of the employees in all other age
groups opted for Life Insurance.
(a )
(b )
(c )
P
1 alone is sufficient while 2 alone is not sufficient to answer the question

I
2 alone is sufficient while 1 alone is not sufficient to answer the question
Either 1 or 2 is sufficient to answer the question
(d )
R
Neither 1 nor 2 is sufficient to answer the question

A
40. A distance-time graph is shown below. The graph shows three boys A, B and C running for a
100 m race.

R K 100

A 90 C
Distance (in metres)

80

S 70
60
50
A

B
40
30
20
10
0
5 10 15 20 25 30 35
Time (in minutes)

1. The race was won by C. 2. A was ahead of B upto 70 m.


3. C is overtaking B at 40 m. 4. B ran very slowly from the beginning.
Which of the above statements are correct?
(a) 1 and 2 (b) 1, 2 and 3 (c) 1, 3 and 4 (d) 2, 3 and 4

PDF DOWNLOAD FROM : WWW.SARKARIPOST.IN


Page 65

Stage 1 Know The Trend 69

41. Examine the information given below.


˜ In the city, over 75% of the people are living in slums and sub-standard houses, which is a
reflection on the housing and urban development policies of the government.
The statement given above is followed by three courses of action numbered 1, 2 and 3. You
have to assume everything in the statement to be true and on the basis of the information given
in the statement, decide which of the suggested courses of action logically follow(s) for
pursuing.
Courses of Action
1. There should be a separate department looking after housing and urban development.
2. The policies in regard to urban housing should be reviewed.
3. The policies regarding rural housing should also be reviewed so that such problems could be
avoided in rural areas.
(a) Only 1 (b) Both 1 and 2 (c) Only 2

N
(d) Either 2 or 3

I
42. A group of naughty boys is sitting in 3 rows, all facing South, such that X is in the middle row.
A is just to the right of X in the same row, B is just in front of A and C is behind X. In which
direction of B is C?
(a) North-West (b) North-East (c) South-East
T . (d) South-West

(a) 12% increase (b) 8% increase


S
43. A gardener increases the length of the field which is in rectangular shape by 20% and
decreased the breadth by 10%. Then, the percentage change in the area of the field will be

O
(c) 8% decrease (d) 10% decrease

44. Examine the following information.


˜

I P
The education of a student at college level, not taking into account maintenance

R
expenses, costs four hundred rupees a year. College education is thus drawing heavily
upon the national resources of an impoverished community. So, college education should

A
be restricted to a brilliant few.

K
Two assumptions numbered 1 and 2 are given. You have to decide which of the assumption(s)
is/are implicit in the statement.
Assumptions
R
(a) Only 1
S A
1. Our resources are very limited.
2. Only a few students should be admitted to the colleges.

(c) Either 1 or 2
(b) Only 2
(d) Both 1 and 2

45. Consider the following figure and answer the item that follows.

What is the total number of triangles in the given figure?


(a) 20 (b) 24 (c) 26 (d) 28

PDF DOWNLOAD FROM : WWW.SARKARIPOST.IN


Page 66

70 Practice Set 3

46. The following table shows the total number of students and percentage of boys in different
colleges.
College Percentage of Total number of
boys students
A 64 550
B 54 600
C 25 672

25% of the girls from college C left the college and took admission in college A. Then, what
will be the difference between the number of girls in colleges A and B now?
(a) 40 (b) 44 (c) 48 (d) Cannot be determined

47. Consider the following figures and examine the situation.

I N
When number 1 is at the top, which number will be at the bottom?
(a ) 1 (b ) 2 (c ) 3

T . (d ) 6

S
48. A part of monthly expenditure of Mr X is fixed and remaining varies with the distance
travelled by Mr X. If he travels 200 km in a month, his total expenditure is ` 3300 and if he

O
travels 500 km in a month, his total expenditure is ` 3900. What will be the total expenditure,
if he travels 800 km in a month?
(a) ` 4200 (b) ` 4500

I P (c) ` 4400 (d) None of these

R
49. Average score of Rahul, Suman and Sanjeev is 63. Rahul’s score is 15 less than Pandey and
10 more than Suman. If Pandey scored 30 marks more than the average score of Rahul, Suman

A
and Sanjeev. Then, what is the sum of Suman and Sanjeev?
(a) 120
(c) 117

R K (b) 111
(d) Cannot be determined

50. In an office, 30% of employees are men and remains are women. Also, 60% of men are married

(a) 30%
A
and 66% of the employees are married. What percentage of women are not married?

S (b) 17
1
7
% (c) 31
3
7
% (d) 20%

Directions (Q. Nos. 51-56) Read the following two passages and answer the questions that follow
each passage. Your answers to these questions should be based on the passages only.

B Passage 1
One of the most fateful errors of our age is the belief that ‘the problem of production’ has been solved.
Things are not going as well as they ought to be going must be due to human wickedness. We must
therefore construct a political system so perfect that human wickedness disappears and everybody
behaves well. In fact, it is widely held that everybody is born good; if one turns into a criminal or an
exploiter, this is the fault of ‘the system’. Modern man talks of a battle with nature, forgetting that, if he
won the battle, he would find himself on the losing side. The illusion of unlimited powers, nourished by
technological achievements, has produced the concurrent illusion of having solved the problem of
production.

PDF DOWNLOAD FROM : WWW.SARKARIPOST.IN


Page 67

Stage 1 Know The Trend 71

The later illusion is based on the failure to distinguish between income and capital where this
distinction matters most. Every economist and businessman is familiar with the distinction and applies
it conscientiously to all economic affairs except the irreplaceable capital which man has not made but
simply found and without which he can do nothing. For larger is the capital provided by nature and not
by man and we do not even recognise it as such. This larger part is now being used up at an alarming rate
and that is why it is an absurd and suicidal error to believe and act on the belief that the problem of
production has been solved.

51. According to the author, the problem of production has not yet been solved because
1. inspite of the huge and cry the world over, man is still exploiting his resource base (i.e. nature),
to reduce the level of exploitation.
2. of man’s failure to realise that his economic activities are adversely affecting nature.
3. technology is yet to be transferred to developing countries.
4. the detrimental effect that the new technologies have when used for achieving economic goals.
Select the correct answer using the codes given below
(a) 2 and 3 (b) Only 3 (c) Only 1
I N
(d) Only 4

systems of society.
1. Faulty systems tend to breed wickedness.
T .
52. Identify the statement that cannot be attributed to the author with regard to the control of the

2. A perfect political system would have all good citizens.


Select the correct answer using the codes given below
O S
(a) Only 1 (b) Only 2

I P (c) Both 1 and 2 (d) None of these

53. Why would man find himself on the losing side, if he were to conquer nature for economic
progress?
(a )
(b )
(c )
R
He is not destined to win this one-sided battle

A
The benefits of scientific and technological development are merely illusory
In conquering nature, he would be killing the goose that lays the golden eggs
(d )
K
He is handicapped by a faulty understanding of the issues related to income and capital

R
S A B Passage 2
A study based on a year-long analysis of data from an extensive mobile phone network has produced
interesting information that might be of use to epidemiologists and social scientists. The data, according
to the researchers, might shed light e.g. how diseases and information (or rumours) are transmitted
through social networks. Researchers ramped the like between a pair of phone were on the basis of the
total time spent talking to each other. ‘Strong’ links exist between members of a close social group.
‘Weak’ links tend to be more long range and join individuals from different social groups.
The researchers observed a dramatically different effect when they removed links in the network in rank
order, depending on whether they removed links starting with the strongest or with the weakest. To their
surprise, removing the strong links first had little effect on the overall structure of the network. But removing
weak links first split the network into a series of unconnected islands, with individual users linked to a small
group of other phone users.
Thus, the researchers have hypothesised that the weak links (the more tenuous connections between
individuals from different social groups) might be very important in maintaining wider social cohesion.
If you lose contact with casual acquaintances you may fragment your social circle, but if you stop talking
to your brother there might be less visible impact on the structure of your social network.

PDF DOWNLOAD FROM : WWW.SARKARIPOST.IN


Page 68

72 Practice Set 3

54. Consider the following statements with regard to the passage


1. ‘Weak’ links are more important than ‘strong’ links.
2. Some people believe that phone-network patterns could be useful to social scientists.
3. Links between family members would be disrupted by terminating ‘weak’ links.
4. The ‘strong’ links are between geographically close individuals.
The passage supports which of the following conclusion(s)?
(a) 2 and 4 (b) Only 2 (c) Only 3 (d) 1 and 2
55. Which of the following can be inferred from the passage?
(a) Information transmission through phone networks is essentially the same as information
transmission through face-to-face contact
(b) The phone-network studied had the same number of users throughout the study
(c) The phone users were unaware of the study
(d) The researchers had not anticipated the specific effects of removing weak links

1.
I
To make the ideas more specific to improve the reader’s understanding.
N
56. Consider the following statements regarding the purpose of the last sentence of the passage

2.
3.
4.
To generalise the argument to make it more appealing. .
To provide a practical illustration of the meaning of a strong link.

T
To concretise an argument about the usefulness of the current research.

(a) Only 1 (b) 2 and 3 S


Which of the above statement(s) convey the author’s intentions?

O (c) 1, 2 and 3 (d) 1 and 2

I P
Directions (Q. Nos. 57-60) Examine the information and answer the questions that follow.
Eight persons— A, B, C, D, E, F, G and H are seated in two rows with four persons in each row.

A R
Each person in any row faces exactly one person in the other row. Also,
A is to the immediate left of the person, who is opposite to F.
E is in the same row as B and one of them is opposite to G, who is not at the end.
Â

R K
There is only one person to the left of H, who is not in the same row as C.
D is not opposite to E.
E is between C and A.

A
57. Who are the other three persons in the same row as H?

S
(a) A, G and F
(c) E, D and G
(b) D, G and F
(d) D, A and C

58. Who among the following are diagonally opposite to each other?
(a) A and H (b) F and B
(c) D and B (d) E and G

59. Who is the person between H and D?


(a ) E (b ) F (c ) A (d ) G

60. If the positions of E and B are interchanged, then who is opposite to E?


(a ) A (b ) G (c ) F (d ) B

61. After two successive discounts, a jeans, whose marked price is ` 300 is sold for ` 216. If the
second discount is of 10%, then the first discount (in %) is
(a) 25 (b) 15 (c) 30 (d) 20

PDF DOWNLOAD FROM : WWW.SARKARIPOST.IN


Page 69

Stage 1 Know The Trend 73

62. In the following question, there is some relationship based on a certain rule between the
numerals given in each figure. Find the rule in each case and the correct alternative from
among the four alternatives given under it to fill in the vacant place.
5 3 6
2 3 7

4 548 2 2 694 2 4 ? 2

2 3 1
1 2 1
Fig (i) Fig (ii) Fig (iii)
(a) 678 (b) 769 (c) 824 (d) 937

63. Examine the following arguments.


˜ Some students are laborious. Some engineers are students. No engineer is an officer.
Which of the following conclusions follow(s) the above arguments?
Conclusions
1. Some laborious are engineers.
I N
2. Some students are officers.
3. Some officers are not students.
T .
(a) Both 1 and 4
(c) Only 3

64. Examine the information given below.


S
(b) Both 2 and 3
(d) None of these

O
˜ A studies in which of the schools B, C, D, E and F?

I P
Two statements, labelled 1 and 2 are given below. You have to decide whether the data given

you have to choose the correct alternative.


Statements
A R
in the statements are sufficient for answering the item. Using the data given in the statements,

(a )
R K
1. A does not study in the same school as either R or J.
2. R and J study in schools D and F, respectively.
1 alone is sufficient while 2 alone is not sufficient to answer the question
(b )
(c )
(d )
A
2 alone is sufficient while 1 alone is not sufficient to answer the question
Either 1 or 2 is sufficient to answer the question

S
Neither 1 nor 2 is sufficient to answer the question

65. If Neha says, ‘Amruta’s father Raj is the only son of father-in-law Mahesh’, then how Bindu,
who is the sister of Amruta is related to Mahesh?
(a) Daughter (b) Wife (c) Daughter-in-law (d) Granddaughter

Directions (Q. Nos. 66-69) The following questions are based on the pie-charts given below.
Percentagewise distribution of students studying in Arts and Commerce in seven different
institutions (A, B, C, D, E, F and G).
Total member of students studying Arts = 3800

G A
12% 15% B
F
13% 8%
E C
14% D 17%
21%

PDF DOWNLOAD FROM : WWW.SARKARIPOST.IN


Page 70

74 Practice Set 3

Total number of students studying Commerce = 4200


11% 12%
F G A B
13% 17%
E C
18% D 15%
14%

66. What is the total number of students studying Arts in institutions A and G together?
(a) 1026 (b) 1126 (c) 1226 (d) 1206

67. How many students from institute B study Arts and Commerce?
(a) 1180 (b) 1108 (c) 1018 (d) 1208

68. The ratio of the number of students studying Arts in institute E to that studying Commerce in
institute D is
(a) 12:17 (b) 12:7 (c) 19:21

I N(d) 17:19

(a) 27:14 (b) 19:27 (c) 19:16

T .
69. The ratio of the number of students studying Arts to that studying Commerce in institute E is
(d) 19:28

70. A box contains 2 white balls, 3 black balls and 4 red balls. In how many ways 3 ball be drawn

(a) 32 (b) 48 S
from the box, if atleast one black ball is to be included in the drawn?

O (c) 64 (d) 96

I P
71. From GBPS, a renowned public school of Delhi, 300 students wrote the board examinations,
out of which, 45% got distinction. If 40% of the girls got distinction, the number of boys, who
wrote the examination, is (here, the number of boys, who got distinction is 35 more than the

(a) 120 R
number of girls who got distinction)

A
(b) 150 (c) 175 (d) 125

follows.
R K
Directions (Q. Nos. 72) Read the following information carefully and answer the question

S A
Four young men Anil, Mukesh, Piyush and Yogesh are lovingly called Munna, Babu, Prince and
Papu by everyone. They are married to Madhu, Isha, Jyoti and Arti.
(i) Arti and Madhu are not married to Piyush or Anil nor is their husband called Babu.
(ii) Babu is not married to Isha and his name is not Piyush.
(iii) Isha is not married to Munna.
(iv) Mukesh is neither Munna nor Prince nor is married to Madhu.

72. Which of the following pairs of husband-wife is not correct?


(a) Munna-Madhu (b) Babu-Jyoti
(c) Papu-Jyoti (d) None of these

73. 40% of the students in a college play basketball, 34% of the students play tennis and the
number of students who play both the games is 234. The number of students who neither play
basketball nor tennis is 52%. Find the student population in the college.
(a) 800 (b) 1050
(c) 900 (d) 850

PDF DOWNLOAD FROM : WWW.SARKARIPOST.IN


Page 71

Stage 1 Know The Trend 75

Directions (Q. Nos. 74-76) The following questions are based on two passages in English to test the
comprehension of English language and therefore, these questions do not have Hindi version. Read
each passage and answer the questions that follow.

B Passage 1
Surveillance by law enforcement agencies constitutes a breach of a citizen’s Fundamental Rights of
privacy and the Freedom of Speech and Expression. It must, therefore be justified against compelling
arguments against violations of civil rights. ‘Right to Privacy’ in India has long been considered too
broad and inovalistic to be defined judicially. The judiciary, though, has been careful enough not to
assign an unbound interpretation to it. It has recognised that the breach of privacy has to be balanced
against a compelling public interest and has to be decided judiciously.

74. Which among the following is the most rational and critical inference that can be made from
the above passage?
(a) The Right to Privacy is a natural right duty supported by statutes.
I N
T .
(b) There have been instances of surveillance by law enforcing agencies resulting in breach of a
citizens fundamental rights.
(c) The judiciary has come to individual as well as government’s rescue by pronouncing a very
judicial use of this Right to Privacy. An individual has all the Right to Privacy but at the same

O S
time, for the larger interest of nation the government has to adhere to surveillance of certain
suspect individuals. A balanced and judicious use of this right is the only solution.
(d) The comprehensive idea of national interest implies the breach of individual’s right to freedom

P
of speech and expression and violation of ‘Right to Privacy’ is justified.

I
B

A R
Passage 2
In India, extra-judicial killings by the police or the security forces are called ‘encounter killings’, meaning

K
that the killing occurred during an armed encounter between the police or security forces and the victim.
The killing by the state forces is most often declared to be defensive, cases of attempted murder and

R
other related offences are registered against the victims and the cases closed without further

S A
investigation since criminal cases come to an end upon the death of the accused. Despite being
‘unnatural deaths’ and the victim having being killed, no investigation ensues to determine whether the
death was in fact in an actual encounter, nor whether the use and the extent of use, of force was justified.
This is an acknowledged strategy of the state for eliminating certain kinds of opposition to the state and
the established order. In Andhra Pradesh, for instance, the naxalites have been the targets; in Punjab, it
was the militant; in Mumbai, it is those who are alleged to be part of the underworld.

75. Accrding to the passage,


1. the armed and security forces are rarely held accountable for the commission of extrajudicial
killings.
2. extra-judicial killings are also known as encounter killings in India.
3. encounter killings have been particularly prevalent in volatile regions such as Jammu and
Kashmir, Panjab and crime-infested areas of Mumbai.
Which of the statements given above is/are in correct?
(a) Only 1 (b) 1 and 2 (c) 1, 2 and 3 (d) All of these

PDF DOWNLOAD FROM : WWW.SARKARIPOST.IN


Page 72

76 Practice Set 3

76. Which of the following explains the status of extra judicial killings in India?
1. Encounter killings also have been employed in efforts by Indian police and security forces to
quash the Maoist/Naxalite armed rebellion.
2. The fabrication of evidence appears to be a regular feature of fake encounter killings.
Select the correct answer using the codes given below.
(a) Only 1 (b) Only 2 (c) Both 1 and 2 (d) Neither 1 or 2

Directions (Q. Nos. 77-80) Given below are four questions. Each question describes a situation
and is followed by four possible responses. Indicate the response you find most appropriate. Choose
only one response for each question. The responses will be evaluated based on the level of
appropriateness for the given situation. Please attempt all questions. There is no penalty for wrong
answers for these four questions.
77. You are a high-ranking official in a government department. One of your colleagues has been

I N
illegally using his official vehicle for personal purposes though the vehicle has been allotted to
him for official work only. You also come to know that he regularly submits fake bills to the
office in order to earn some extra money. What would you do?
(a) Talk to your colleague and advise him to mend his ways
(b) Report the issues to his immediate boss
T .
(c) Share the matter with your colleagues and ask them to spread the word about your colleague’s
improper activities

O S
(d) Pay no attention to the matter since it does not affect you in any manner

P
78. You are the General Manager of District Industry Centre. A person has decided to start a new
business and he has come to meet you with a proposal in the same context. He has already met

I
your subordinate and enquired about the existing scenario before designing the proposal. As a
GM, what will you do?

A R
(a) Know the financial capability of the person to meet the initial expenditure
(b) First see his proposal and its viability

R K
(c) Ask him to visit some industrial units similar to one that he has proposed
(d) Advise him to fill up the forms and submit the proposal and confirmation from a bank by
which he intends to finance his project

S A
79. You are an Income Tax Commissioner. You come to know about tax evasion by a very reputed
firm. You got this information from one of your very reliable officers. You know that the firm
has a very clean image but the officer is sure about the tax evasion. He is a very honest officer.
You will
(a) Conduct a raid immediately
(b) Consult your predecessor who worked in the area for long as his advice could be valuable in
implementing an effective decision
(c) Send a notice to the firm
(d) Call a meeting of some reliable officers and gather all the inputs
80. You are a traffic police officer and are on night duty. You have instructed a car to stop as it was
passing a red light. As you step over to the car, you recognise the driver to be an MLA. When
you approach him, he tells you that he misjudged the light and he sincerely apologises for it.
What would you do?
(a) Tell the MLA that you know who he is but you are just doing your duty
(b) Tell the MLA that you will excuse him from this traffic violation but that he should be careful
in future
(c) Tell him that he can go as he is the MLA of the city
(d) Salute the MLA and let him go

PDF DOWNLOAD FROM : WWW.SARKARIPOST.IN


Page 73

Stage 1 Know The Trend

Answer with Explanations


1. (d) Rural development means lives of rural people get 9. (d) Options (a) and (b) have been talked about in the
changed due to land reforms, more educational facilities, passage. It also talks of regional imbalances and the need
credit availability at easy terms, health and sanitation. All for a more balanced policy of regional development.
this is possible through decentralisation of powers and in Hence, we can summarise that regional development
rural areas Panchayati Raj System is the best example of policies have failed to achieve their targets. Thus, all of the
an instrument responsible for economic development of choices are responsible for population explosion.
rural masses.
10. (a) The passage talks of integrating population policy with
2. (c) Deontological pacifism stands against violence or health and education policies. Option (a) is the primary
aggression of any kind and supports non-intervention in aim of the Government.
the face of even a mortal danger irrespective of who the
11. (a) The last few lines of the passage indicate that the
victim is. So, Statement 1 is ruled out as a covert violent
population policy should also focus on empowerment of
approach is against its principles. Statement 2 too is ruled
out for the same reason. Statement 3 advocates peace in
the face of aggression and hence is in concord with its
idealism.
I N
people so that they become assets and not liabilities.
12. (c) Option (c) is correct as both statements 1 and 2 are true.
13. (b) India is a vast country. Penetration of internet and
3. (b) The author is not in agreement with the ideas of pacifism
since he foresees problems if these are followed. The
author agrees with Statement 1 which implies that there
T .
smartphones has really helped e-commerce industry in
India to register high growth.
14. (c) ‘No student is lazy’ can be represented as
could be situations where use of force becomes pertinent
and which go against the pacifist’s ideals. According to
Statement 2, these ideals cause no collision of duties. The
passage cites various examples to illustrate otherwise.
O S Student Lazy

Hence, the author is unlikely to agree with it. Statements 3


and 4 are situations that go against pacifism.
4. (d) While Statements 1 and 2 are supported by the passage,
I P ‘All the lazy persons
represented as
are unsuccessful’ can be

but Statement 3 is not.

clear that Edward Lorenz’s findings


A R
5. (d) The first paragraph of the passage makes it specifically
led to the
Lazy Unsuccessful

developmont of chaos theory.

R K
6. (c) Only 3, from second passage it is clear that the poppy
seeds are captured by ‘strange attractors’ staggeringly
On combining the two, we get

S A
complex pathways, whose tangle appear accidental.
7. (d) In the last paragraph, the passage states that the Earth’s
weather may be an example of a chaotic system. So,
Statement 2 can be left out. Poppy seeds placed on an
Student

Unsuccessful
upside-down bowl are described in the second paragraph Lazy
as an example of a non-chaotic system that creates (i)
divergence. So, Statement 3 is not the correct answer. Or
Butterfly flight patterns are nowhere mentioned as a
system. According to the last paragraph, the ‘Butterfly
Effect’ is caused by the flapping of a single butterfly’s Student
wings to potentially affect atmospheric systems. So,
option (d) is the right answer.
Unsuccessful
8. (d) Option (d) two baseballs placed into an active tumble Lazy
dryer are analogous to two poppy seeds placed in ‘bread (ii)
dough being mixed by a machine’: parts of the system are
separated, intermingled and brought back together again In Fig. (ii), ‘some students are unsuccessful’ and ‘some
in perfectly regular, though complex, ways. unsuccessful persons are students’. So, options (a) and
(b) are not valid. Also, only ‘some unsuccessful persons
The pathways of the two baseballs will diverge and
are lazy’ and not ‘all’. So, option (c) is valid and option (d)
converge repeatedly, as in any other chaotic system.
is not valid.

PDF DOWNLOAD FROM : WWW.SARKARIPOST.IN


Page 74

80 Practice Set 3

15. (b) The inability of the small banks to compete with the bigger Now, put the value of y in Eq. (i), we get
ones shall not ensure security and good service to the 2 x + 3 × 175 = 1025
customers, which is an essential concomitant that has to be ⇒ 2 x = 1025 − 525
looked into by the Reserve Bank. Statement 1 seems to be a
⇒ 2 x = 500 ⇒ x = 250
remedial step for the same.
Then, x − y = 75
16. (d) Option (d) explains the relation among factory, product and Hence, difference between cost of one table and one
machinery. chair = `75
22. (d) Let t denotes the time.
So, t = LCM (t AB , t BC ) [relative speed]
Product 1000 1000
∴ t AB = = 200 s, t BC = = 200 s
Machinery 15 − 10 20 − 15
Factory So, after 200 s they will meet first time.

17. (a) Here, A → All Solutions (Q. Nos. 23-24)


I → Some
E → No
Statement 1 of argument + Statement 2 of argument gives no
Babu(+) N
From the given information the blood relation is as follows.
Amita(–) Brother

I Diwakar(+)

conclusion [ QI + I = no conclusion].
Similarly, Statement 2 + Statement 3 gives no conclusion.
[Q I + I = no conclusion]. Also, Statement 3 + Statement 4
(+) Eshoo

T . Husband
Mother
Chanda(–)

Here, (+ ) = male, (− ) = female


gives no conclusion due to same reason. Hence, no
conclusion follows.
18. (d) According to the question,
O S and (↔) = married couples
23. (d) From above diagram, it is clear that Babu is
father-in-law of Eshoo.
C ⇔ E Opposite face
A ⇔ D Opposite face
B ⇔ F Opposite face

I P 24. (c) From above diagram, it is clear that Amita is


mother-in-law of Eshoo.

R
Hence, B, C, E and F would be the faces adjacent to face A. 25. (c) Original quantity of milk in the mixture = 40 × 3/5 = 24 L
19. (d) Original quantity of water in the mixture = 40 × 2/5

6
1
2

5
3
4

K A =16 L
After first operation, milk = (24 – 6+ 10) = 28 L
water = (16 – 4) = 12 L

R
From above figure, it is clear that we can place only six coins

A
where one coin is in the centre as shown in the above
[Q during first operation, (10 × 3/ 5 = ) 6 L milk and
(10 × 2 / 5 = ) 4 L water is removed and 10 L milk is
added)

20.
diagram.

S 8
(a) Hire charges = (4 × 60) + (5 × 60) + × (320 − 120)
5
= 240 + 300 + 320 = ` 860
After second operation, Milk = (28 – 7+ 10) = 31 L
Water = (12 – 3) = 9 L
[during second operation, 7 L milk and 3 L water is
removed and 10 L milk is added]
Let Suman had initially be ` x. Hence, final ratio of milk and water = 31: 9
1
Then, x − 860 = × 860 ⇒ x = ` 1075 26. (a) Marked price of an article = ` 280
4
Marked price × Discount percentage
Discount =
21. (a) Let cost of each chair be ` x 100
280 × 10
and cost of each table be ` y. = = ` 28
According to the question, 100
2 x + 3 y = 1025 ...(i) ∴ Salling price = Marked price − Discount
3x + 2 y = 1100 ...(ii) = 280 − 28 = ` 252
Multiplying Eq. (i) by 3, Eq. (ii) by 2 and then subtracting Eq. Now, SP = ` 252 and profit = 26%
100
(ii) from Eq. (i), we get ∴ CP = × SP
100 + Profit %
6x + 9 y = 3075
100 100
− 6x ± 4 y = − 2200 = × 252 = × 252 = `200
100 + 26 126
5 y = 875
⇒ y = 175 Hence, the actual cost of the article is ` 200.

PDF DOWNLOAD FROM : WWW.SARKARIPOST.IN


Page 75

Stage 1 Know The Trend 81

27. (d) Given, total distance = 180 km 39. (a) From Statement 1,
Distance covered in first 3 h = 50 × 3 = 150 km Total number of employees who opted for Life Insurance
Distance covered in last hour = 180 − 150 = 30 km = (18% of 950 officer employees) + (6% of the 1100 other
Distance covered in last hour cadre employees)
∴ Speed in last hour = 18 6 17100
Time = × 950 + × 1100 = + 66 = 171 + 66 = 237
100 100 100
30
= = 30 km/h
1 From Statement 2,
Number of employees who opted for Life Insurance =( 28% of
28. (a) As per the given information, final ranking of height in employees in the age group of 51 to 56) + (17% of employees
descending order is in all other age groups opted for Life Insurance).
Roshan > Susheel > Hardik > Niza > Harry It is clear from the Statement 2 that we cannot get the required
Therefore, the tallest person is Roshan. answer until and unless the number of employees in age
29. (c) From Statements 1 and 2, group of 51 to 56 and other age group is known.
mn = 221 = 17 × 13 So, Statement 1 alone is sufficient while Statement 2 alone is
not sufficient to answer the question.
∴ m = 17 and n = 13
∴Total number of games = 13
C2 =
13 × 12 × 11!
11! 2 !
= 78

I N
40. (b) Boy C had covered the maximum distance in minimum time.
Also, he reached the 100 m mark in the shortest time. So, race
was won by C. A was ahead of B upto 70 m and then B
Hence, the total number of students that participated
in the competition is 30 i.e. (17 + 13) and the number
of games in which both players were girls, is 78.

T .
overtake A at 70 m. Also, C is overtaking B at 40 m, where boy
C crosses the boy B. So, Statements 1, 2 and 3 are correct.
Boy B was running faster than C and he covered more
30. (d) The opening sentence of the passage says ‘the perils
tend to be exaggerated’, which means that regarding
globalisation, the author is of the opinion that the
apprehensions about this phenomenon may not be
S
distance than C, upto 40 m. So, it is not true that B ran very
slowly from the beginning. Hence, the answer is option (b).

O
41. (b) The statement talks of housing conditions in urban areas only.
entirely justified.
31. (c) Refer to the lines ‘…..most dramatic change is in
degree to which governments have intervened to
I P So, 3 does not follow. Also, to improve the deteriorating housing
conditions, the urban housing policies need to be studied and
the lacunae removed by a team of efficient personnel deployed

reduce obstacles to the flow of trade…..’


32. (d) Refer to the lines ‘…….. repealed the Corn laws …..
bringing free trade …..’.
A R for the same. So, both 1 and 2 follow.
42. (b) According to the given information, sitting arrangement of the
boys will be as follow.

and tenth sentences of the passage.

R K
33. (d) All the three statements are mentioned in eighth, ninth

34. (c) This option is clearly mentioned in the first sentence of


Facing
A
C
NW

W
N
NE

A
the passage according to the passage, the Economic

S
Survey can be described as optimistic, pointing to the
favourable aspects without ignoring the challenges.
35. (a) The second, third and fourth sentences of the
South

B
X
SW
S
SE

From above figure, it is clear that C is in the North-East


passage bring out this option as the correct one. direction of B.

36. (c) We have to find the statements that weaken the idea 43. (b) Let length of field be x and breadth be y.
that destroying botflies and mosquitoes is beneficial. Area = xy
Statements 1, 2 and 4 give reasons why destroying 120 6x
Now, length after 20% increase = x × =
these insects might create more problems. 100 5
90 9y
37. (b) The last paragraph is giving details of the modifying or Breadth after 10% decrease = y × =
qualifying that the author has done in the third 100 10
54xy
paragraph of the earlier definition of a sanctuary. So, New area =
only Statement 2 is correct. 50
Now, increase in area = xy  − 1 =
54 4xy
38. (c) From the words ‘I am speaking of tonight’ (middle of  50  50
the second paragraph) we can infer that the words
4xy
were delivered orally and not during the daytime. So,
the only possible answer is option (c). ∴ Percentage change = 50 × 100 = 8%
xy

PDF DOWNLOAD FROM : WWW.SARKARIPOST.IN


Page 76

82 Practice Set 3

44. (d) The use of the words ‘impoverished community’ in the 49. (b) Total score of Rahul, Suman and Sanjeev = 63 × 3 = 189
statement makes 1 implicit while the phrase ‘college
Pandey’s score = 63 + 30 = 93
education should be restricted to a brilliant few’ makes 2
implicit. Rahul’s score is 15 less than Pandey’s score.
∴Rahul’s score = 93 − 15 = 78
45. (d) B C
A Hence, sum of Suman and Sanjeev = 189 − 78 = 111
K O
U S 50. (c) Let total number of employees be 100.
I
L P
D N Men = 30% = 30
J R
H T Married men = 30 × 60% = 18
M Q
E Women employees = 70
G F
Married women = 66 − 18 = 48
Total number of triangles = AKU, AIR, AGR, ABR, ACR
Unmarried women = 70 − 48 = 22
ACE, AGE, BCR, COS, CDR, CER, DER, EQT, EFR, EGR,
22 3
FGR, GHM, GIR, HJU, JKM, KLR, KMR, LMR, NST, NOQ, ∴Percentage of unmarried women = × 100 = 31 %
70 7
OQR, OPR, PQR = 28
46. (c) Number of girls in college A =

=
(100 − 64)
100
36 × 55
× 550

= 198
Alternate Method

I N 100

Number of girls in college B =


10
(100 − 54)
100
× 600

T .30
M W

70
= 46 × 6 = 276
Number of girls in college C =
(100 − 25)
100
× 672

O S Unmarried

12
Married

18
Unmarried

22
Married

48
=
75
100
× 672 = 504

Now, 25% of girls of college C = 504 ×


25 504
100
=
4
= 126
I P ∴Required percentage =

Hence, option (c) is correct.


22
70
3
× 100 = 31 %
7

So, 126 girls took admission in college A.

A R
∴ Total number of girls in college A = 198 + 126 = 324
51. (d) According to the author,the problem of production has not
yet been solved because of illusion of unlimited
powers,nourished by technological achievements. It has

and college B = 324 − 276 = 48

R K
Now, difference between the number of girls in college A

47. (d) From the given condition of dice, we can say that number
produced the concurrent illusion of having solved the
problem.
52. (a) Refer to line of the passage ‘human wickedness
2 is common in both.

S A
∴ Opposite side of number 2 = Number 4
Opposite side of number 5 = Number 3
Opposite side of number 1= Number 6
disappears and everybody behaves well. In fact, it is
widely held that everybody is born good’, so, option (a)
can be attributed to a ‘widely held’ belief not to the author.
53. (c) Refer to passage, we are using up the ‘irreplaceable
Hence, when number 1 is at the top, number 6 will be at capital’ and option (c) shows an irreplaceable loss.
the bottom.
54. (b) Statement 2 is clearly implied in the first paragraph.
48. (b) Let fixed expenditure be ` x Statement 3 can be eliminated because weak links are
and variable be y per km. less likely to be between family members. Also, since
geographical distances are not mentioned anywhere in
According to the question,
the passage, Statement 4 is incorrect.
x + 200 y = 3300 ...(i)
x + 500 y = 3900 ...(ii) 55. (d) There is no mention anywhere in the passage that the
number of users was constant or that the users were
On solving Eqs. (i) and (ii), we get
aware that their network was being monitored; hence
300 y = 600 options (b) and (c) are incorrect. Also, we can eliminate
⇒ y=2 option (a) because the words ‘essentially the same’ in it
Now, x = 3300 − 400 cannot be justified.
= 2900 56. (d) The author is making a point clear for the reader, so
∴Total expenditure on travelled 800 km Statement 1 is the correct choice. Statement 2 is too
= 2900 + 2 × 800 narrow as the author is doing more than just giving an
= ` 4500 illustration of a ‘strong’ link.

PDF DOWNLOAD FROM : WWW.SARKARIPOST.IN


Page 77

Stage 1 Know The Trend 83

Solutions (Q. Nos. 57-60) 66. (a) Total number of students studying Arts in institutions A
From the given information, arrangement can be drawn as and G together
given below. (15 + 12 )
= 3800 × = 38 × 27 = 1026
D G H F 100
• • • •
• • • • 67. (c) Number of students studying Arts in institute B
C E A B 8
= 3800 × = 304
57. (b) The other three persons in the same row as H are D, G and 100
F. Number of students studying Commerce in institute B
17
58. (c) D and B are diagonally opposite to each other. = 4200 × = 714
100
59. (d) The person between H and D is G. ∴Total number of students = 304 + 714 = 1018
60. (c) If the positions of E and B are interchanged, then the 68. (c) Number of students studying Arts in institute E
person opposite to E is F. 14
= × 3800 = 532
61. (d) Let the first discount be of x %. 100


Discount = 300 − 216 = ` 84
Now, 10% of 300 + x% of (300 − 10% of 300) = 84
30 +
x
× 270 = 84 ⇒ x = 20%
= 4200 ×

∴Required ratio =
532
I N
Number of students studying Commerce in institute D
14
100
= 588

= 19 : 21
62. (a) In fig (i), 5 × 1 = 5
2 ×2 = 4
100

⇒ (548)
T . 588
69. (b) Number of students studying Arts in institute E
14
4×2 = 8
In fig (ii), 3 × 2 = 6
3×3=9 ⇒ (694)
O S = 3800 ×
100
= 532

Number of students studying Commerce in institute E


= 4200 ×
18
= 756
2 ×2 = 4
Similarly, in fig (iii) 6 × 1 = 6
7 × 1= 7 ⇒ (678)
I P 100
∴Required ratio = 532 : 756 = 19 : 27
70. (c) We have the various combinations as given below.
4×2 = 8
63. (d) Statement 2 of the argument can be converted to ‘Some
students are engineers.’
A R (1 black and 2 non-black) Or (2 black and 1 non-black) Or
(3 black)
∴Required number of ways

other conclusion is possible.


K
This plus Statement 3 gives no conclusion ( I + E = 0). No,

R
64. (d) As given in Statements 1 and 2, R studies in school D and
= ( 3 C1 × 6C 2 ) + ( 3 C 2 × 6C1 ) + ( 3 C 3 )
= 45 + 18 + 1= 64
71. (c) QNumber of students who got distinction

S A
J studies in school F. So, A does not study in school D or
school F. Thus, A studies in anyone of the schools B, C or
E. So, neither Statement 1 nor Statement 2 is sufficient to
answer the question.
= 45% of 300
= 135
Let the number of girls who got distinction be x.
Then, the number of boys who got distinction
65. (d) According to the given information,
= (x + 35)
(+) Mahesh
∴ x + (x + 35) = 135
Father-in-law
Father ⇒ 2 x = 100
(+) Raj Neha (–) Granddaughter ⇒ x = 50
Couple
Again, let the number of girls who wrote the exam be n,
Father
then
n =   × 100 = 125
(–) Amruta Bindu (–) 50
Sister  40 
From above relation tree, we can say that Bindu is the So, the number of boys who wrote the exam
granddaughter of Mahesh. = 300 − 125 = 175

PDF DOWNLOAD FROM : WWW.SARKARIPOST.IN


Page 78

84 Practice Set 3

72. (c) On the basis of given information, we can summarise the data in 77. (a) Since the person involved is your colleague, it is
tabular form as given below. reasonable for you to talk to him and give
meaningful advice. If he further violates the rules,
Nickname Wife
Name you should report the issues to his immediate
Munna Babu Prince Papu Madhu Isha Jyoti Arti
boss. Option (c) may give your colleague a bad
Anil û ü û û û û ü û name in the office. You should follow proper
Mukesh û û û ü û û û ü channels instead of randomly spreading negative
Piyush û û ü û û ü û û information about someone. Option (d) does not
Yogesh ü û û û ü û û û take care of the issue at all shows your
irresponsible attitude.
Hence, it is clear from the above table that, Pappoo (Mukesh) is
married to Arti. 78. (d) As the person has already met the subordinate and
designed the proposal after considering all the
73. (c) Let the total number of students be N. scenarios, so he is now in need of guidance and
40N 34N help. As a GM, you should ask him to fulfil the
Again, let n( A) = , n(B) =
100 100 procedural formalities and help him in submitting

N
52 N the form, so that he can start his business as soon
and n( A ∪ B) =
100 as possible.
Q n( A ∪ B) = 1 − n( A ∪ B)
= 1−
52 N 48N
100
=
100
. I
79. (d) Arranging a meeting will give you a chance to
know about all facts and information about the
firm as well as about tax evasion. The various
Q


n( A ∩ B) = n( A) + n(B) − n( A ∪ B)
234 =
40N 34N 48N
+
100 100 100
234 × 100 = 26N

S T
officers who work in this field will suggest some
points about the issue.
80. (a) As an officer, it is unethical to give special favours

⇒ N = 900
74. (c) The judiciary has been wise enough to state that in the compelling
P O to anyone (whether MLA or MP) who violates the
law. If you let him go without proper investigation,
then it is a form of misconduct. So, option (a) is the
right choice.
public interest, ‘Right to Privacy’ can be modified. But judiciary
has recognised the importance of ‘Right to Privacy’ as a
fundamental right of any individual.

R I Option (b) shows that you are letting the MLA go


after giving a warning because he is an MLA. You
should always remember that any person who

from lines 3 to 5 and last 2 lines.

K A
75. (d) All the 3 statements can be identified indirectly in the passage

76. (c) Last 3 lines of the passage explains where and why extra judicial
violates the law is just a culprit and you should
treat every person equally before law.
Option (c) is unethical and against the

doubt in India..

A R
killings are operated in India and lines 8, 9 explain, there are
occurrence of fake encounters or encounters that are subjected to
responsibilities of your duty as a traffic police
officer. Option (d) shows your cowardice and
selfish attitude.

PDF DOWNLOAD FROM : WWW.SARKARIPOST.IN


Page 79

CSAT
C i v i l S e r v i c e s A p t i t u d e Te s t
Paper 2

Practice Set 4
I N
Time : 2 hrs
T . MM : 200

Instructions
1. There are 80 questions in this paper.
O S
There is only one answer to be selected by you.
Penalty for wrong answer
I P
2. The answer of any question you are thinking that more than two answers are true, then you must choose the nearest one.

deducted in your total marks.

A R
3. There are four alternative answers in every question. When you select a wrong answer, then 1/3rd mark of that question is

4. If any candidate gives more than one answer and one of them is true but it is treated as a wrong answer and the candidate

R K
is penalised for that and1/3rd marks will be deducted.

Directions (Q. Nos. 1-10) Read the following five passages and answer the questions that

A
follow each passage. Your answers to these questions should be based on the passages only.

S B Passage 1
TREND

In the approach to the Smart Cities Mission, the objective is to promote cities that provide core
infrastructure and give a decent quality of life to its citizens, a clean and sustainable environment and
application of ‘Smart’ solutions. The focus is on sustainable and inclusive development and the idea is
THE

to look at compact areas, create a replicable model which will act like a light house to other aspiring
cities. The Smart Cities Mission of the government is a bold, new initiative. It is meant to set examples
that can be replicated both within and outside the Smart City, catalysing the creation of similar Smart
KNOW

Cities in various regions and parts of the country.


The purpose of the Smart Cities Mission is to drive economic growth and improve the quality of life of
people by enabling local area development and harnessing technology, especially technology that leads
to Smart outcomes. Area-based development will transform existing areas (retrofit and redevelop),
Stage 1

including slums, into better planned ones, thereby improving live ability of the whole city.

PDF DOWNLOAD FROM : WWW.SARKARIPOST.IN


Page 80

86 Practice Set 4

New areas (greenfield) will be developed around cities in order to accommodate the expanding
population in urban areas. Application of Smart Solutions will enable cities to use technology,
information and data to improve infrastructure and services. Comprehensive development in this way
will improve quality of life, create employment and enhance incomes for all, especially the poor and the
disadvantaged, leading to inclusive cities.

1. According to the passage which of the following statements is/are true?


1. There is a great and urgent need of a comprehensible development of physical, institutional,
social and economic infrastructure. Development of Smart City is a step in that direction.
2. The focus of Smart City project is on sustainable and inclusive development which will
transform existing areas (retrofit and redevelop, including slums into better planned cities).
3. Smart City Mission is very ambitious and not likely to succeed in India.
Select the correct answer using the codes given below.
(a) Only 1 (b) Only 3 (c) Both 1 and 2 (d) None of these

B Passage 2 I N
T .
The median legal age for drinking in India is 21 years. For driving and voting, it is 18 years. The Juvenile
Justice Act (JJA), 2015, passed by both (otherwise unproductive) Houses of Parliament lowers the age

O S
for Juveniles to be tried as adults for some ‘‘serious and heinous’’ crimes to 16 years. Is that wise?
As one UK penal reform note has it, ‘‘depriving children of their liberty can lead to long-term and costly
psychological and physical damage, whilst overcrowding and poor detention conditions threaten their

I P
development, health and well-being. The removal of children from networks as well as from educational
or vocational opportunities at critical and formative periods in their lives can compound social and
economic disadvantage and marginalisation. Exposure to criminal influences and violent behaviour in

A R
detention and in the worst instances, exposure to adult offenders, is likely to encourage repeat offending.’’
Neither retribution nor rehabilitation will likely be served.

implementation.

R K
In any case, the real issue with Juvenile law in India is less about the law and more about its

2. Which is the critical inference of the given passage?

S A
(a) Recently hastily passed Juvenile Justice Act, is not a wise decision.
(b) In India, the real issue with Juvenile law is more about its correct implementation.
(c) A lot of Indians are emotional and impulsive and not enough reasoning is behind passing of JJA
(d) A Juvenile criminal needs a different type of atmosphere which is not available in India’s reform
homes.

B Passage 3
We can define a Payment Bank in India as a type of bank which is a non-full service niche bank. A bank
licensed as a payment bank can only receive deposits and provide remittances. It cannot carry out
lending activities. Thus, payment banks can issue ATM/debit cards, but cannot issue credit cards as they
are not empowered to carry out lending activities these banks have been created to help India reach its
financial inclusion targets. This type of bank can be highly useful for migrant labourers, low income
households, small businesses, and other unorganised sector entities.
What kind of impact will payment banks and small finance banks will have on existing banking
customers? ‘‘We expect the existing digital banking customers to try services of payments banks that
will kick off operations. Some of these banks, which are backed by deep pockets, can create disruptive
offerings in the Indian market. We look forward to innovation and high competition.

PDF DOWNLOAD FROM : WWW.SARKARIPOST.IN


Page 81

Stage 1 Know The Trend 87

3. Which among the following is the most logical corollary to the above passage?
(a) Government’s decision to introduce new type of payment bank is going to help large number of
people belonging to unorganised sectors.
(b) Introduction of payment banks will not help in achieving the targets of financial inclusion of
larger number marginalised people.
(c) The payment banks will create disruptive offerings in the Indian markets.
(d) Introduction of payment bank will help the rich investors only.

B Passage 4
New discoveries frequently undermine accepted findings and give rise to new theories in Archaeology, like
Physical Sciences. This trend can be seen in the reaction to the recent discovery of a set of 3.3-million year old
fossils in Ethiopia, the remains of the earliest well-preserved child ever found. The fossilised child was

N
estimated to be about 3 years old at death, female and a member of the australopithecus afarensis species. The
afarensis species, a major human ancestor, lived in Africa from earlier than 3.7 million to 3 million years ago.

I
‘Her completeness, antiquity and age at death make this find of unprecedented importance in the history of
paleo-anthropology’, said Zeresenay Alemseged, a noted paleo-anthropologist.

.
Prior to this discovery, it had been thought that the afarensis species had abandoned the arboreal habitat of

T
their ape cousins. However, while the lower limbs of this fossil supported findings that afarensis walked

S
upright, its gorilla-like arms and shoulders suggested that it retained the ability to swing through trees. This
has initiated a re-examination of many accepted theories of early human development. Also, the presence of

O
a hyoid bone, a rarely preserved bone in the larynx that supports muscles of the throat, has had a

P
tremendous impact on theories about the origins of speech. The fossil bone is primitive and more similar to
that of apes than to that of humans, but it is the first hyoid found in such an early human related species.

4. The passage quotes Zeresenay Alemseged in order to


I
1.
2.
A R
provide evidence to qualify the main idea of the first paragraph.
question the claims of other scientists.
3.
4.
K
provide evidence to support the linguistic abilities of the afarensis species.
provide evidence that supports the significance of the find.

R
Which of the statements given above is/are correct?
(a) Only 1

S A (b) Only 2 (c) 1 and 3 (d) Only 4

5. It can be inferred from the passage’s description of the discovered fossil hyoid bone that
1. Australopithecus afarensis were capable of speech.
2. the discovered hyoid bone is less primitive than the hyoid bone of apes.
3. the discovery of the hyoid bone necessitated the re-examination of prior theories.
Which of the statements given above is/are correct?
(a) Only 1 (b) 2 and 3 (c) 1 and 3 (d) Only 3

6. According to the passage, the impact of the discovery of the hyoid bone in the field of
Archaeology could best be compared to which one of the following examples in another field?
1. The discovery and analysis of cosmic rays lend support to a widely accepted theory of the
origin of the universe.
2. The original manuscript of a deceased 19th century author confirms ideas of the development
of an important work of literature.
3. Newly revealed journal entries by a prominent Civil War era politician lead to a questioning of
certain accepted historical interpretations about the conflict.
Which of the statements given above is/are correct?
(a) Only 1 (b) 1 and 2 (c) Only 3 (d) None of these

PDF DOWNLOAD FROM : WWW.SARKARIPOST.IN


Page 82

88 Practice Set 4

B Passage 5
Sea ice has a bright surface, so much of the sunlight that strikes it is reflected back into space. As a result,
areas covered by sea ice do not absorb much solar energy. Sea ice also affects the movement of ocean
waters.
Water below sea ice has a higher concentration of salt and is denser than surrounding ocean water and
so it sinks. In this way, sea ice contributes to the ocean’s global ‘conveyor-belt’ circulation. Cold, dense,
polar water sinks and moves along the ocean bottom towards the equator, while warm water from
mid-depth to the surface travels from the equator towards the poles. Changes in the amount of sea ice
can disrupt normal ocean circulation, thereby leading to changes in global climate.
‘Satellites can provide detailed measures of how much ice is covering the pole right now, but sediment
cores are like fossils of the ocean’s history’, said Mr Gregery. Sediment cores are essentially a record of
sediments that settled at the sea floor, layer by layer and they record the conditions of the ocean system
during the time they settled.

I N
Scientists can search for a bio-chemical marker that is tied to certain species of algae that live only in ice.
Polar bears, whales, walrus and seals are changing their feeding and migration patterns. Both the
atmospheric currents and the ocean currents can be expected to change.

1. Sediment cores.
T .
7. According to the passage, what helps Mr Gregery study the ocean’s history?

2. Bio-chemical markers.
Select the correct answer using the codes given below.
(a) Only 1
O S (b) Only 2
(c) Both 1 and 2

I P (d) None of these

8. Which of the following statements is supported by the author about the usage of bio-chemical
markers?

A R
1. To find the history of ocean structure.
2. To find the existence of ice at that time.

R K
3. To find the changes in total volume of ice.
4. To find the changes in surface area.
Which of the statements given above is/are correct?

S A
(a) 1 and 2
(c) 3 and 4

9. The second paragraph in this passage explains


(b) Only 2
(d) Only 3

1. the formation of sea ice.


2. rise of temperature on Earth due to sea ice.
3. how the ocean currents are formed.
Select the correct answer using the codes given below.
(a) Only 1 (b) Only 2
(c) 1 and 2 (d) Only 3
10. All of the following are possible side effects of sea ice melting except
(a ) change in ocean current
(b ) changes in migration patterns of polar bears
(c ) increase in temperature
(d ) increase in the density of Arctic sea water

PDF DOWNLOAD FROM : WWW.SARKARIPOST.IN


Page 83

Stage 1 Know The Trend 89

11. A dishonest milkman professes to sell his milk at cost price, but he mixes with water and there
by gains 25%. The percentage of water in the mixture is
1
(a) 25% (b) 20% (c) 8% (d) 10 %
2

12. In the firing range, 4 shooters are firing at their respective targets. The first, the second, the
third and the fourth shooter hits the target once every 5 s, 6 s, 7 s and 8 s, respectively. If all of
them hit their target at 10 : 00 am, when till they hit their target together again?
(a) 10 : 14 am (b) 10 : 28 am (c) 10 : 30 am (d) 10 : 31 am

13. All graduates own bikes. Does Akash, an engineer, own a bike?
1. All graduates are engineers. 2. All engineers are graduates.
Mark your answer as
(a) if
(b) if
(c) if
1
2
1
along is sufficient to answer the question
alone is sufficient to answer the question
and 2 together are sufficient to answer the question
I N
(d) if 1 and 2 together are not sufficient to answer the question

T .
14. In an election, there were only 2 candidates Nishant and Arvind. The total voters were 10000
and all the voters cast their votes, out of which 20% were declared invalid. If Nishant got 70%
of the valid votes, then how much did Arvind got?
(a) 2400 (b) 4000 (c) 3000
15. How many dots are there on the face opposite the face with two dots? O S (d) 5600

I P
(a ) 1 (b ) 5

A R (c ) 4 (d ) 6

following explanation and table.

R K
Directions (Q. Nos. 16-19) The following questions are to be answered with reference to the

Ten judges were asked to judge the relative sweetness of five compounds A, B, C, D and E by the

A
method of paired comparisons. In judging each of the possible pairs, they were required to state

S
unequivocally which of the two compounds was sweeter a judgement of equality or of no difference
was not permitted.
The results of their judgements are summarised in the given table below. In studying the table,
note that each cell entry shows the number of comparisons in which the ‘row’ compound was
judged to be sweeter than the ‘column’ compound.

A B C D E
A 5 8 10 2
B 5 3 9 6
C 2 7 7 8
D 0 1 3 4
E 8 4 2 6

16. How many comparisons did each judge make?


(a ) 5 (b) 10 (c) 15 (d) 20

17. Which compound was judged to be least sweet?


(a ) A (b ) B (c ) C (d ) D

PDF DOWNLOAD FROM : WWW.SARKARIPOST.IN


Page 84

90 Practice Set 4

18. Which of the following statements is most nearly correct?


(a) There was almost perfect agreement among the ten judges
(b) The nearest discrimination was between compounds B and C
(c) The judges were not experts in discriminating sweetness
(d) Compound D was most clearly discriminated from the other four compounds

19. Between which two compounds was the discrimination least consistent?
(a) A and D (b) B and E (c) C and E (d) A and B

Directions (Q. Nos. 20-26) Read the following two passages and answer the questions that
follow each passage. Your answers to these questions should be based on the passages only.

B Passage 1

I N
The standard methods of science proceed from observations to hypotheses to test these hypotheses in
controlled experiments. However, it would be a mistake to suppose that every hypothesis that comes out

.
of observation lends itself to rigorous scientific scrutiny. There are, infact, many questions that can be
asked of science that science is not in a position, for one reason or another, to answer. The recent debate

T
over melanoma (skin cancer) screening provides an interesting example of this area of ‘science that is not

S
scientific’ or ‘trans science’ as a few eminent thinkers have termed it. Let’s start with the observations.
There has been an increase in the number of early stage melanoma cases over the last twenty years. As a

O
result of the reported numbers, some physicians recommend screening for melanoma.

P
The ‘hypothesis’ that is implied here is that screening for melanoma will decrease the death rate from the
disease. But how do we test it? The conventional way to evaluate the effectiveness of a medical

I
technique is the double blind trial. In this case we would have to assign some people to receive screening

R
and some control people would not be screened. Then, we would look at the death rate for melanoma in
the two groups. The problems are logistic and ethical. If the answers are to reach statistical significance

A
we need very large numbers and we need to follow people over whole lifetimes neither of which is

K
practical. And how do we decide who is to receive what might be a life saving screening and who will be
denied its potential benefits?

R
20. The author would apparently agree with which of the following?
(a )
(b )
(c )
A
The effectiveness of screening for melanoma is not proven

S
Double blind trials are the best methods to evaluate
The death rate from melanoma is rising rapidly
(d ) None of the above

21. The word ‘hypothesis’ is placed in inverted commas to


(a ) suggest that the contention in the same sentence cannot be tested scientifically
(b ) emphasise the importance of framing hypotheses correctly
(c ) draw attention to the main word in the sentence
(d ) indicate that the author is using someone else’s view

22. Which of the following does the author mention as an example/examples of the ‘reasons’
mentioned in the sentence? ‘There are, in fact, many questions that can be asked of science
that science is not in a position, for one reason or another, to answer’?
1. Insufficiency of sample size 2. Ethical considerations
3. Ambiguous data
Which of the statements given above is/are correct?
(a) Only 1 (b) 1 and 2 (c) 1, 2 and 3 (d) Only 3

PDF DOWNLOAD FROM : WWW.SARKARIPOST.IN


Page 85

Stage 1 Know The Trend 91

B Passage 2
In the long run a government will always encroach upon freedom to the extent to which it has the power
to do so, this is almost a natural law of politics, since, whatever the intention of the people who exercise
political power, the sheer momentum of government leads to a constant pressure upon the liberties of
the citizen. But in many countries, society has responded by throwing up its own defenses in the shape
of social classes or organised corporations, which enjoining economic power and popular support, have
been able to set limits to the scope of action of the executive.
The fascist dictatorships of today are the first truly tyrannical governments, which Western Europe has
known for centuries and they have been rendered possible only because on coming to power they
destroyed all forms of social organisations, which were in any way rivals to the state.

23. The passage can most accurately be described as a discussion of the


(a) safeguards of individual liberty (b) functions of trade union
(c) limited power of monarchies

24. Fascist dictatorship differs from monarchies of recent times in


N
(d) ideal of liberal government

I
(a) eradicating people’s organisations
(c) exerting constant pressure on liberties

T .
(b) setting limits to their scope of action
(d) rivalling the state in power

25. The natural relationship between government and individual liberty is one of
(a) moderate complicity
(c) marked indifference
S
(b) inherent opposition

O
(d) fundamental interdependence

26. The passage suggests which of the following about fascist dictatorships?
(a )
(b )
I P
They maintain their dominance by rechanneling opposing forces in new directions
They represent a more efficient form of the executive
(c )
(d )
Their rise to power came about through an accident of history
They mark a regression to earlier despotic forms of government
A R
below.
R K
Directions (Q. Nos. 27-31) Study the following graph carefully and answer the questions given

S A
Number of employees working in various departments of two different companies
Number of employees (in hundred)

8 Company A
7 Company B
6
5
4
3
2
1
0
IT HR Marketing Finance Admin
Department

27. The number of employees working in the Marketing department of company B is


approximately what per cent of the total number of employees working in the company B?
(a) 30% (b) 28% (c) 23% (d) 32%

PDF DOWNLOAD FROM : WWW.SARKARIPOST.IN


Page 86

92 Practice Set 4

28. What is the ratio of the total number of employees working in the Admin department of both
the companies together and the total number of employees working in the Finance department
of both the companies together?
(a ) 2 : 3 (b ) 4 : 3 (c ) 3 : 2 (d ) 3 : 4

29. The number of employees working in the HR department of company A is approximately what
per cent of the number of employees working in the Finance department of company B?
(a) 44% (b) 207% (c) 53% (d) 267%

30. If the number of employees working in the IT department of company B is increased by 20%,
what would be the difference between the number of employees working in the IT department
of company B and the Admin department of company A?
(a) 560 (b) 350 (c) 700 (d) 400

(a) 600 (b) 585 (c) 620

I N
31. What is the average number of employees working in all the departments together in company A?
(d) 610

32. 4 History books, 3 Geography books and 2 Civics books are kept on a shelf randomly. The
probability that books of same type are kept together, is
(a) 1/35 (b) 1/105 (c) 209/210
T . (d) 1/210

O S
33. If the second half of the following alphabet is written in the reverse order, then which will be
the tenth letter to the left of the ninth letter coming from your right?
ABCDEFGHIJKLMNOPQRSTUVWXYZ
(a ) I (b ) C

I P (c ) J (d ) H

34. A, B, C, D, E, F and G are sitting in a line facing the East. C is immediate right of D. B is at an

(a) A and E
R
extreme end and has E as his neighbour. G is between E and F. D is sitting third from the South
end. Who are the persons sitting at the extreme ends?

A
(b) A and B (c) F and B (d) C and D

R K
35. In a feast organised in connection with a marriage, some of the participants were vegetarians,
some others were non-vegetarians. Another group was of non-vegetarians not eating meat and

Statistics?

(a) S A
yet another group of people eating meat, but not fish. Which of the following represents this

(b) (c ) (d)

36. Heavier coins are costlier. Ram’s coin is heavier than Mohan’s and costlier than Ramesh’s.
Naresh’s coin is costlier than Ram’s, but lighter than Yogesh’s. Ramesh’s coin is costlier than
Mohan’s. Who is the owner of the costliest coin?
(a) Ram (b) Ramesh (c) Yogesh (d) Naresh

37. Examine the following three figures in which the numbers follow a specific pattern.
27 42 27

4 13 16 13 11 65 8 ? 72

3 7 9

The missing number (?) in the third figure is


(a ) 9 (b) 18 (c) 12 (d ) 6

PDF DOWNLOAD FROM : WWW.SARKARIPOST.IN


Page 87

Stage 1 Know The Trend 93

1 2
38. The chance of Geeta winning a competition is and that of Seema winning it is . Find the
9 15
probability that only one Geeta and Seema wins it.
29 135 29 29
(a ) (b ) (c ) (d )
132 233 135 59

39. Examine the following statement.


˜A State Government suspended two additional District Judges.
Three assumptions numbered 1, 2 and 3 are given.
Assumptions
1. They were negligent in discharging duties.
2. There was a charge of misconduct against them.
3. The government officials were biased against them.
Which of the above assumptions is/are implicit in the statement?
(a) None is implicit
I
(b) Either 1 or 2 is implicit
N
(c) Any one of the three is implicit

T .
(d) 1 and 3 are implicit

40. The total money collected for new year celebrations in a certain building was ` 20500. The

O S
ratio of the amount contributed by the people of the A wing to that contributed by the people
of the B wing was 8 : 5. Also, the ratio of the amount contributed by the people of the B wing to
that contributed by the people of the C wing was 2 : 3. Find the amount contributed by the
people of B wing.
(a) ` 4000 (b) ` 5000

I P (c) ` 5500 (d) ` 4950

A R 1
41. A garrison had provisions for a certain number of days. After 10 days, of the men desert and
5
it is found that the provisions will now last just as long as before. How long was that?
(a) 15 days

R
42. Examine the following information. K
(b) 25 days (c) 35 days (d) 50 days

A
Monitoring has become an integral part in the planning of social development
programmes. It is recommended that Management Information System (MIS) be

S
developed for all programmes. This is likely to give a feedback on the performance of the
functionaries and the efficacy with which services are being delivered.
Two conclusions numbered, 1 and 2, are given below. You have to assume everything in the
statement to be true, then consider the two conclusions together and decide which of them
logically follows beyond a reasonable doubt from the information given in the statement.
Conclusions
1. All the social development programmes should be evaluated.
2. There is a need to monitor the performance of workers.
(a ) Only Conclusion 1 follows
(b ) Only Conclusion 2 follows
(c ) Either 1 or 2 follows
(d ) Both 1 and 2 follow

PDF DOWNLOAD FROM : WWW.SARKARIPOST.IN


Page 88

94 Practice Set 4

Directions (Q. Nos. 43-47) Read the following two passages and answer the questions that
follow each passage. Your answers to these questions should be based on the passages only.

B Passage 1
Classical music is termed ‘classical’ because it can be heard over and over again without the listener
tiring of the music. A raga can be heard many times over with the same or even heightened enjoyment
with every subsequent hearing. It is unfortunate that the Compact Disc sales of classical music are
dismal compared to other types of music. Perhaps this is because many people in our generation were
not exposed to classical music at an early age and therefore did not get to know the music.
In contrast to classical music, rock and contemporary music has a high impact on the listener, but
unfortunately is not evergreen. Its enjoyment lasts only as long as there is current interest in the topic or
emotion that the music portrays. This invariably lasts for three months or so until other music replaces

I N
it, especially when another best selling song comes out. The reason why the impact of this music is not
as great when it first comes out is thought to be because technically the intricacy of the music is not high
and not sophisticated, although many critics believe it is because the music elicits a particular emotional
feeling which gradually wears out overtime.

(a) they buy only the best selling song and music
T .
43. According to the passage, most young people do not like classical music because

(b) they
(c) they
(d) they
grow tired of classical music

O S
do not have the sophistication of a true music-lover

did not hear that type of music in their youth

piece of classical music is


I P
44. The reason the enjoyment of a particular piece of contemporary music may not last as long as a

R
(a) changes in the emotions of a person
(b) high sophistication of the classical music and its technical intricacy

A
(c) the emergence of new pieces of contemporary music that replaces the ones before

R K
(d) the desire of the youth for something novel

Passage 2
S A B
If the world is going to use wind energy to reduce greenhouse gas emissions, there are going to have to
be an awful lot more installations. The problem is that in our haste to cash in on the obvious
environmental benefits of wind power, we are largely ignoring the ecological damage that turbines can
do. Some ecologists are warning that unless we think carefully about where wind farms are sited, they
could disrupt fragile ecosystems and even contribute to global warming. Worldwide, wind energy still
accounts for little more than 0.5% of total electricity generation. It is not surprising that government
have looked at growing electricity demand and the public’s fears about global warming and seen wind
energy as part of the solution.
But there is a problem, where do you put hundreds, if not thousands, of wind turbines? Lindsay is an
advocate for renewable energy but has become concerned by the scale and number of wind farm
developments on peat bogs in Europe.
‘This is the Cinderella ecosystem’, he says. ‘Peat land is busy performing a series of important functions
for us and we just don’t see it’, Bogs often play a critical role in providing clean drinking water. More
significantly, in the context of renewable energy, they store three times as much carbon as is held, in
tropical rainforests.

PDF DOWNLOAD FROM : WWW.SARKARIPOST.IN


Page 89

Stage 1 Know The Trend 95

Mike Hal from the Cumbria Wildlife Trust in North-West England has developed a formula to give a
wind energy carbon dioxide ‘budget’ that balances the carbon dioxide savings that a project is expected
to provide against the carbon dioxide costs from the manufacture and shipping of the turbines and
construction work at the site. The major CO 2 debt incurred by a wind turbine on a peat-rich site is not
in its manufacture and installation but in the ongoing degradation of peat.
45. The key question regarding wind energy is
1. Is wind energy really eco-friendly?
2. Where do you put the hundreds or thousands of turbines?
3. Have you assessed all the pros and cons of wind energy?
4. Is wind energy more harmful than beneficial, ecologically, commercially and aesthetically?
Select the correct answer using the codes given below
(a) 1 and 2 (b) Only 2 (c) 3 and 4 (d) Only 3

46. Which of the following statements regarding bogs is not true?


1. Prior to 2005, constructing on peat bogs had posed no threat.
2. From an ecological point of view, bogs are best left alone.
I N
Select the correct answer using the codes given below.
(a) Only 1 (b) Only 2 (c) Only 3
T .
3. Bogs are delicate ecosystems that hold the largest quantity of carbon on land.

(d) None of these

1. draw our attention to the fragility of the system.


O S
47. The author uses the term ‘Cinderella ecosystem’ for wind mills to

2.
3.
4.
P
focus on the fact that wind turbines are an eyesore.

I
show that it is a fairy tale to expect returns from wind turbines.
emphasise that the peat bogs have been ignored for a long time and deserve greater attention.

(a) Only 1 (b) 2 and 3 R


Select the correct answer using the codes given below.

A (c) Only 3 (d) Only 4

follow.
R K
Directions (Q. Nos. 48-50) Examine the information given below and answer the questions that

S A
Six friends L, M, N, P, Q and R are sitting around a rectangular table facing the centre such that
two friends are seated along the each of the longer sides and one is seated along each of the shorter
sides. It is also known that
1. M is sitting diagonally opposite to L.
2. only R is sitting between M and Q.
3. P is sitting opposite to M.
48. Who is sitting opposite to N?
(a ) Q (b ) R
(c ) L (d) Cannot be determined
49. Who are the two friends sitting along one of the longer sides of the table?
(a) P and N (b) M and R (c) R and P (d) R and Q

50. Which of the following statements is definitely true?


(a) M is sitting to the immediate right of N (b) R is sitting to the immediate left of Q
(c) L is sitting between P and Q (d) None of these

PDF DOWNLOAD FROM : WWW.SARKARIPOST.IN


Page 90

96 Practice Set 4

Directions (Q. Nos. 51-55) Examine the information given below and answer the questions that
follow.
The growth of the aircraft services is driven by the increase in the number of people using aircraft
services and the increase in per person use of the airplanes. In 2010, it was expected that there
will be 200 million aircraft service users in India or about 20% of the population will generate
50 billion in aircraft services revenues. Services revenues should expand from 50 to 150 billion by
2014, while the number of users should grow to over 560 million or to about half the population of
India in the same period.
51. What is the estimated population of India in 2010?
(a) 98 crore (b) 1000 crore
(c) 100 crore (d) 115 crore

52. It is believed that, if 50% of the population of any country can afford aircraft services
use, it is economically developed. Can we say that India will be a developed country
in 2013?
(a) Yes
(c) Cannot say
(b) No

I
(d) Data inadequate
N
2010-14?
T .
53. What will be the simple average growth rate of population of India in the given period

(a) 2% (b) 3%

O S (c) 4% (d) 4.5%

54. What will be the percentage growth of the revenues of the aircraft services in the given
period 2010-14?
(a) 200% (b) 230%

I P (c) 260% (d) 300%

55. If 18 persons can build a wall 140 m long in 42 days, then the number of days that 30 persons

(a) 18

A R
will take to complete a similar wall 100 m long, is
(b) 21 (c) 24 (d) 28
56. Three statements, labelled 1, 2 and 3 are given below. You have to decide whether the data

R K
given in the statements are sufficient for answering the item. Using the data given in the
statements, you have to choose the correct alternative.
Statements

S A
1. Vidhya’s basic salary is ` 100 more than Rajni’s salary who also serves in Vidhya’s company.
2. Other allowances drawn by Rajni besides her basic salary are ` 2000 per month which is ` 50
less than that of Vidhya.
3. Rajni’s basic salary is ` 1550 per month.
What is the total monthly salary of Vidhya?
(a ) Only 2 is sufficient to answer the question
(b ) Both 2 and 3 are sufficient to answer the question
(c ) Both 1 and 2 are sufficient to answer the question
(d ) All 1, 2 and 3 are needed to answer the question
57. Examine the information given below.
˜All the research scholars are psychologists. Some psychologists are scientists.
Which of the following is a valid conclusion regarding the above argument?
(a) All the research scholars are scientists (b) Some research scholars are scientists
(c) All scientists are psychologists (d) Some psychologists are research scholars

PDF DOWNLOAD FROM : WWW.SARKARIPOST.IN


Page 91

Stage 1 Know The Trend 97

58. Select the answer figure in which the question figure is hidden/embedded.
Question figure

Answer figures

(a) (b) (c ) (d)

59. If Deepesh had walked 20 km/h faster he would have saved 1 h in the distance of 600 km, then
what is the usual speed of Deepesh?
(a) 100 km/h (b) 120 km/h (c) 150 km/h
N
(d) None of these

I
60. Kavita, Babita and Samita started a work 5 days later Samita left the work and Babita left the

work, if they take 20, 60 and 30 days individually to finish a work?


(a ) 4 (b ) 5 (c ) 6
T .
work after working 8 days. In how many more days Kavita would have completed the rest

(d ) 8

lines, then find the number of squares that can be formed.


O S
61. In the following figure, if the centres of all the circles are joined by horizontal and vertical

I P
A R
(a ) 6

R K
(b ) 7 (c ) 8 (d ) 1
62. In a row of children, Harish is eleventh from the left and Mangesh is seventeenth from the
right. When they exchange their places, Harish will be thirteenth from the left, then which of

(a) 11th

S A
the following will be the new position of Mangesh from the right?
(b) 12th (c) 19th (d) 29th
63. Amit said to Sushma, “Your father’s son is the only brother of my sister’s daughter.” How is
Sushma related to the son of Amit?
(a) Maternal grandmother (b) Cousin (c) Paternal grandmother (d) Aunt
64. If 15th February, 1933 was Tuesday, then what was the day on 19th February, 1932?
(a) Tuesday (b) Monday (c) Wednesday (d) Thursday
65. Six faces of a cube are painted with six different colours namely blue, brown, green, pink,
red and yellow. Diagrams given below show two difference positions of the painted cube.

Red Pink
Green
Green

Yellow
Blue

Which colour is on the face opposite to brown coloured face?


(a) Blue (b) Yellow (c) Green (d) Red

PDF DOWNLOAD FROM : WWW.SARKARIPOST.IN


Page 92

98 Practice Set 4

66. What is the water image of given question figure?


Question figure

C D O

Answer figures

C D O C C
(a) (b) (c ) D (d) D
C D O O

I N O

.
Directions (Q. Nos. 67-73) The following questions are based on three passages in English to
test the comprehension of English language and therefore, these questions do not have Hindi version.
Read each passage and answer the questions that follow.
T
B Passage 1 O S
I P
On 31st October, 1517, Luther nailed a long list of points for disputation—95 thesis—to the door of the
church near the castly in the Saxon capital of Wittenberg. It is a moment that has reverberated in

R
history, the dead day on which the protestant reformation was born and the middle ages suddenly
dropped. The reality is more prosaic. Some scholars have denied that the thesis were ever posted at all. It

A
seems recently likely that they were, but this was hardly a world-shattering act. Luther was now a

K
professor at the founded University of Wittenberg and the conventional method of initiating academic
debate within the theology faculty was to post thesis in advance. Because of its handy location, the door

R
of the Castle Church served as the university’s bulletin board and Luther’s gesture has been seen as no

A
more dramatic is pinning up a lecture list in a modern college.

S
67. When in the popular perception, died the middle age cease to exist?
(a ) After the birth of Luther
(b ) After Luther’s declaration of his thesis in Wittenberg
(c ) After Luther became a professor
(d ) After the foundation of the University of Wittenberg

68. Which of the following explanations reduces the dramatic effect of the nailing of Luther’s
thesis?
1. It was mere professor lecture list.
2. It was posted in a rather unknown church.
3. That the thesis were non-academic in nature.
4. Protestant reformation was not born with nailing act.
Select the correct answer using the codes given below
(a) Only 1 (b) 1 and 3 (c) Only 4 (d) 2 and 4

PDF DOWNLOAD FROM : WWW.SARKARIPOST.IN


Page 93

Stage 1 Know The Trend 99

B Passage 2
The aim of the Platonic philosophy was to exalt man into a God. The aim of the Baconian philosophy
was to provide man with what he requires while he continues to be a man. The aim of Platonic
philosophy was to raise us far above the vulgar wants. The aim of Baconian philosophy was to supply
our vulgar wants. The former aim was noble; but the latter was attainable.
Plato drew a good bow; he aimed at the stars; and therefore, though there was no want of strength or
skill, the shot was thrown away. His arrow was indeed followed by a track of dazzling radiance; but it
struck nothing. Bacon fixed his eye on a mark, which was placed on Earth and hit it in the white. The
philosophy of Plato began with words and ended in words, noble words indeed, words such as were to be
expected from the finest of human intellects exercising boundless dominion over the fittest of languages.

69. The above passage presents Platonic philosophy as


1. giving rise to vulgar wants.
2. too idealistic in terms of a realistic assessment of a man.
Select the correct answer using the codes given below
I N
(a) Only 1 (b) Only 2 (c) Both 1 and 2

T .
70. Which one of the following best reflects the underlying tone of the passage?
(d) None of these

1. All ideas regarding man are couched in noble words.


2. Man when exalted into a God comes to nothing.

O S
3. It is the image of man conceived differently that makes the basic distinction between different
systems.

(a) Only 1 (b) Only 2


I P
Select the correct answer using the codes given below
(c) 2 and 3 (d) Only 3

B
A R
Passage 3

R K
Intellectual property law firms are playing a major role in the global IP protection and enforcement.
Many individuals and companies are utilising the services of Indian law firms and lawyers to protect

S A
their respective IP rights. However, techno-legal issues have complicated the traditional IP rights
management. These days information technology is increasingly being used world over that is both
facilitating and infringing the IP right of others. Thus, role of traditional IP law firms has significantly
changed. These days an IP rights can be protected only if the law firm or lawyer is/are good at both IP
and technical aspects. Indian IP law firms and lawyers must adopt the technological revolution that is
changing the entire service sectors world over. As more and more IT is being used to protect and enforce
IP rights at global level, ignoring IT issues is not a wise option.
International treaties and agreements have further extended the otherwise territorial nature of IP
protection and rights. For instance, with the adoption of Madrid Agreement and Madrid Protocol by
India, trademarks protection in India would be given a new meaning. The IP law firms and lawyers of
India must change gears now as even the Indian Government has adopted technological methods to file
and pursue IP rights in India. E-filing of various IP applications alongwith their online dealing is
gradually becoming popular in India.

71. Consider the following statements


1. Intellectual property rights can be protected, if the lawyers are technically sound.
2. Use of IT services world wide, has decreased the role of traditional intellectual property law
firms.

PDF DOWNLOAD FROM : WWW.SARKARIPOST.IN


Page 94

100 Practice Set 4

3. Detailed knowledge of intellectual property laws, is the only way, to protect the interest in case
of infringement of the concerned rights or interest.
4. India adopted Madrid Agreement and Madrid Protocol, for the protection of trademarks.
Which of the statements given above is/are correct?
(a) Only 4 (b) 2 and 4 (c) 3 and 4 (d) All of these

72. Which one of the following, is the best ‘title’ for the given passage?
(a ) Cyber law firms in India
(b ) Intellectual property law firms in India
(c ) Role of information technology in legal sphere
(d ) Role of international agreements in protecting IP rights

73. Consider the following statements


1. Intellectual property law firms, should concentrate on the core law, in order to succeed in any
litigation, resulting from the infringement of any interest.
N
2. Indian Government is implementing new methods, which are technologically sound to
safeguard the intellectual property rights.
I
.
3. Though, Information Technology has influenced each and every sphere of our lives, but there
is no difference between the traditional and contemporary IP rights services.
Which of the statements given above is\are correct?
T
(a) Only 3

74. Examine the situation given below


(b) Only 2

O S (c) 1 and 3 (d) 1, 2 and 3

P
You are the team leader of a team. A new assignment is given to your team, which is to be
completed in two months. Although you have great experience, your team does not have
much experience.
I
you will do.

A R
Arrange the following in the order of the most effective to least effective steps in terms of what

1. Request higher authorities for change of team members.

R K
2. Arrange training in skills necessary for the assignment.
3. Do all the work on your own.

Codes

S A
4. Assign work to the people based on their strengths and weaknesses.

(a) 2, 4, 1, 3 (b) 2, 4, 3, 1 (c) 4, 2, 1, 3 (d) 4, 2, 3, 1

75. Three statements, labelled 1, 2 and 3, are given below. You have to decide whether the data
given in the statements are sufficient for answering the item. Using the data given in the
statements, you have to choose the correct alternative.
Statements
1. Pointing to the photograph, Prem said, ‘‘She is the mother of my father’s only granddaughter’’.
2. Prem has no siblings.
3. Pointing to the photograph, Prem said, ‘‘She is the only daughter-in-law of my mother.’’
How is the girl in the photograph related to Prem?
(a ) Any two of the three are sufficient to answer the question
(b ) Both 1 and 2 are sufficient to answer the question
(c ) Both 2 and 3 are sufficient to answer the question
(d ) Either only 3 or both 1 and 2 are sufficient to answer the question

PDF DOWNLOAD FROM : WWW.SARKARIPOST.IN


Page 95

Stage 1 Know The Trend 101

Directions (Q. Nos. 76-80) Given below are five questions. Each question describes a situation
and is followed by four possible responses. Indicate the response you find most appropriate. Choose
only one response for each question. The responses will be evaluated based on the level of
appropriateness for the given situation. Please attempt all the questions. There is no penalty for wrong
answers for these five questions.

76. You are the executive director of an upcoming Infotech company which is making a name for
itself in the market. In a short period of time, Mr A has helped in doubling the revenues as well
as creating a high brand equity for the company so much so, that you are thinking of
promoting him. However, you have been receiving information from many corners about his
attitude towards the female colleagues, particularly his habit of making loose comments on
women. What would you do?
(a) Ask him his resignation
(b) Transfer him to another branch office
(c) You yourself go for fact finding
(d) Let it go as he is a valuable worker in your organisation

I N
.
77. You have been posted as a District Magistrate in a district where construction work is going on
a large scale. One day suddenly you receive a call from someone that many labourers have

T
been injured in an accident. When you reach there, you find a huge crowd of labourers, their
relatives and media persons. You will
S
(a) hold a meeting with experts and direct the rescue team for further action
(b) tell the crowd to vacate the place
O
(c) call the owner of the construction project

P
(d) as certain how many labourers were on duty and how many were injured to take any further step

I
78. You led a team, which has recently completed a very important task, assigned by the

A R
government. A felicitation programme was organised by the Mayor of your city. In the
programme, he was showering accolade on you without mentioning about your team. What
would you do?

R K
(a) Thanked him and then said that you did not work alone and it was a team effort
(b) Took the praise and then told your team that you did not know how to explain in front of the
Mayor that the work was a team effort

S A
(c) Accept the accolades and try to steal the spotlight
(d) Keep mum and do nothing

79. You are working as a Sales Manager in Indian Railways and your senior is trying to make you
understand some difficult, but important project. You are not able to understand some parts of
it. What should be your reaction?
(a ) Listen silently without showing any interest
(b ) Remain aloof from all the discussion
(c ) Try to excuse yourself and ask your senior to explain it to you again
(d ) Make others feel that you have understood everything

80. You are working with Delhi Police. A task has been assigned to one of your co-workers, but
you believe that you can do it in a better way because of your knowledge and prior experience
in that case. What would you do?
(a) Inform your senior to assign this task to you, explaining him that you can better deal with the
case
(b) Work with your co-worker in parallel and submit to your senior for his approval
(c) Show your feelings to your co-worker in achieving the task with cooperation
(d) Concentrate on your work

PDF DOWNLOAD FROM : WWW.SARKARIPOST.IN


Page 96

Space for Rough Work

I N
T .
O S
I P
A R
R K
S A

PDF DOWNLOAD FROM : WWW.SARKARIPOST.IN


Page 97

Stage 1 Know The Trend

Answer with Explanations


1. (c) Nearly 31 per cent of India’s current population lives in Statement 1 is wrong because history of ocean structures
urban areas and contributes 63 per cent of India’s GDP. can be found by sediment core. Statements 3 and 4 are
With increasing urbanisation there is a need of sustainable wrong because these are not talked about in the passage.
and comprehensive development of cities. It is expected 9. (d) It explains how cool water moves towards the equator and
that Smart Cities Mission in India will definitely boost up the warm water at equator moves towards the poles (ocean
employment and enhance income for all sections of currents). Statement 3 is chosen because the whole
people leading to a better quality of life. paragraph explains how the ocean currents are formed.
2. (c) In India, playing on the tunes of what media plays, Indians Statement 1 is wrong because the lines give no explanation
are not very rational in approach. What was required was about the formation of sea ice. Statement 2 is wrong
severe sentence for what Juvenile offender did in because melting of sea ice causes the temperature to rise.
Nirbhaya’s case. Lowering of age for severe punishment 10. (d) Option (d) is chosen because melting of sea ice does not
for Juveniles is not a guarantee that long-term interests will
be taken care of the JJA is unlikely to stand the test of
time.
3. (a) Introduction of payment banks is going to help a large
I N
increase the density of the Arctic sea. Also, nothing about
it is mentioned in the passage. Options (a), (b) and (c) are
the possible effects when the sea ice melts.

number of people from lower strata like migrant labours,


low income household, small businesses and a large
number of people belonging to unorganised sectors. It is
.
11. (b) Let CP of 1 L milk be ` 1.
Then, SP of 1 L mixture = ` 1

T
CP of 1 L mixture =
100
125
×1= `
4
5
really a welcome step and will certainly help government’s
decision to wider financial inclusion.
4. (d) The point of this paragraph is to illustrate that Archaeology
O SBy using Alligation rule,

CP of 1 L of CP of 1 L

P
is like a physical science in that important factual pure milk of water
discoveries lead to theoretical changes. The quotation `1 0

one. So, the correct answer is option (d).

R
5. (d) The passage states that the discovery of the hyoid bone I
provides evidence that this discovery is in fact a significant

4– = —
— 4
4
`—
5
4=—
1– — 1

K A
has had a tremendous impact on theories about the
origins of speech. The passage goes on to say that it is
the first hyoid found in such an early human related
species, suggesting that the timeline of human verbal
5 0 5

∴ Milk : Water =
4 1
: = 4:1
5 5
5 5

R
development would be changed by the discovery. Thus, it
can be inferred that the discovery made the

A
re-examination of prior theories necessary. So, the correct
Hence, percentage of water in the mixture
1
= × 100 = 20%
5
answer is option (d).

S
6. (c) The option (c) correctly describes a discovery that causes
a re-examination of earlier ideas in another field. In this
case, newly uncovered journal entries by a politician spur
a re-evaluation of certain historical ideas regarding an
12. (a) LCM of 5, 6, 7, 8
2 5, 6, 7, 8
5, 3, 7, 4

= 2 × 5 × 3 × 7 × 4 = 840 s
important conflict. So, the correct answer is option (c). 840
Again, they hit target at = 10 : 00 am + min
7. (a) Deposits on a sea floor help Mr Gregery to study ocean 60
history. Statement 1 is chosen because sediment cores = 10 : 00 am + 14 min =10 :14 am
are essentially a record of sediments that settled at the
sea floor, layer by layer and they record the conditions of
13. (b) Statement 1 alone is not sufficient to as we cannot
conclude that Akash is a graduate. From Statement 2, we
the ocean system during the time they settled.
can say that Akash is a graduate. Thus, Akash owns a
Statement 2 is wrong. A bio-chemical marker is the study
bike.
of the chemical process that occurs within living things.
Hence, Statement 2 alone is sufficient answer the question
8. (b) Existence of the marker in the sediment reveals the
existence of the frozen land. Statement 2 is correct since it 14. (a) Valid votes = 80% of 10000 = 8000
agrees with author’s view. Author says that scientists can Nishant got = 70% of valid votes
search for biochemical marker that is tied to certain When Arvind got 30% of the valid votes, then
species of algae that live only in ice. Therefore, this marker Number of votes in favour of Arvind
is used to find the existence of ice at that time. = 30% of 8000 = 2400

PDF DOWNLOAD FROM : WWW.SARKARIPOST.IN


Page 98

106 Practice Set 4

15. (d) There are 2, 3, 4 and 6 dots respectively on the adjacent 28. (d) Number of employees in Admin department of both
faces of the face having five dots. Therefore, one dot will companies = 6
be opposite the face having five dots. There are 1, 3 and Number of employees in Finance department of both
5 dots respectively on the adjacent faces of the face companies = 8
having two dots. Therefore, either 4 or 6 dots will be on 6
∴ Required ratio = = 3 : 4
the face opposite of the face having two dots. 8
Now, 4 and 6 dots cannot be on the opposite faces. 29. (d) Number of employees working in HR department of
Clearly, 6 dots are there on the face opposite to the company A = 8
face with two dots. Number of employees working in Finance department of
16. (b) Each compound is compared with all the other, giving company B = 3
20 comparisons or results but since each comparison has 8
∴ Required percentage = × 100 ≈ 267%
20 3
been counted twice, we have = 10 comparisons.
2
30. (a) Increased number of employees working in IT department
of compnay B =  8 + 8 ×
17. (b) B was judged sweeter only 6 times. 20 
 × 100 = 9.6 × 100 = 960
 100 
18. (d) The other compounds were judged sweeter 25, 23, 24
and 20 times, D was judged sweeter only 8 times.
19. (d) Five judges called A sweeter and five judges called B
I N
From the given graph, number of employees working in
Admin department of company A = 400
∴Required difference = 960 − 400 = 560
sweeter.
20. (a) The author would apparently agree with the effectiveness
of screening for melanoma is not proven. .
31. (d) Average number of employees working in all departments

T
together in company A
= 
5.5 + 8 + 8.5 + 4.5 + 4   30.5  × 100 = 610
21. (a) The word ‘hypothesis’ means the contention in the same
sentence cannot be tested scientifically.
22. (b) According to the author, the meaning of sentence which
O S  5
32. (d) Here, total books = 4 + 3 + 2 = 9
 × 100 = 
  5 

is mentioned in the question is insufficiency of sample


size and ethical considerations.
23. (a) The author says that all forms of government tend to
I P Total number of ways of keeping the books on the
shelf = 9!
Number of ways in which the books can be kept together
= (4! 3! 2!) 3!

their individual liberties.


A R
become some what dictatorial. He shows how society
protects itself from this tendency. Throughout, he
demonstrates how people tend to protect or safeguard
∴Required probability = (4! 3! 2!) 3! / 9! = 1 / 210
33. (d) Rewriting the second half of English alphabets in reverse

K
order
24. (a) The last sentence says that the fascist dictatorship ABCDEFGHIJKLMZYXWVUTSRQPON

any way rivals to the state.

A R
destroyed all forms of social organisations, which were in

25. (b) The author says that the tendency for a government to
∴Ninth letter from right end = V
Hence, 10th letter to the left of V = H
34. (b) Sitting arrangement of A, B, C, D, E, F and G are as follows:

S
encroach upon individual liberty to the extent, to which it
has the power to do so, is ‘almost a natural law of
politics’. Thus, government and individual liberty are
inherently opposite to one another.
B
E
G
F W
N

E
26. (d) If the fascist dictatorships ‘are the first truly tyrannical D
governments, which Western Europe has known for C
A S
centuries’, then it can be inferred that centuries ago there
were tyrannical or despotic governments in Western Hence, A and B are sitting at the extreme ends.
Europe. Thus, the fascist governments represent a
35. (b)
regression to an earlier form of government.
Non-vegetarian
Solutions (Q. Nos. 27-31)
Meat Fish
We are not taking population multiplied by 100 wherever eating eating
not necessary.
27. (b) Number of employees in the Marketing department of Vegetarian
company B = 7
Total number of employees in company B = 25
7
∴ Required percentage = × 100 = 28%
25

PDF DOWNLOAD FROM : WWW.SARKARIPOST.IN


Page 99

Stage 1 Know The Trend 107

36. (c) According to the question, 43. (d) It is clearly mentioned in the passage that many people in
Ram > Mohan ...(i) our generation were not exposed to classical music at an
Ram > Ramesh ...(ii) early age and therefore, did not get to know the music.

Yogesh > Naresh > Ram ...(iii) 44. (c) In the second para of the passage, it is mentioned that
Ramesh > Mohan ...(iv) contemporary music has a high impact on the listener but
unfortunately is not evergreen. Its enjoyment lasts only as
From Eqs. (i),(ii),(iii) and (iv), we get
long as there is current interest in the topic or emotion that
Yogesh > Naresh > Ram> Ramesh >Mohan the music portrays. This invariably lasts for three months
Therefore, owner of the costliest coin is Yogesh. or so until other music replaces it, especially when another
27 16 best selling song comes out.
37. (c) From first figure, = 9 and =4
3 4
45. (b) The author does not question whether wind energy is
⇒ 9 + 4 = 13 (middle number) eco-friendly. He only points to the problem in the selection
42 65
From second figure, = 6 and =5 of place for setting up a wind farm. In the passage, he
7 13 focuses on the problem, where do you put hundreds, if not
⇒ 6 + 5 = 11 (middle number) thousands, of wind turbines.
From third figure,


27
9
= 3 and

3 + 9 = 12
72
8
=9

(middle number)
had posed non threat.

I N
46. (a) It is not true that prior to 2005 constructing on peat bogs

47. (d) The word ‘Cinderella’ (from the fairy tale) means
∴ ? = 12
38. (c) Probability that only one of Seema or Geeta wins the
competition = P (Seema wins and Geeta does not win) or P
T .
something that has been ignored for a long-time and
deserves to receive more attention.

Solutions (Q. Nos. 48-50)


(Geeta wins and Seema does not win)
= P (S ) × P′(G ) + P (G ) × P′(S )
=
2 8 1 13 16
× + × = +
13
=
29

O S
The two possible sitting arrangements as per the given
information be depicted as shown in the diagrams below.
L P P L
15 9 9 15 135 135 135
39. (b) A person holding an office is generally suspended on
charges of misconduct or negligence of duty. So, either
1 or 2 is implicit. Assumption 3 seems to be vague and so
I P Q N N Q

it is not implicit.

A R
40. (b) Since, the amount collected by wing B is common to both
the ratios, it is used to compare the collections of all
R M
48. (a) Q is sitting opposite to N.
M R

3 wings. So, find the LCM of 5 and 2.


∴LCM of 5 and 2 = 10

R K
∴Ratio of the amounts contributed by the people of all the
49. (b) M and R are the two friends sitting alone one of the longer
sides of the table.
50. (c) L is sitting between P and Q is definitely true.

S A
three wings = 16 : 10 : 15
Let the amount contributed by each wing be 16x, 10x and
15x respectively.
Then, 16x + 10x + 15x = 20500 ⇒ 41x = 20500
51. (c) As 200 million = 20% of population
∴Required population = 200 ×
100
20
= 100 crore
= 1000 million

∴ x = 500 52. (d) By 2014, half or 50% of the Indian population will use
Amount contributed by wing B = 10x = 10 × 500 = ` 5000 aircraft services. As we do not have any information about
2013, so the data is inadequate.
41. (d) Let initially then be x men having food for y days after
10 days, x men had food for ( y − 10) days. Also,  x − 
x 53. (b) Population in 2010 = 1000 million
 5 Population in 2014 after 4 yr = 560 × 2 = 1120 million
men had food for y days. [as half of population (= 560) million]
∴ x( y − 10) =
4x
× y ⇒ 5xy − 50x = 4xy (1120 − 1000)
∴Average (simple) growth rate = × 100
5 1000 × 4
⇒ xy − 50x = 0 120 12
= × 100 = = 3% per annum
⇒ x( y − 50) = 0 1000 × 4 4
⇒ y = 50 (as x ≠ 0 )
54. (a) Total revenue in 2010 = 50 billion
42. (d) According to the statement, monitoring and evaluation of Total revenue in 2014 = 150 billion
social development programmes their function, (150 − 50)
performance and efficiency is absolutely essential. So, ∴Growth in percentage = × 100 = 200%
50
both 1 and 2 follow.

PDF DOWNLOAD FROM : WWW.SARKARIPOST.IN


Page 100

108 Practice Set 4

55. (a) Here, M1 = 18, M 2 = 30 and D1 = 42 Work completed by all the three before Samita left
6× 5 1
Let D2 = x, W1 = 140 and W2 = 100 = =
60 2
By using the formula, 1 1
M1 D1 M 2 D2 Remaining work = 1 − =
= 2 2
W1 W2 So, the remaining work is done by Kavita and Babita.
18 × 42 30 × x One day’s work of Kavita and Babita
⇒ =
140 100 1 1 3+ 1 4
= + = =
⇒ 140 × 30 × x = 100 × 18 × 42 20 60 60 60
100 × 18 × 42 4 × 3 12 1
∴ x= = 18 3 day’s work of Babita and Kavita = = =
140 × 30 60 60 5
1 1 3
Hence, the required number of days are 18 . Remaining work to be done by Kavita = − =
2 5 10
56. (d) From Statement 3, Rajni’s basic salary = ` 1550 Remaining work
From Statement 1 and 3 Vidhya’s basic salary Number of days required =
One day′ s work of Kavita
= ` (1550 + 100) = ` 1650
From statement 2, Rajni’s other allowances = ` 2000
and Vidhya’s other allowances = ` 2050
∴ Vidhya’s monthly salary = (1650 + 2050) = ` 3700 61. (c)
I N
=
3 / 10
1 / 20
=3×2=6

So, all three statements are needed to answer the


question.
57. (d) The above arguments can be represented as
T . D

E
J

K
C

Scientists Scientists

O S F L H
Psychologists

Or

Psychologists
Research scholars
Research
scholars
I P A

Number of squares formed


G
B

= AFLG, GBHL, LHIK, FLKE, KICJ, KEJD, AEIB, FHCD


From the above diagram only option (d) i.e. some
psychologists are research scholars follows.
58. (b) It is clear from the given option that, figure (b) has the
A R So, total of 8 squares will be formed.
62. (c) According to the question,
Harish Mangesh
above figure embedded in it.

R K Left
11th 17th

Right

S
59. (a) Let the speed of Deepesh be x km/h.
Distance = 600 km
A (given)
On exchanging their places, we get
Mangesh Harish
13th

According to the question, Left Right

= 1 ⇒ 600  −
600 600 1 1 
− =1 Q Total number of children = L + R − 1= 13 + 17 − 1= 29
x x + 20  x x + 20 
Mangesh’s position from right = ( 29 − 11 + 1) =19th
⇒ 600 × 20 = x (x + 20)
⇒ x 2 + 20x − 12000 = 0
63. (b) It is obvious from the given information that Sushma is
daughter of Amit’s sister. Hence, son of Amit will be cousin
⇒ (x − 100)(x + 120) = 0 of Sushma.
⇒ x = 100, − 120
64. (d) Number of odd days in the year 1932 (a leap year) = 2
So, x = 100 or x = −120 (not possible)
The day on 15th February, 1932 must have been 2 week
Hence, the speed of Deepesh is 100 km/h. days before the day on 15th February, 1933 i.e. Sunday.
60. (c) Work done by Kavita, Babita and Samita in one day Hence, the day on 19th February, 1932 was Thursday.
1 1 1 65. (c) It can be inferred from the diagrams, the four adjacent faces
= + +
20 60 30 to green face are red, blue, yellow and pink.
3 + 1+ 2 6 Hence, brown coloured face will be opposite to the green
= =
60 60 coloured face of the cube.

PDF DOWNLOAD FROM : WWW.SARKARIPOST.IN


Page 101

Stage 1 Know The Trend 109

66. (b) The water image of given question figure will be formed as As team members do not have much experience, so
arranging training sessions for them can help them in
gaining skills and these skills can be utilised in the
project. So, Statement 2 is the second best response.
C D O As the project is to be completed on time, so requesting
for team members, who are capable of doing the
C D O
project, is the third best response. The work is to be
completed by a team and as an individual you may not
be able to complete the work within the deadline. So,
Statement 3 is the worst response. Hence, the order is
67. (b) From the lines “on 31st October, 1517, Luther nailed a long 4-2-1-3.
list of points for disputation-95- thesis-to the door of the 75. (d) From Statement 1, we conclude that the girl is either
church near the castle in the Saxon capital of Wittenberg. It is Prem’s or his brother’s wife. But, according to the
the moment that has reverberated in history, the dead day on Statement 2, Prem has no siblings.
which the protestant reformation was born and the middle So, from both 1 and 2, we conclude that the girl is Prem’s
ages suddenly dropped”. It can said that the middle age
cease to exist after Luther declaration of his thesis in
Wittenberg. So, option (b) is valid while option (a), (c) and (d)
are irrelevant.
wife.

I N
wife. From Statement 3, we find that the girl is the only
daughter-in-law of Prem’s mother, so she is Prem’s

76. (c) Without knowing overall situation, you should not ask
68. (a) From the line “Luther’s gesture has been seen as no more
dramatic as pinning up a lecture list in a modern college.” It
can be said that lists of points of disputation (95 thesis) were
T .
his resignation. He is valuable worker in you
organisation and there may be some conflict of interest
with other employees there. So, it’s better to go for fact
nothing, but mere professor’s lecture list, which only reduces
the dramatic effect of the nailing of Luthers thesis. Hence,
Statement 1 is only authentic. Statements 2, 3 and 4 are
irrelevant from the point of view of the question. Hence, the
O S finding by yourself and then take the decision
considering his contribution to the upcoming company.
Simultaneously, if he is found to guilty as alleged, you
should take stern action. So, option (c) is right here.
correct answer is option (a).

I
69. (b) The Platonic philosophy is too idealistic in terms of a realistic
assessment of a man. The aim of the Platonic philosophy
P 77. (a) In an emergency situation, the right way to handle the
situation is to start the rescue work as soon as possible,
so as to save maximum persons.
was to exalt man to the status of a God.

A R
70. (d) The underlying tone of the passage is that it is the image of
man conceived differently that makes the basic distinction
78. (a) Option (a) exhibits the integrity and team spirit of the
person. A team leader should acknowledge and
appreciate the hard work and inputs provided by the

philosophers at different times.

R K
between different systems as mentioned by the distinct

71. (d) Statements 2 and 4 are correct, in 2nd paragraph, 3rd line
team members. If you do not let the other members
share praise with you, your professional relations with
your subordinates will turn bitter.

S A
justifies Statement 2, as it states that the role of traditional IP
law firms has changed, here the sense is that role of
traditional IP law firms has decreased due to increase in use
of Information Technology.
79. (c) Option (c) shows your courage to ask questions about
something you have not understood. It also shows your
desire to learn difficult and interesting things. Also, this
will help you in doing your work effectively. Options (a)
72. (b) The most appropriate title for this passage is “Intellectual and (b) show your indifferent attitude towards the work.
Property Law Firms in India”, the first line of the passage itself Option (d) shows that you are shying away from
supports it. Although, all the other options are also correct, learning concepts important to the pursuit of your
but they are subsidiary functions of intellectual property rights career.
law firms. 80. (c) Your senior has assigned the task to your co-worker as,
73. (b) Statement 2 is correct, as first 2 lines of the 7th paragraph he would have felt that your co-worker might be the
justifies this. Statement 1 is wrong/incorrect because right person to do the task. Hence, you cannot inform
according to 3rd paragraph, the intellectual property rights your senior to assign the task to you. Also it is your
can be protected only if the law firm or lawyer is/are good at feeling that you can do the task in a better way, but your
both IP and technical aspects. Statement 3rd is incorrect, senior may think otherwise.
because according to 3rd paragraph, there is a clear So, option (a) can be eliminated. Option (b) is not right
distinction between traditional and contemporary intellectual because working on the task without prior informing
property rights. your senior is not just. Option (d) is also not right
74. (c) As each and every one in the team has his/her own because if you have prior experience, then you should
strengths, so identifying them and assigning work on this at least try to help your co-worker in achieving the task
basis is the most effective response. with cooperation. So, option (c) is the right answer.09

PDF DOWNLOAD FROM : WWW.SARKARIPOST.IN


Page 102

Space for Rough Work

I N
T .
O S
I P
A R
R K
S A

PDF DOWNLOAD FROM : WWW.SARKARIPOST.IN


Page 103

CSAT
C i v i l S e r v i c e s A p t i t u d e Te s t
Paper 2

Practice Set 5
I N
Time : 2 hrs
T . MM : 200

Instructions
1. There are 80 questions in this paper.
O S
There is only one answer to be selected by you.

I P
2. The answer of any question you are thinking that more than two answers are true, then you must choose the nearest one.

WEAKER AREAS
Penalty for wrong answer

deducted in your total marks.

A R
3. There are four alternative answers in every question. When you select a wrong answer, then 1/3rd mark of that question is

4. If any candidate gives more than one answer and one of them is true but it is treated as a wrong answer and the candidate

R K
is penalised for that and1/3rd marks will be deducted.

Directions (Q. Nos. 1-7) Read the following four passages and answer the questions that

A
follow. Your answers to these questions should be based on the passage only.

S B Passage 1
YOUR

India’s approach to climate change has shifted dramatically in the span of a few years. India has
developed a comprehensive climate change program domestically and has adopted a new stance in the
international negotiations that has earned it the reputation of being a ‘deal maker’.
IMPROVE

The authors argues that this shift in India’s climate change strategy can be understood if seen in the
context of India's economic and development aspirations and the changes occurring in the larger
geopolitical landscape: India's desire to play a strategically important role in a new global order as well
as deal with domestically critical issues like energy security and energy access, all coalesce with the
climate issue. By engaging proactively on climate change, India may be able to advance all of these
objectives at once. However, to succeed, it must demonstrate that action on climate change does not
come at the expense of economic growth or development goals, and that these can go hand in hand.
Stage 2

PDF DOWNLOAD FROM : WWW.SARKARIPOST.IN


Page 104

112 Practice Set 5

1. Which among the following is the critical and the most rational inference that can be made
from the above passage?
(a) India has been termed as ‘deal maker’ as far as International negotiations on climate change are
concerned.
(b) India’s new stand on climate change is based on India’s desire to play a strategically important
role in a new global order as well as deal efficiently with domestically critical issues.
(c) India is serious when it comes to tackling problems arising out of climate change negotiations.
(d) Economic growth and development issues are left behind while dealing with the problem of
climate change.

B Passage 2
Public spaces in India’s cities are often eyesores, full of rotting piles of trash along the streets, in

I N
neighbourhoods, public parks and play-grounds and outside fancy air-conditioned hall and five star
hotels. Trash and litter is everywhere and it does not bother any one in the world, all around the globe,
in taking a note of India’s economic growth but unfortunately Indians have moved very sluggishly in the
field of environmental cleanliness and taking care of the surroundings. You can easily find a well dressed

T .
and educated person trashing and empty coke bottle in the middle of the street. We need to change this
attitude and make litting a socially unacceptable aspect. Having waste and garbage scattered around
should be a matter of concern for everyone.

the above passage?


O S
2. Which among the following is the most logical and rational assumption that can be made from

P
(a) Our surroundings can be absolutely neat and clean if local Government bodies have adequate
staff with required skills and competencies.

I
(b) Indians do not take any note of their environmental cleanliness. For them trash lying scattered

R
on the roads does not bother.
(c) If everyone gets conscious of cleanliness and takes care of throwing garbage at public places, this

A
problem can be tackled.

R K
(d) Strict laws and fines should be imposed on those who make our surrounding unhealthy and dirty.

B Passage 3

S A
Tourism in India has a strong relevance to economic development, cultural growth and national
integration. As mentioned earlier, India is a vast country of great beauty and diversity and her tourists
potential is equally vast. With her rich cultural heritage as superbly manifest in many of the
architectural wonders (palaces, temples, mosques, forts, etc), caves and prehistoric wall paintings, her
widely varied topography ranging from the monotonous plains to the loftiest mountains of the world,
her large climatic variations ranging from some of the wettest and the driest as well as from the hottest
and the coldest parts of the world, beautiful long beaches on the sea coast, vast stretches of sands,
gregarious tropical forests and above all, the great variety of the life-style, India offers an unending
choice for the tourists.
3. Which among the following is the most logical and rational inference that can be made from
the above passage?
(a) India has a great potential for tourism industry which has a great relevance to economic
development, cultural growth and national integration.
(b) India’s tourists potential is very vast with unparallel beautiful landscapes and diverse cultural
heritage.
(c) Under the present scenario, threat of terrorist’s attacks, regional clashes and corruption at large,
India can not become a tourist destination.
(d) India’s foreign exchange reserve has gone up due to rise in number of tourists.

PDF DOWNLOAD FROM : WWW.SARKARIPOST.IN


Page 105

Stage 2 Improve your Weaker Areas 113

B Passage 4
Some economists believe that the United States can be utilised as a ‘land-bridge’ for the shipment of
containerised cargo between Europe and the ‘Far East’. Under this concept, containerised freight
travelling between Europe and the ‘Far East’ would shipped by ocean carrier to the United States East
Coast unloaded and placed on special railway flat cars and shipped via railroad to a West Coast port. At
this port, the containers would then be loaded on ships bound to a ‘Far East’ port of entry. Thus, a land
transportation system would be substituted for marine transportation during part of the movement of
goods between Europe and the ‘Far East’.
If a land-bridge system of shipment were deemed feasible and competitive with alternative methods, it
would open a completely new market for both United States steamship lines and railroads. At present,
foreign lines carry all ‘Far East’-Europe freight. American carriers get none of this trade and all water
routes exclude the railroads.

N
Thus, in addition to the land bridge getting this new business for the railroads, it also gives the United
States East Coast Ships an opportunity to compete for this trade. The land-bridge concept has the

I
potential of offering new job openings for United States railway workers and seamen. In addition, there

.
would be expansion of labour requirements for people in the ship building and container manufacturing
business. Such a system also has the potential of relieving the United States of a part of the burden it

T
now bears in the form of subsidies to the ship industry. By making United States rail transportation an

O S
export service, the land bridge system would have a favourable effect on our balance of payment.
4. According to the passage, the land transportation system would be replaced by
(a )
(b )
(c )
air transportation system
water transportation system
freight container services
I P
(d ) None of these

A R
5. According to the passage, if a land bridge system were feasible, it would
1. increase foreign trade.

R K
2. decrease the amount of air freight.
3. create a new market for steamship lines and railroad.

(a) 1 and 2

S A
Select the correct answer using the codes given below
(b) 1 and 3 (c) Only 3 (d) All of these
6. The author implies that which of the following would be provided employment by the
development of a land bridge?
1. Dock workers.
2. US sailors.
3. US railway workers.
Select the correct answer using the codes given below
(a) Only 1 (b) Only 2 (c) 2 and 3 (d) All of these
7. The passage states that a land bridge would improve United States
(a ) balance of payment
(b ) international relations
(c ) railroad industry
(d ) foreign trade

PDF DOWNLOAD FROM : WWW.SARKARIPOST.IN


Page 106

114 Practice Set 5

Directions (Q. Nos. 8-10) Examine the pie chart and graph carefully and answer the questions
that follow.
Portion of wage earners engaged in Yearly income of the city’s
various occupations in a city manufacturing company

Professional 60
50

Income ( ` in crore)
50
20°
40
Personal
services 120° Manufacturing 30 30
100° 30
20
20
10 12
90° 30°
0
Commerce Transportation 1995 2000 2005 2010 2015

8. What was the average annual income of the company from 2000 to 2005?
I N
Years

(a) ` 15 crore (b) ` 16 crore


9. If 1980 workers were engaged in Commerce, then how many were engaged in Manufacturing?
(a) 1485 (b) 1782 (c) 2200
T .
(c) ` 25 crore (d) ` 30 crore

(d) 2640

O S
10. The average income of Professional workers was 50% greater than that of the Transportation
workers. If the total income of the Transportation workers was ` 23760000, then what was the
total income of the Professional workers?
(a) ` 10560000 (b) ` 23760000

I P (c) ` 31680000
11. Pipes P and Q can fill a tank in 36 min and 90 min, respectively. Pipe R can empty it at 3 L / min.
(d) ` 35640000

(a) 150
R
All pipes were opened simultaneously. They filled the tank in 45 min. Find its capacity (in L).

A
(b) 270 (c) 225 (d) 180

R K
12. Amit goes from city A to city B by car at 60 km/h and after that his car had some problem. So,
he left his car at his friend’s home in city B and came back by bus at 40 km/h. If the whole

S A
journey took 5 h, then the distance between cities A and B (in km) is
(a) 120 (b) 150 (c) 90
13. Five bells toll at regular intervals of 10, 15, 20, 25 and 30 s, respectively. If they toll together at
(d) 80

8:00 am. Then, at what time will they toll together for the first after 8:00 am?
(a) 8:04 pm (b) 8:08 am (c) 8:05 am (d) 8:07 am
14. Sonu and Ramu can do a job in 60 and 40 days, respectively. They earned ` 450 for completing
work. What is the share of Sonu?
(a) ` 270 (b) ` 225 (c) ` 180 (d) ` 250
15. A water tank has the dimensions of 22 m × 10 m × 7 m. A pipe of radius 7 cm is connected to it.
If water is flowing through the pipe at the rate of 100 m/min, then the tank will be completely
filled by the pipe in
(a) 500 min (b) 1000 min (c) 800 min (d) 1200 min

PDF DOWNLOAD FROM : WWW.SARKARIPOST.IN


Page 107

Stage 2 Improve your Weaker Areas 115

Directions (Q. Nos. 16-22) Read the following two passages and answer the questions that
follow each passage. Your answers to these questions should be based on the passages only.

B Passage 1
Deficiencies in allocation in financial market are ignored by most traditional financial-market analysis
because of analysts’ inherent preferences for the simple model of perfect completion. Conventional
financial analysis pays limited attention to issue of market structures and dynamics, relative costs of
information and problems of income distribution. Market participants are viewed as acting as entirely
independent and homogeneous individuals with perfect foresight about capital-market behaviour.
Also, it is assumed that each individual in the community at large has the same access to the market and
the same opportunity to transact and to express the preference appropriate to his or her individual
interest. Moreover, it is assumed that transaction costs for various types of financial instruments
(stocks, bonds, etc.) are equally known and equally divided among all community members.

I N
16. The passage states that traditional studies of the financial market overlook imbalances in the
allocation of financial resources because
.
1. an optimum allocation of resources is the final result of competition among participants.

T
2. those performing the studies choose an over simplified description of the influences on
competition.
S
3. the analysts who study the market are unwilling to accept criticism of their methods as biased.

O
4. such imbalances do not appear in the statistics usually compiled to measure the market’s
behaviour.

(a) Only 1 (b) Only 2


I
Select the correct answer using the codes given belowP (c) 3 and 4 (d) Only 4

A R
17. A difference in which of the following would be an example of inequality in transaction costs
as alluded to in the last lines of the paragraph.
(a )
(b )
(c )
K
Maximum amounts of loans extended by a bank to businesses in different areas.

R
Fees charged to large and small investors for purchasing stocks.
Prices of similar goods offered in large and small stores in an area.
(d )

S A
Exchange rates in dollars for currencies of different countries.
18. According to the passage, analysts have conventionally tended to view those who participate
in financial markets as
1. basing judgements about events mainly on market structure.
2. varying in market power with respect to one another.
3. having equal opportunities to engage in the transactions.
Select the correct answer using the codes given below
(a) Only 1 (b) 1 and 2 (c) Only 2 (d) Only 3
19. The author’s main point is argued by
1. showing that the view opposite to the author is self-contradictory.
2. criticising the pre-suppositions of a proposed plan.
3. showing that omissions in a theoretical description make it inapplicable in certain cases.
Select the correct answer using the codes given below
(a) 1 and 2 (b) Only 2 (c) 2 and 3 (d) Only 3

PDF DOWNLOAD FROM : WWW.SARKARIPOST.IN


Page 108

116 Practice Set 5

B Passage 2
Political theories have, in fact, very little more to do with musical creation than electronics theories
have. Both merely determine methods of distribution. The exploitation of these methods is subject to
political regulation and is quite rigidly regulated in many countries.
Political party-influence on music is just censorship and performances can be forbidden and composers
disciplined for what they write, but the creative stimulus come from elsewhere. Nothing really ‘inspires’
an author but money or food or love. That persons or parties subventioning musical uses should wish to
retain veto power over the works used is not at all surprising. That our political masters (or our
representative) should exercise a certain negative authority, a censorship, over the exploitation of works
whose content they consider dangerous to public welfare is also in no way novel or surprising. Our
musical civilisation is older than any political party. We can deal on terms of intellectual equality with
engineers, architects, poets, painters and historians. We cannot be expected to take very seriously the

They can pay us to get emotional all right. Anybody can.

I N
inspirational dictates of persons or of groups who think they can pay us to get emotional about ideas.

Nothing is as emotion-producing as money. But emotions are factual; they are not generated by ideas.

.
On the contrary, ideas are generated by emotions. To have any inspirational quality there must be
present facts or immediate anticipations, not pie-in-the-sky. Now, pie-in-the-sky has its virtues as a

T
political ideal, I presume. Certainly most men want to work for an eventual common good. It is

S
notorious that musical description of hell, which is something we can all imagine, are more varied and
vigorous than the placid banalities that even the best composers have used to describe heaven and that

O
all composers do better on really present matters than on either, matters like love and hatred and
hunting and war and dancing around and around.

I P
20. The author implies that political doctrines usually fail to generate artistic creativity because
they are too
(a) abstract

A R
(b) rigidly controlled
21. The author is making a statement defending
(c) concrete (d) ambiguous

R K
1. intellectual freedom.
2. emotional honesty.

A
3. the apolitical stance of most musicians.
Select the correct answer using the codes given below

S
(a) Only 1 (b) Only 2 (c) 1 and 2
22. The author’s reaction to political influence on music is one of
(d) All of these

(a) deference (b) surprise (c) disbelief (d) resignation


23. Given below is a map to reach G from A via 5 cities. The number against each route denotes
the cost incurred between two cities. e.g. Travel between cities B and E costs ` 4. What would
be the least cost (in `) incurred to reach G from A?
4
B E
2 5
3
6 6
A D G
2
3 C F 4
5
(a ) 8 (b) 11 (c) 12 (d) 14

PDF DOWNLOAD FROM : WWW.SARKARIPOST.IN


Page 109

Stage 2 Improve your Weaker Areas 117

24. Examine the information given below.


˜Some engineers are bureaucrats. No bureaucrat is rich.
Which of the following is a valid conclusion regarding the above arguments?
(a) No engineer is rich (b) No bureaucrat is engineer
(c) Some engineers are rich (d) Some bureaucrats are engineers
25. Read the following passage carefully and answer the question based on it.
It is well known that the world urgently needs adequate distribution of food, so that everyone
gets enough. Adequate distribution of medicine is just as urgent. Medical expertise and
medical supplies need to be redistributed throughout the world so that people in emerging
nations will have proper medical care. This paragraph best supports the statement that
(a ) the majority of the people in the world has never been seen by a doctor
(b ) food production in emerging nations has slowed during the past several years
(c )
(d )

I N
many people who live in emerging nations are not receiving proper medical care
most of the world’s doctors are selfish about giving time and money to the poor
26. A father has three sons. He distributes some sweets and is left with 12 sweets. The number of

Find the initial number of sweets with the father.


T .
sweets each of them had initially was thrice the number of sweets eaten by them individually.

(a) 72 (b) 48 (c) 108

both games. How many people play atmost one of the two games?
O S (d) 54
27. In a club of 240 members, 122 people play cricket, 118 people play football and 40 people play

(a) 122 (b) 200

I P (c) 118 (d) 240


28. Two articles are sold for ` 13860 each. There is loss of 10% on one article and a profit of 10%

(a ) 0
29. Examine the following statement.
(b) 138.60

A R
on the other. What is the overall profit or loss (in `)?
(c) 280 (d) 272.20

R K
Poverty is increasing because the people, who are deciding how to tackle it, know
absolutely nothing about the poor.

(a )
(b )
(c ) S A
Which of the following courses of action logically follows for pursuing?
The decision makers should go to the grass root levels
The decision makers should come from the poorer sections of the society
A new set of decision makers should replace the existing ones
(d ) All of the above
30. The elements of the problem figures given below are changing with a certain rule as we
observe them from left to right.

According to this rule, which of the following would be the next figure, if the changes were
continued with the same rule?

(a) (b) (c) (d)

PDF DOWNLOAD FROM : WWW.SARKARIPOST.IN


Page 110

118 Practice Set 5

Directions (Q. Nos. 31-34) Read the following passage and answer the questions that follow.
Your answers to these questions should be based on the passage only.

B Passage
Colonialism was hated for its explicit assumption that the civilisations of colonised people were inferior.
This orientation was particularly blatant among the French. In their colonies, business was conducted in
French. Schools used that language and employed curriculum designed for children in France. In the
Metropole, intellectuals discoursed on the weakness of Arabic-Islamic culture. A noted historian accused
Islam of being hostile to science. There was of course an element of truth in the criticism. After all, Arab
reformists had been engaging in self-criticism for decades.
The reaction of colonised was of course to defend their identity and to label colonial policy, ‘cultural
asphyxia’. Throughout North Africa, nationalist made the defence of Arabic-Islamic civilisation a major

I N
objective, a value in whose name they demanded independence, yet the crisis of identity, provoked by
colonial experiences, has not been readily assured and lingers into the post-colonial period. Factors
producing militant and romantic cultural nationalism are anchored in time.

T .
Memories of colonialism are already beginning to fade and which the Maghreb has had a few decades in
which to grow dislocations associated with social change can also be expected to be fewer. A
preoccupation with identity and culture and an affirmation of Arabism and Islam have characterised the

O S
Maghreb, since independence these still remain important elements in North African life.
A second great preoccupation in independent North Africa is the promotion of a modernist social

P
revolution. The countries of the Maghreb do not pursue development in the same way and there have
been variations in policies within each country but all three spend heavily on development.

I
A problem, however, is that such advances are not always compatible with objectives flowing from North

R
African nationalism. In Morocco, for instance when the government decided to give children an ‘‘Arab’’
education, it was forced to limit enrolments because among other things, most Moroccans had been

A
educated in French and the country consequently had few teachers qualified to teach in Arabic. Two

R K
years later, with literacy rate declining, this part of the Arabisation programme was postponed.
31. The author provides information that would answer which of the following questions?
(a )
(b )
(c )
(d ) S A
When did colonialism end in North Africa?
Why did Europeans impose their way of life on their colonies?
Why was colonialism bad?
Why was colonialism disliked?
32. Which of the following does the author mention as evidence of cultural colonialism?
(a ) Native children in North Africa learned little about local culture
(b ) Science was not taught in the Arabic language
(c ) Colonialists spent little on development
(d ) Colonial policy was determined in France
33. Which of the following titles best describes the content of the passage?
1. Culture and Language.
2. Nationalism in North Africa.
Select the correct answer using the codes given below
(a) Only 1 (b) Only 2
(c) Both 1 and 2 (d) None of these

PDF DOWNLOAD FROM : WWW.SARKARIPOST.IN


Page 111

Stage 2 Improve your Weaker Areas 119

34. Which of the following is not used to present the author’s arguments?
1. Colonialism produced an identity crisis.
2. Decolonisation does not always run smoothly.
3. Colonialist assumed that local cultures were inferior.
4. Cultural nationalism will soon disappear.
Select the correct answer using the codes given below
(a) Only 1 (b) Only 2 (c) 1 and 3 (d) Only 4

Directions (Q. Nos. 35-37) Examine the information given below and answer the questions that
follow.
There are seven students in a class A, B, C, D, E, F and G. They sit on three benches 1, 2 and 3, such
that atleast two students are on each bench and atleast one girl and one boy on each bench. C, who is
a girl student, does not sit with A, E and D. F, the boy student, sits with only B. A sits on the bench 1
with his best friends. G sits on the bench 3. E is the brother of C.
35. How many girls are there, out of these seven students?
I N
(a ) 3
36. Who sits with C?
(b) 3 or 4 (c ) 4

T . (d ) 2

(a ) B (b ) D
37. On which bench, there may be two boys?
(a) Bench 2 or 3 (b) Bench 3
(c ) G

O
(c) Bench 2
S (d ) E

(d) Bench 1

I P
Directions (Q. Nos. 38-40) Examine the information carefully and answer the questions that follow.

A R
A company having 450 employees has sent all its employees for training in one or more areas out of
HRM, computer skills and financial skills. The employees are classified into two categories: officers
and clerks, who are in the ratio 4 : 5. 10% of the officers take training only in computer skills, 16% of
the clerks take training only in HRM, which is equal to the number of officers taking training only in

R K
financial skills and 50% of the number of officers taking training in HRM and financial skills both.
6% of the total employees take training in all the three, of which two-thirds are officers. 10% of the
total employees take training in HRM and computer skills both, which is five times the number of

A
clerks taking training in computer skills and financial skills. 10% of the clerks take training in HRM

S
and computer skills both. The number of officers taking training only in HRM is 25% of the number
of clerks taking training only in HRM. 20% of the total number of employees take training only in
computer skills. Number of clerks taking training in HRM and financial skills both is 20% of the
total number of clerks.
38. How many clerks take training in computer skills but not in HRM?
(a) 113 (b) 70 (c) 88 (d) 79
39. How many employees take training in financial skills but not in HRM?
(a) 106 (b) 135 (c) 162 (d) 134
40. What per cent of the total number of officers take training in computer skills but not in
financial skills?
(a) 11% (b) 40% (c) 25% (d) 15%
41. Manish has 7 relatives, 3 men and 4 women and his wife also has 7 relatives, 4 men and
3 women. In how many ways can they invite 3 men and 3 women for dinner, such that 3 are
Manish’s relatives and 3 are his wife’s relatives?
(a) 485 (b) 385 (c) 185 (d) 970

PDF DOWNLOAD FROM : WWW.SARKARIPOST.IN


Page 112

120 Practice Set 5

Directions (Q. Nos. 42-47) Read the following two passages and answer the questions that
follow each passage. Your answers to these questions should be based on the passages only.

B Passage 1
While many points are worth making in an evaluation of the single 6 year presidential term, one of the
most telling points against the single term has not been advanced. This kind of constitutional limitation
on elections is generally a product of systems with weak or non-existence political parties. Since there is no
party continuity or corporate party integrity in such systems, there is no basis for putting trust in the
desire for re-election as a safeguard against mismanagement in the executive branch. Better under those
conditions to operate on the basis of negative assumptions against incumbents. I do not know if the
earliest proposal for a single, non-repeatable term was made in the 1820s because that was a period of
severely weak political parties. But I do feel confident that this is a major reason, if not the only reason,
that such a proposal has been popular since the 1940s. Though the association of the non-repeatable

I N
election with weak political parties is not in itself an argument against the limitation, the fallout from this
association does contribute significantly to the negative argument. Single-term limitations are strongly
associated with corruption.

T .
In any weak party system, including the presidential system, the onus of making deals and compromises,
both shady and honourable, rests heavily upon individual candidates. Without some semblance of
corporate integrity in a party, individual candidates have few opportunities to amortise their obligations

O S
across the spectrum of elective and appointive jobs and policy proposals. The deals tend to be
personalised and the payoffs come home to roost accordingly. If that situation is already endemic in
conditions of weak or non-existent parties, adding to it the limitation against re-election means that

I P
candidates and officials, already prevented from amortising their deals across space, are also unable to
amortise their obligations temporally. This makes for a highly beleaguered situation. The single 6 year
term for President is an effort to compensate for the absence of a viable party system, but it is a

A R
compensation ultimately paid for by further weakening the party system itself.
Observers, especially foreign observers, have often noted that one source of weakness in American
political parties is the certainity of election every 2 or 4 years, not only because any artificial limitation

R K
on elections is a violation of democratic principles, but also because when elections are set in a certain
and unchangeable cycle, political parties do not have to remain alert, but can disappear into inactivity
until a known point prior to the next election. To rigidify matters by going beyond the determinacy of

S A
the electoral cycle to add an absolute rule of one term would hang still another milestone around the
neck of already doddering political parties.
42. Consider the following statements
According to the author which is true of a political system with weak political parties?
1. Politicians appoint unqualified people to important posts.
2. Political parties favour frequent election.
3. Political bargains are made by individual candidates.
Which of the statements given above is/are correct?
(a) 1 and 2 (b) 2 and 3 (c) only 3 (d) All of these
43. Suppose that America adopted a single-term political system. Considering the foreign observers
mentioned in the passage. How would they be expected to respond to such a development?
(a) They would endorse it because it further strengths American democracy.
(b) They would condemn it because it further limits American democracy.
(c) They would neither endorse nor condemn it.
(d) They would condemn it because it gives the President too much power.

PDF DOWNLOAD FROM : WWW.SARKARIPOST.IN


Page 113

Stage 2 Improve your Weaker Areas 121

44. Which of the following if true would most weaken the authors claim about single-term
political systems?
1. The discovery that foreign observers like this system.
2. The discovery that most politicians are honest.
3. The discovery that parliamentary systems are more democratic.
Select the correct answer using the codes given below
(a) 1 and 2 (b) 2 and 3 (c) only 2 (d) All of these

B Passage 2
It is true that the working of liberal democracy has almost universally been characterised by a gap
between its promises and its delivery. The promises made to ‘‘we the people’’ in idealistic constitutions

N
drafted in grand constitutional assemblies, made up of the most highly educated elites, have been denied
to the people in real life. We have now reached a stage where about 80 individuals own half the world’s

I
wealth and they have acquired it largely in the most advanced democratic set-ups. Yet, the option of

T .
delivering the promises made by the same liberal democratic constitutions is opening up. Several
experiments in Latin American states and societies, in Iceland and most recently, in Greece have
demonstrated the feasibility of looking after the interests of the poor and the downtrodden— and of the
ecology— within the available framework of ‘democracy’.

O S
In other words, the ideology that has guided these experiments is redefining Marxism itself by drawing it
away from the premise of irreconcilable class antagonism. The objective here is not to do away with the

I P
capitalist system but to compel it to fulfil the promises made by the ‘‘bourgeois’’ constitutions. Honesty of
purpose seems to have replaced the ideological commitment to class antagonism. This appears to be the
guiding perspective of the AAP. But, then perspectives do not evolve through deliberations among

A R
intellectuals. They evolve through interactions with the people and a deliberate as well as intuitive
understanding of their problems and aspirations. It would be interesting to investigate how many
epoch-making leaders, like Mahatma Gandhi and Nelson Mandela,developed their perspectives by

R K
reading learned treatises. As recent exciting experiments in direct democracy. In Iceland and earlier
Venezuela, have demonstrated perspectives traverse many diverse and complex terrains as they evolve.

S A
Will this experiment evaporate into thin air? There is no guarantee that it will survive the expected
onslaught of vested interests. But there is no guarantee that it will succumb to threats or temptations
held out by vested interests either. The AAP has always laid stress on the sincerity of intentions and
shown no sign of wavering from them, although it is too early to decide and power has the habit of
corrupting.
45. Consider the following statements
1. Experiments in the recent times in democracy are redefining marxism by drawing it away from
the premise of irreconcilable class antagonism.
2. Advanced democracies have failed to redistribute the wealth among the people.
Which of the statement(s) given above is/are incorrect?
(a) Only 1 (b) Only 2 (c) Both 1 and 2 (d) Neither 1 nor 2
46. Which of the following statements the author is most likely to agree with?
1. Perspectives in direct democracy are traversing across varied spectrum.
2. It is the sincerity of intentions and the strength of conviction that will determine the success or
failure of the AAP.
Select the correct answer using the codes given below
(a) Only 1 (b) Only 2 (c) Both 1 and 2 (d) Neither 1 nor 2

PDF DOWNLOAD FROM : WWW.SARKARIPOST.IN


Page 114

122 Practice Set 5

47. The author’s attitude towards the evolving political paradigm in democracy is,
(a) critical (b) indifferent (c) of cautions optimism (d) None of these
48. How many litres of water should be added to 60 L of acidic solution, so that the acid
concentration reduces from 60% to 40%?
(a) 15 L (b) 20 L (c) 30 L (d) 60 L
49. What digit will come in place of question mark?
5 8

3 28 2 8 42 ?

4 4

(a ) 3 (b ) 9 (c ) 1 (d ) 2

I N
Direction (Q. No. 50) Read the passage carefully and answer the question given below it.

.
Mathematics allows us to expand our consciousness. Mathematics tells us about economic trends,
patterns of disease and the growth of populations. Mathematics is good at exposing the truth but it

T
can also perpetuate misunderstandings and untruths. Figures have the power to mislead people.
50. This paragraph best supports the statement that
(a )
(b )
the study of Mathematics is dangerous
words are more truthful than figures
O S
(c )
(d )
I P
the study of Mathematics is more important than other disciplines
figures are sometimes used to deceive people

(a) 720
R
51. Amit and Sumit start from Delhi to Kanpur at 40 km/h and 60 km/h. If Sumit reaches Kanpur
4 h earlier than Amit, then the distance between Delhi and Kanpur (in km) is

A
(b) 360 (c) 640 (d) 480

R K
52. A zookeeper counted the heads of the animals in a zoo and found it is 60. When he counted the
legs of the animals he found it is 200. If the zoo had either peacocks or tigers, then how many

(a) 30

S A
tigers were there in the zoo?
(b) 45 (c) 20 (d) 40
53. A group of workers can do a piece of work in 20 days. But due to some reason, 8 of them went
absent and hence it took 20 days longer to complete the work. How many people actually
worked on the job to complete it?
(a) 18 (b) 16 (c) 24 (d) 20
54. If the ratio of difference between simple interest and compound interest at a certain rate of
interest for 2 yr and 3 yr is 16 : 5, then the rate of interest is
(a) 15% (b) 20% (c) 10% (d) 12%
55. A goat is tied to one corner of a field of dimensions 16 m × 10 m with a rope of 7 m long. Find
the area of the field that the goat can graze.
(a) 77 m 2 (b) 38.5 m 2 (c) 49 m 2 (d) 58.5 m 2
56. A box contains 5 brown and 4 white socks. A man takes out two socks. The probability that
they are of the same colour is
(a) 5/108 (b) 1/6 (c) 5/18 (d) 4/9

PDF DOWNLOAD FROM : WWW.SARKARIPOST.IN


Page 115

Stage 2 Improve your Weaker Areas 123

Directions (Q. Nos. 57-60) Read the following passage and answer the questions that follow.
Your answers to these questions should be based on the passages only.

B Passage
Just as no two individuals have the same fingerprint, no two people have identical theories on how to
achieve a goal or to live life from day-to-day. Some people follow religious outlines to aspire to a level of
moral excellence, while others employ other approaches. Towards the end of the 19th century and on
through the mid-20th century, a movement called existentialism was born a kind of ‘philosophical
theory of life’. Existentialism is not a philosophy, but instead has been likened as a label for several
widely different revolts against traditional philosophy. Existentialism rejects traditional ethical
endeavors, which is contrary to the tenants of many philosophers prior to the existentialism movement.
Philosophers, since the time of Aristotle have held that most people aim for a common peak of ethical
achievement.

I N
Aristotle argued for the existence of a divine being, described as the Prime Mover, who is responsible for
the unity and purposefulness of nature. In order for humanity to attain such a climax, everyone must

T .
imitate the Almighty’s perfect profile and strive to mimic his actions. Aristotle’s basic philosophy
deduces that humanity strives for an identical peak of moral excellence, as judged by a higher entity.
Existentialism declares that the individual must choose his way. Most importantly, there is no

standards. The universe does not predetermine moral rules.

O S
predetermination. Since, the universe is meaningless and absurd, people must set their own ethical

Each person strives toward a unique moral perfection. The 19th century Danish philosopher Soren

I P
Kierkegaard, who was the first writer to call himself existential, reacted against tradition by insisting
that the highest good for an individual is to find his uniqueness. Kierkegaard’s journal reads, ‘I must find
a truth that is true for me ... the idea for which I can live or die’. Existentialists believe that morality

A R
depends on the individual, rather than a supreme being. Existentialism asserts that people do not have a
fixed nature, as other animals and plants do. Our choices determine who we are.

K
The 20th century French philosopher Jean Paul Sartre proclaimed that the most important choices that
we make are those that directly affect ourselves. Each character makes choices that create his nature.

R
Existence suggests freedom where mankind is open to a future that is determined by choice and action.

S A
Choice is inescapable and central to human existence; the refusal to choose is a choice.
57. The author’s primary purpose of writing this passage is which of the following?
1. To explain a few tenants of the existential movement and to enumerate a few of the most
prominent existentialists.
2. To downplay existentialism and support classical divine philosophy.
3. To describe the shortcomings of Aristotle’s philosophies.
4. To explain existentialism and to discuss the philosophies of several prominent living
existentialists.
Select the correct answer using the codes given below
(a) Only 1 (b) 2 and 3 (c) Only 3 (d) Only 4
58. The author implies which of the following?
(a) Existentialism will not succeed because Aristotle’s philosophies are too deeply rooted into
today’s cultures
(b) Goals are attainable only if a person has a personal philosophy about life
(c) A peak of ethical achievement is attainable
(d) The universe is unpredictable and existentialism permits for humans to find meaning and
purpose in everyday activities

PDF DOWNLOAD FROM : WWW.SARKARIPOST.IN


Page 116

124 Practice Set 5

59. Which of the following is explicitly stated in the passage?


1. Existentialism is a philosophy.
2. The Prime Mover is Aristotle’s embodiment of a supreme being.
3. Soren Kierkegaard was well read and studied Aristotle’s philosophies.
Select the correct answer using the codes given below
(a) Only 1 (b) Only 2 (c) Only 3 (d) None of these
60. What is the purpose of writing the first sentence of the passage?
(a) To explain that we are all unique
(b) To explain that there are no two people who will ever have identical theories on how to set goals
(c) To explain that living life from day-to-day is difficult
(d) To introduce the concept of individualism, which later will help refute the idea that a common
philosophy cannot be applicable to everybody
61. Examine the following statement.

I N
Should people with educational qualification higher than the optimum requirements be
˜

debarred from seeking jobs?

question.
T .
Four arguments numbered 1, 2, 3 and 4 are given. On the basis of the information, answer the

Arguments

S
1. No. It will further aggravate the problem of educated unemployment.

O
2. Yes. It creates complexes among employees and affects the work adversely.

4. Yes. This will increase productivity.


(a) 1 and 3 are strong
I P
3. No. This goes against the basic rights of the individuals.

(b) All are strong (c) 2 and 4 are strong (d) Only 3 is strong

A R
62. Two men start from a point in opposite directions at speeds of 1.2 m/s and 1.5 m/s
simultaneously. Find the distance between them after 10 s.
(a) 27 m

R K (b) 75 m (c) 45 m (d) 65 m


63. Three dogs, Tom, Dick and Jerry, have 40 loaves of bread amongst them. If it is known that,
Jerry has as many as four times the number of loaves with Tom and also, 5 more than that of

S
(a ) 5 A
Dick. The number of loaves with Dick is
(b) 15 (c) 20 (d) 12
64. One fine morning, Rishu started to walk in such a way that his back is towards the Sun (his face
is opposite to the Sun). After covering some distance, he turned to left and then again to the left
and then after covering some distance, again to his left. Now, in which direction, is he facing?
(a) South (b) West (c) East (d) North

Directions (Q. Nos. 65-69) Read the following information carefully to answer the given
questions.
Six films P, Q, R, S, T and U are to be released on consecutive Friday. The schedule of the release is
to be in accordance with the following conditions.
Â
P must be released a week before T.
Â
R must not be released immediately after the first release.
Â
Q must be released on Friday following the Friday on which U is released.
Â
S must be released on fifth Friday and should not be immediately preceded by Q.
Â
T must not be released in the last.

PDF DOWNLOAD FROM : WWW.SARKARIPOST.IN


Page 117

Stage 2 Improve your Weaker Areas 125

65. Which of the following films preceded T?


(a ) P (b ) Q (c ) S (d ) U
66. Which of the following films is released immediately after Q?
(a ) P (b ) R (c ) T (d ) U
67. Film R cannot be released on which of the following Fridays in addition to second Friday?
(a) First (b) Third (c) Fourth (d) Fifth
68. In between which of the two films S is to be released?
(a) Q and T (b) R and T (c) P and T (d) T and U
69. Which of the following films released first?
(a ) P (b ) Q (c ) R (d ) U
70. Examine the information given below.
• ‘A * B’ means ‘A is the sister of B’.
• ‘A + B’ means ‘A is the father of B’.
• ‘A # B’ means ‘B is the mother of A’.
• ‘A = B’ means ‘B is the brother of A’.
I N
Which of the following means ‘M is the maternal uncle of N’?
(a ) M = P + Q * N
(c ) N * P # Q = M
T .
(b ) N + P = Q * M
(d ) M + Q = P # N
71. Select the missing numbers/ letters from the given responses.

O S
ii
iii

i
A

I P K

AD
R M
L
K

C
L

R
(a) (i—O), (ii—E), (iii—P)K M

(b) (i—N), (ii—E), (iii—O)

S A
(c) (i—M), (ii—E), (iii—N) (d) (i—P), (ii—F), (iii—Q)

Direction (Q. No. 72) Examine the information carefully and answer the following question.
In the following diagram, three classes of population are represented by three figures. The triangle
represents the school teachers, the square represents the married persons and the circle represents
the persons living in joint families.

E
B C
D

72. Married persons living in joint families but not working as school teachers are represented by
(a ) C (b ) F (c ) D (d ) A

PDF DOWNLOAD FROM : WWW.SARKARIPOST.IN


Page 118

126 Practice Set 5

Directions (Q. Nos. 73-77) The following questions are based on two passages in English to test
the comprehension of English language and therefore, these questions do not have Hindi version. Read
each passage and answer the questions that follow.

B Passage 1
Philosophical logic deals with formal descriptions of natural language. Most philosophers assume that
the bulk of ‘normal’ proper reasoning can be captured by logic, if one can find the right method for
translating ordinary language into that logic. Philosophical logic is essentially a continuation of the
traditional discipline that was called ‘logic’ before the invention of mathematical logic. Philosophical
logic has a much greater concern with the connection between natural language and logic.
As a result, philosophical logicians have contributed a great deal to the development of non-standard
logics (e.g. free logics, tense logics) as well as various extensions of classical logics (e.g. modal logics), and

I N
non-standard semantics for such logics. Logic and the philosophy of language are closely related.
Philosophy of language has to do with the study of how our language engages and interacts with our
thinking. Logic has an immediate impact on other areas of study.

T .
Studying logic and the relationship between logic and ordinary speech can help a person better structure
their own arguments and critique the arguments of others. Many popular arguments are filled with
errors because so many people are untrained in logic and unware of how to correctly formulate an
argument.
73. What is the central idea of this passage?
O S
(a )
(b )
(c )
P
The significance and application of philosophy in the study of logic

I
The history of logic and various types of their applications
Studying logic and philosophy together to get the maximum out of both subjects
(d )
R
The evolution of the branch of logic called philosophical logic and its significance

A
74. Which of the following options best summarises the passage given above?

and argument.

R K
(a) The philosophy of language should be studied alongside logic to discover logical errors in speech

(b) Philosophical logic was invented to create logical patterns in normal speech. The study of

S A
philosophical logic helps one to formulate argument better.
(c) Philosophical logic is the continuation of what was called ‘logic’ before the advent of
mathematical logic and deals with formal description of normal language.
(d) Philosophical logic has contributed a great deal towards development of non-standard logic like
tense logic, free logic, etc.

B Passage 2
India’s under-development was not traditional or inherited from the pre-colonial past. India of the
eighteenth century was undeveloped and not under-developed. On a world scale, it was not less but
perhaps more developed than many other national economies, as most of the world development has
occurred after the eithteenth century and basically after 1850.
The basic feature of colonialism in India in its long history since the 1750s was the appropriation by
Britain of the social surplus produced in India. There were of course, many changes and some of them
were positive for example, the development of the railways-when seen is isolation. But these changes
came within and as part of the process of under-development. Colonialism is best seen as a totality or a
unified structure.

PDF DOWNLOAD FROM : WWW.SARKARIPOST.IN


Page 119

Stage 2 Improve your Weaker Areas 127

75. Which of the following is the basic feature of colonialism?


(a) Interest on public debt (b) Unequal exchange
(c) Appropriation of social surplus (d) Expropriation of the industrial profit
76. Why 18th century India could not be called under-developed?
(a ) World development occurred after 18th century
(b ) India did not lag behind other nations
(c ) Most other nations developed after 1850
(d ) Indian Economy was comparable with contemporary Europe and Japan
77. Why was India’s under-development not traditional?
(a ) India in the 18th century was undeveloped
(b ) India did not inherit under-development from the past
(c ) India in the 18th century was not under-developed
(d ) India was not a poor nation

I N
Directions (Q. Nos. 78-80) Given below are three questions. Each question describes a situation

T .
and is followed by four possible responses. Indicate the response you find most appropriate. Choose
only one response for each question. The responses will be evaluated based on the level of
appropriateness for the given situation. Please attempt all questions. There is no penalty for wrong
answers for these three questions.

O S
78. You are working in a government department. Your boss gives a piece of work and orders you to

(a ) persuade him to extend the time


I P
follow his instructions to complete the work but the work is difficult to finish within time. You
will

(b )
(c )
(d )
R
start the work and try to finish it within the time limit, even working on holidays

A
start the work, try to finish it and if you cannot, then you ask your boss for time extension
take the help of other colleagues in finishing the work

R K
79. While on patrol duty, you come across a scene of eve teasing and molestation. When you go
near to the scene, you find that the molester is the son of a local MLA, who is a law abiding and

girl
S A
a very social person. You will
(a) enquire about the situation and carryout further investigations after ensuring the safety of the

(b) ignore the situation as the molester is the son of a law abiding MLA
(c) decide to let him go after giving him a warning as he is the son of a law abiding and dutiful
person
(d) call up your senior officer; narrate the situation and ask him what should be done
80. You are the police commissioner of a state. Honour killing is very common in your state.
Honour killing is the killing of sons or daughters, who get married in other clan due to the fear
of the Panchayats and to save their honour. One or more such cases are reported everyday.
What will you do to control such situation?
(a) Launch a campaign to open the minds of the people and to make them realise that honour
killing is a crime
(b) Order the police to arrest all those persons who are involved in such heinous crimes
(c) Talk to the people directly and warn them not to get involved in honour killing
(d) Give protection to the people who get married in other clan

PDF DOWNLOAD FROM : WWW.SARKARIPOST.IN


Page 120

Space for Rough Work

I N
T .
O S
I P
A R
R K
S A

PDF DOWNLOAD FROM : WWW.SARKARIPOST.IN


Page 121

Stage 2 Improve your Weaker Areas

Answer with Explanations


1. (b) India has emerged as a new key player as far as dealing 9. (d) Let the number of workers engaged in Manufacturing be x.
with climate change issues in concerned. Moreover a new 90°
Then, = 1980 …(i)
era of economic growth and development is sought while 360°
dealing with domestic issues related to climate changes 120°
and =x …(ii)
such as use of biofuels, clean energy sector revival and 360°
capitulation of solar energy sector. On dividing Eq. (i) by Eq. (ii), we get
90° 1980
2. (c) Cleanliness should be everybody’s endeavour. India can =
120° x
be a clean country only when people of all strata join 1980 × 120
together in this cleanliness drive and do not allow anyone ∴ x= = 2640
90
to litter at public places, Joint endeavour and awareness,
both are a must to deal with this problem.
10. (b) From the given pie chart,

3. (a) India is a one of the most favoured ground for tourists all
over the world because of diversity of culture and beautiful
I N
Number of Transportation workers =

and number of Professional workers =


30°
360°
20°
360°
of total workers

of total workers
but varied land scapes and gregarious tropical forests.
Focus on this industry can certainly speed up economic
development and bring prosperity to the country.

T .
Thus, it is clear that there are 50% more Transportation
workers than Professional workers.
So, if Professional workers = 20
4. (b) In the last lines of the first paragraph, it is mentioned that a
land transportation system would be substituted for
marine transportation during part of the movement of

O S Then, Transportation workers = 30


Given, the total income of Transportation workers
= ` 23760000
goods between Europe and the ‘Far East’. Marine refers to
sea and hence water transportation is the correct answer.
5. (b) 1 and 3 both options are correct. In the second paragraph
I P Average income of Transportation workers
=
23760000
30
= `792000

were deemed feasible it would open a completely new


market for both United States steamship lines and

A R
it is mentioned that if a land-bridge system of shipment
and average income of Professional workers
= 792000 ×
150
100

R K
railroads,which satisfy the statement 3.
In this paragraph it is also mentioned that the land bridge
will give the US East coast ships an opportunity to
= `1188000
Hence, total income of Professional workers
= 1188000 × 20

A
compete for the trade which American carriers could not
get before and is occupied by foreign lines that carry all

S
Far East -Europe freights which clearly says increase in
foreign trade statement 1 is now satisfied but no mention
of decrese in the amount of air freight.
= `23760000
11. (d) It is given that, P and Q can fill a tank in 36 min and 90 min.
LCM of 36 and 90 = 180
180 180
Now, filling rate of P and Q = +
6. (c) This question is developed from an idea implied in the 36 90
passage. Option (c) is correct because it is argued that = 5 + 2 =7 L / min
new jobs will be created for US railway workers and sailors After all pipes are opened, then tank deposits water at the
but not for dock workers. rate = 7 − 3 = 4 L / min

7. (a) In these type of questions, you may be asked about specific It is given that, tank filled up in 45 min.
facts or details the author has stated explicitly in the So, capacity of tank = 45 × 4 = 180 L
passage. It is mentioned in the passage that land-bridge 12. (a) Let the distance between the cities A and B be x km.
system would have a favourable effect on our balance of Then,
x
+
x
=5
payment. 60 40
120
8. (b) Average annual income from 2000 to 2005 ∴ x=5×
5
12 + 20 32
= = = ` 16 crore = 120 km
2 2

PDF DOWNLOAD FROM : WWW.SARKARIPOST.IN


Page 122

132 Practice Set 5

13. (c) LCM of 10, 15, 20, 25 and 30 22. (d) The author does not find censorship surprising, nor does he
take it seriously. He is resigned to attempts at censorship,
3 10, 15, 20, 25, 30
although he does not believe it can inspire creativity.
5 10, 5, 20, 25, 10
23. (b) The possible routes between A and G (along with the
2 2, 1, 4, 5, 2 respective costs in `) are given below
2 1, 1, 2, 5, 1 ABEG : 2 + 4 + 5 = `11
1, 1, 1, 5, 1 ADEG : 6 + 3 + 5 = `14
= 3 × 5 × 2 × 2 × 5 = 300 ADG : 6 + 6 = `12
ADFG : 6 + 2 + 4 = `12
After 300 s, bells toll together.
ACFG : 3 + 5 + 4 = `12
∴ Required time = 8:00 am + 300 s
Thus, the least cost is `11.
= 8:00 am + 5 min = 8:05 am
24. (d) ‘Some engineers are bureaucrats’ can be represented as
14. (c) It is given that, Sonu and Ramu can do a job in 60 and
40 days, respectively.
Engineers Bureaucrats
Thus, the ratio of their wages = 60: 40 = 3:2
It is given that, their total earning is ` 450.
∴Share of Sonu =
2
(2 + 3)
× 450 = `180
I N
‘No bureaucrat is rich’ can be represented as

15. (b) Volume of the tank = 22 × 10 × 7 = 1540 m3


Volume of water flowing per minute = Area of cross-section
of the pipe × Rate of flow of water per minute T .
Bureaucrats Rich

=
22  7 
7
× 
2
 × 100 = 1.54 m / min
 100 
3

O S On combining the two, we get


Rich

So, the tank will be filled in 1540/1.54 = 1000 min


16. (b) According to the passage, the studies ignore allocation
problems because of analysts’ inherent preferences for the
I P Engineers

Bureaucrats
is statement 2 i.e. option (b).

A R
simple model of perfect completion. So, the correct answer

17. (b) It is mentioned that transaction costs are the costs incurred
(i)
Rich

R K
when buying or selling assets takes place. In this case, the
costs incurred in buying or selling financial instruments
such as stocks. In other words, fees for purchasing stocks
are transaction costs. Different fees for large and small
Bureaucrats

A
investors are an appropriate examples of inequality in
transaction costs. So, the correct answer is option (b).

S
Maximum amount of loans, prices of goods and exchange
rate are not transaction costs for financial instruments.
Engineers
(ii)
So, in fig. (ii), it can be seen that ‘there are some
18. (d) Analysts assume that all market participants have the same engineers who are rich’. So, option (a) is wrong. Also in
access to the market and the same opportunity to engage
both figures, it is clear that ‘there are some bureaucrats,
in transactions. So, the correct answer is statement 3.
who are engineers.’ So, option (b) is invalid. ‘Some
19. (d) The passage mentions that the minority communities are a engineers are rich’ is valid from fig. (ii) but not from fig. (i).
practical exception to the theoretical rule and also explains Hence, option (c) is not a valid conclusion. Option (d) is
why this exception is typically omitted from consideration. valid in both figures.
Option (d) accurately describes the author’s approach.
25. (c) This answer is implied by the statement that redistribution
20. (b) The author implies that political doctrines usually fail to is needed so that people in emerging nations can have
generate artistic creativity because they are too rigidly proper medical care. Options (a) and (d) are not
controlled. Political influence on music is negative like mentioned in the passage. Option (b) is also incorrect as
censoring and disciplining the music which forbids any the passage indicates the distribution of medicine, not its
creativity. production.
21. (c) The author is arguing that musicians will not confirm to any 26. (d) Let the number of sweets initially with the father be x.
control over their creativity. Thus, they want to be x
Then, number of sweets each son initially =
intellectually free and emotionally honest. It does not mean 3
that they could not be active in politics (apolitical).

PDF DOWNLOAD FROM : WWW.SARKARIPOST.IN


Page 123

Stage 2 Improve your Weaker Areas 133

According to the question, Bench 2 → Fb , Bg

= 3  − 12  ⇒ Bench 3 → G
x x x
= x − 36
3 3  3
where, b → boy, g → girl
x 2x
⇒ x − = 36 ⇒ = 36 C is a girl and does not sit with A, E and D. So, she must sit
3 3
on bench 3. Also, E is the brother of C, which means E is a
∴ x = 54 male and he must sit on bench 1. Also, D sits on bench 1 and
27. (b) D should be a girl as atleast one girl and one boy should be
Cricket Football on each bench. So, G should also be a boy. Hence,
Bench 1 → Ab , Eb, Dg
122 40 118 Bench 2 → Fb, Bg
Bench 3 → Gb, Cg
35. (a) The number of girls are 3.
Total number of members = 240 (given) 36. (c) G sits with C.
But 40 members are playing both the games.
People who play atmost one of the two games
= 240 − 40 = 200
Solutions (Q. Nos. 38-40)

I N
37. (d) There may be two boys on bench 1 only.

The information given in the question can be represented as


28. (c) Total loss in percentage = x + y − xy

= 10 − 10 −
100
10 × 10
100
= − 1%
T .
Number of officers = 450 ×
4
(4 + 5)
Number of clerks = (450 − 200) = 250
4
= 450 × = 200
9

Now, SP of two articles = 2 × 13860 = ` 27720


and CP of two articles = 27720 ×
100
99
= ` 28000 S
Officers in computer skills =

O
Clerks in HRM =
16
10
100
× 200 = 20

× 250 = 40 = Officers in financial skills


∴Loss = 28000 − 27720 = ` 280
29. (a) The statement indirectly asserts that the decision makers
can work effectively to eliminate poverty, only if they get to
I P 100
Clerks in HRM = 50% of officers in HRM and financial skills
both
know the basic problems afflicting the poor people

A R
through interaction with them. So, only (a) follows.
30. (d) Triangle is present in middle and above while circle is
So, officers in HRM and financial skills both = 40 ×
100
50
Total employees taking training in all three = 6% of 450
= 80

K
changing anti-clockwise. The lines are also changing
anti-clockwise. Hence, option (d) is correct.

R
31. (d) This theme begins in line one and continues throughout 2
=
6
100
Officers in all three = × 27 = 18
× 450 = 27

A
the passage.Colonialism was hated because it assumed
the colonised subjects inferior and impose its own culture.

S
32. (a) The fact that children were taught very little about their own
culture and history was due to cultural colonialism. It was
3
So, clerks in all three = ( 27 − 18) = 9
Employees taking training in HRM and computer skills both

= 10% of 450 =
10
× 450 = 45
because of the colonised labelled colonial policy, ‘cultural 100
asphyxia’.
Employees in HRM and computer skills both = 5 × Clerks in
33. (b) Clearly, the main subject of the passage is nationalism. computer skills and financial skills both
45
34. (d) Statements 1, 2 and 3 are mentioned in the passage. So, clerks in computer skills and financial skills both = =9
Cultural nationalism is the seed to revive their identity from 5
the clutches of imperial culture. Clerks in HRM and computer skills both = 10% of 250 = 25
Solutions (Q. Nos. 35-37) Officers only in HRM = 25% of clerks only in HRM
25
It is given that, A sits on bench 1 and G sits on bench 3. i.e. = × 40 = 10
100
Bench 1 → A
Clerks in HRM and financial skills both = 20% of 250 = 50
Bench 2 →
Bench 3 → G 38. (b) Number of clerks taking training in computer skills but not
in HRM = 70
F is a boy student and sits with B only. So, they must sit on
bench 2 and B should be a girl. 39. (c) Number of employees taking training in financial skills but
Bench 1 → Ab not in HRM = 74 + 40 + 48 = 162

PDF DOWNLOAD FROM : WWW.SARKARIPOST.IN


Page 124

134 Practice Set 5

40. (a) enard against the onslanght of vested interests which will
be determined by sincerity of intentions.
Computer skills
HRM 47. (c) The author does not criticise the practice of democracy.
2 (O) On the contrary, he takes keen interest in evolving
10 (O) 20 (O)
16 (C)
40 (C) 70 (C) perspectives. Moreover he talks about its various
18 (O) O Officer
Clerks
determinants.
9 (C) C
62 (O) 48 (O) 48. (c)
41 (C) 0 (C) Acidic solution Water
60% 0%

40 (O)
74 (C) 40%

Financial skills
40 20
Required Percentage 40
Ratio = = 2 :1
=
Officers taking training in computer

(20 + 2 )
skills but not in financial skills
Total number of officers
× 100
20
Here, 2 parts = 60 L

I N
∴ Amount of water should be added = 1part = 30 L
=
200
× 100 = 11%

41. (a) 3 women and 3 men can be selected in the following ways
49. (c) As,

T .
(5 × 4) + (3 × 2 ) + 2 = 28
Same as, ( 8 × 4 ) + ( 8 × 1) + 2 = 42
Hence, 1 will be come in place of question mark.
Manish’s
relatives
(4 women and
Manish’s wife’s
relatives
(4 men and
Number
of ways
Value

O S
50. (d) This answer is clearly stated in the last sentence of the
paragraph. Option (a) can be ruled out because there is
no support to show that studying maths is dangerous.
Case I
Case II
3 men
3 men)

3 women
3 women)
3 women
3 men
3

4
C 3 × 3C 3
C3 × C3
4
1
16

I P Words are not mentioned in the passage, which rules out


option (b). Option (d) is a contradiction of the information
in the passage. There is no support for option (c).
Case III 2 men, 1 woman 2 women, 1 man 4
C1 × C 23

× 3 C 2 × 4C 1

A R 144 51. (d) Distance to be covered by Amit in 4 h = 4 × 40 =160 km


So, Sumit has gained 160 km over Amit at a relative speed
of 20 km/h, so for gaining 160 km, he has travelled for

K C 2 × 3C 1
4
Case IV 2 women, 1 man 2 men, 1 woman 324 =160/20 = 8 h
× 4C 2 × 3 C 1 ∴Distance between Delhi and Kanpur = (8 × 60)

A
∴Total number of ways = 485 R
42. (c) It was clearly stated in the opening lines of 2nd paragraph
= 480 km
52. (d) Let the number of tigers be x and the number of peacocks
be y, then

S
that in any weak party system including the presidential
system, the onus of making deals and compromises rests
heavily upon individual candidates.
43. (b) They would condemn it because it will further weaken the
and
x + y = 60
4x + 2 y = 200
...(i)
...(ii)
Multiplying Eq. (i) by 2 and then subtracting Eq. (i) from
Eq. (ii), we get
party system. 4x + 2 y = 200
44. (c) Author’s claim about single-term political system is based − 2 x ± 2 y = − 120
upon the nation that single-term limitations are strongly 2 x = 80
associated with corruption. So if it is discovered that most ⇒ x = 40
politicians are honest, then it will weaken his claim. Now, put the value of x in Eq. (i), we get
45. (d) Experiments in Latin America, Iceland and Greece have 40 + y = 60
been successful in looking after the interests of the poor ⇒ y = 20
and the downtrodden. The last sentence of the first ∴ x = 40 and y = 20
paragraph criticises the advanced democracies for their
failure to redistribute the wealth. 53. (b) Let n people actually worked on the job.
Then, total work = 20n
46. (c) The last sentence of third paragraph talks about the
Also, total work = (n − 8) 40
evolution of different perspectives. The AAP will have to

PDF DOWNLOAD FROM : WWW.SARKARIPOST.IN


Page 125

Stage 2 Improve your Weaker Areas 135

So, (n − 8) 40 = 20 n the performance of the individual that matters and that


⇒ 2 n − 16 = n ⇒ n = 16 makes him more or less wanted and not his educational
qualifications. So, neither 1 nor 2 holds strong.
54. (b) Let the rate of interest be r % per annum and the sum be ` P.
Besides, the needs of a job are laid down in the desired
Then, difference between CI and SI for 2 yr qualifications for the job. So, recruitment of more
 2  Pr 2
=  P  1 +
r  2 qualified people cannot augment productivity.
 − P − P × r × = 4
  100   100 10 Thus, 4 also does not hold strong. However, it is the right
of an individual to get the post for which he fulfills the
Also, difference between CI and SI for 3 yr
eligibility criteria, whatever be his extra merits. Hence,
 3   Pr 2  (300 + r )
=  P  1 +
r  3 Argument 3 holds strong.
 − P − P × r × =  4 
  100   100  10  100
62. (a) Relative speed of two men = 1.5 + 1.2 = 2.7 m/s
 Pr 2  (300 + r )
 4  Distance travelled by two men = 2.7 × 10 = 27 m
 10  100 (300 + r ) 16
Now, = 16 : 5 ⇒ = 63. (b) Let Tom, Dick and Jerry have x, y and z loaves of bread
 Pr 2  100 5
 4  with them respectively, then



 10 
1500 + 5 r = 1600
r = 20%


I N
x + y + z = 40, z = 4x and y = z − 5 = 4x − 5
x + 4x − 5 + 4x = 40
9x = 45 ⇒ x = 5
55. (b) Area which the goat can graze = 90 × 22 × 7 × 7 = 38.5 m2
360
56. (d) Total number of socks in the box = 5 + 4 = 9
7
64. (d)
T .
So, number of loaves with Dick = 5 × 4 − 5 = 15
N

2 socks can be drawn from the box in


9
C 2 ways = 36 ways
Number of ways of drawing 2 brown socks = 5C 2 =10
O S W E

Number of ways of drawing 2 white socks = C 2 = 6


4

∴Probability of drawing 2 socks of the same colour


I P S
It is clear from the figure, that he is facing towards North.
Solution (65-69) Given information can be summarised as

=
10
+
6
=
36 36 36 9
16 4
=

A R
= P( 2 brown socks) + P( 2 white socks) U>Q>P>T>S>R
1 2 3 4 5 6
65. (a) Clearly, the release of P precedes T.

R K
57. (a) The author’s primary purpose is to explain existentialism
and to support the thoughts of some prominent
existentialists. Statement 2 is against the theme of the
66. (a) P is released immediately after Q.
67. (d) R cannot be released on fifth Friday as well because S
has to be staged on that day.

S A
passage. Statement 3 cannot be substantiated from the
passage. Statement 4 is partially correct as author has
discussed the existentialist of nineteenth and twentieth
century. So, the correct answer is statement 1.
68. (b) Clearly, S is released between T and R.
69. (d) Clearly, U is released first.
70. (c) N * P means N is the sister of P.
58. (d) Option (a) cannot be concluded from the passage. The use
of the word ‘only’ makes option (b) incorrect. Option (c) is P # Q means Q is the mother of P as well as of N.
not stated in the passage. Author has mentioned that Q = M means Q is the sister of M.
universe is meaningless, and absurd so people must set Hence, M is the maternal uncle of N.
their own ethical standards.
71. (b)
59. (b) Statement 1 is not mentioned in the passage. Statement 3 is ii iii
A
not stated in the passage. The prime mover is Aristotle’s N i J K
divine philosophy. M
D K B
L
60. (d) Author has started with the concept of individualism, which M L
later paved the way for existentialism. C

61. (d) The issue discussed in the statement is nowhere related to


By going through inner circle, it is clear that each letter is
increase in unemployment, as the number of vacancies
increased by its next one.
filled in will remain the same. Also, in a working place, it is

PDF DOWNLOAD FROM : WWW.SARKARIPOST.IN


Page 126

136 Practice Set 5

+1 +1 +1 +1 76. (b) India of 18th century could not be called under-developed,


i.e. J → K → L → M → N
(i) because India did not lag behind other nations, rather it was
Thus, N will come at the place of (i). more developed than many other national economies.
Similarly, in upper circle we see that 77. (a) India’s under-development was not traditional because India
+1 +1 +1 +1 of the 18th century was undeveloped and not
A → B → C → D → E
( ii ) under-developed, it is proved from the fact that India was
more developed than many other economies.
+1 +1 +1 +1
and K → L → M → N → O
( iii ) 78. (c) Asking for extension of time before starting the work will
Thus, (i — N, ii — E, iii — O) prove to be of no use as you cannot know about the time
needed in finishing the work. So, the best way is to start the
72. (c) Married persons living in joint families are represented work and try to finish it within the time limit. In case, you
by the region common to the square and the circle i.e. D cannot complete the work, you can ask for the time
and B. But according to the given condition, the persons extension.
should not be school teachers. So, B is to be excluded.
Hence, the required condition is denoted by region D. 79. (a) As a public servant, it is your duty to enquire into such cases
and take appropriate action, otherwise citizens will stop
73. (d) Option (a) just mentions the significance and application
and not what philosophical logic means. Option (b) talks
about logic as a whole and not philosophical logic.
N
believing you. Option (b) is ethically and morally wrong and

I
your duty as a police officer does not permit such conduct.
Option (c) is in violation of your duty and a pretentious reason
Option (c) is a conclusion that can be made after
reading the passages. So, the options (a), (b) and (c)
are incorrect while only option (d) mentions the central
idea of the passage. Hence, option (d) is correct.
.
for letting an offender go. Option (d) shows your
indecisiveness and inability to handle situation.

T
80. (a) The best way to deal with such a situation is to launch a
74. (c) The passage summary have two basic points i.e. how
philosophical logic came into being and what is its
purpose, both of which are explained in the passage.
O S
campaign on a large scale to open the minds and to make
them realise that honour killing is a crime. This action will
have a large scale impact. Changing the mindsets of the
people on a large scale will prose a long-term solution.
Option (a) draws conclusion based on the passage.
Option (b) mentions both points however the first point
is incorrect while option (d) uses examples that
I P Options (b) and (c) are not practically feasible to administer
absolutely as the situation is rampant. Warning the public
may be felt as going against their traditional social believe

R
shouldn’t be used in summaries. Option (c) list both
these points down. Hence, option (c) is correct.

A
75. (c) The basic feature of colonialism in India was the
which may aggravate the situation and public may not
cooperate. So, option (a) is the best solution.

in India.

R K
appropriation by Britain of the social surplus produced

S A

PDF DOWNLOAD FROM : WWW.SARKARIPOST.IN

You might also like